{"instruction": "求一个整数的所有素数因子的思路是什么?", "input": "", "output": "1. 一般的小数可以用简单筛法找出质数列表,然后一个个试。这种方法简单暴力,但是对几亿以下的数字可以很快。\n\n2\\. 再大一点的数 $$n$$ 就用Pollard的 $$\\rho$$ 算法,思路:\n\n任取一个数 $$x_0$$ 开始,不断计算 $$x_k=(x_{k-1}^2+1) \\text{ mod } n$$ ,则如果 $$n$$ 有质因数$$p$$ ,那么 $$x_k \\text{ mod } p$$ 应该能更快地进入循环,可以用龟兔赛跑算法(图形状像 $$\\rho$$,因此算法得名)试图找出这个循环点,一旦找到 $$x_k\\equiv x_l \\text{ mod } p$$ ,立刻可以算 $$x_k-x_l$$ 和$$n$$ 的最大公约数得到分解。有一定几率失败。\n\n3\\. 更大的但在 $$10^{25}$$ 以下的 $$n$$ 可以用Lenstra椭圆曲线算法(ECM),思路:\n\n挑选椭圆曲线 $$y^2=(x^3+ax+b) \\text{ mod } n$$ 外加上面某一点 $$P$$ ,然后取一个较小的阶乘$$n!$$,反复用椭圆曲线加法算出 $$2!P, 3!P, \\cdots, n!P$$ ,在重复计算加法中每一步都会计算和曲线切线斜率 $$x/y$$同余的整数,即找出整数 $$z$$ 使得$$yz\\equiv x \\text{ mod } n$$ ,方法为找 $$y$$ 和 $$n$$的最大公约数,在这个过程中如果得出公约数大于1则分解成立,有一定几率失败。\n\n4\\. 更大的 $$10^{100}$$ 以下的 $$n$$ 可以用二次筛(QS),思路\n\n费尔马分解法试图把奇数 $$n$$ 写成 $$n=a^2-b^2$$ 的方法,这样立刻可得因数分解 $$(a+b)(a-b)$$,二次筛就是一种可以高效找到此类数字的方法,试图测试多个介于 $$\\sqrt n$$ 和 $$n-1$$ 之间的数 $$a$$ ,如果 $$a^2\\text{ mod } n$$ 正好是完全平方那就成了,不然找到几个不同的 $$a_i^2 \\text{ mod } n$$,如果乘积正好是完全平方数,那么 $$a=\\prod_i a_i \\text{ mod } n$$ 满足条件。\n\n5\\. 再往上只能用普通数域筛选法(GNFS),这个就很复杂了,思路\n\n也是用费尔马分解法,只是比二次筛更高效,核心定理:\n\n取系数为整数的 $$d$$ 次多项式 $$f(x)$$ 有复根 $$\\theta$$ ,并存在不是 $$n$$ 倍数的整数 $$m$$ ,使得$$f(m)$$ 为$$n$$ 倍数,将所有次数不超过 $$d-1$$的整系数多项式代入$$\\theta$$后的值定义为一个“环”(因为任何两个这样形式的数的和或积依然是这样形式的数),那么必然存在一个将这个环里的复数映射到不是$$n$$ 倍数的整数上的函数 $$\\phi(x)$$ 满足:\n\ni - $$\\phi(ab)=\\phi(a)\\phi(b)$$\n\nii - $$\\phi(a+b)=\\phi(a)+\\phi(b)$$\n\niii - $$\\phi(1)\\equiv1 \\text{ mod } n$$\n\niv - $$\\phi(\\theta) \\equiv m \\text{ mod } n$$\n\n已知这个定理后,如果找到整数对 $$(a_i,b_i)$$ 使得复数 $$\\beta = \\prod_i (a_i+b_i\\theta)$$ ,整数 $$y= \\prod_i (a_i+b_im)$$ ,以及 $$\\beta$$ 的映像 $$x=\\phi(\\beta)$$ ,则很容易根据i-iv得到$$x^2\\equiv y^2 \\text{ mod } n$$ ,有2/3机会能由此找到 $$n $$ 的一个约数。\n\n2009年,232位的RSA-768数通过上百台计算机耗时两年成功分解,用的就是高度优化后的GNFS。", "task_type": {"major": ["问答"], "minor": ["知乎问答"]}, "domain": ["通用"], "metadata": "qid:353400503, aid:878928931", "answer_from": "human", "human_verified": true, "copyright": "暂无版权及作者信息"} {"instruction": "两个人连续投一枚硬币,如果先出现“正正反”则甲胜,如果先出现“正反反”则乙胜,两人获胜概率是否一样?假设硬币质地均匀,每次投硬币正反两面概率都是50%。连续投很多次,直到出现“正正反”或“正反反”为止。", "input": "", "output": "第一次如果投反,那么甲乙都等于推倒重练,两人获胜概率不会有任何变化。\n\n所以现在只需要考虑第一次投正的情况,假设此时甲胜率为 $$x$$ ,乙为 $$y$$ ,那么有两种概率各位1/2的情况:\n\n1 - 第二次投出正,这时甲将100%获胜,因为这一刻开始接下来除非不断投出正,只要一出现反甲就赢了。\n\n2 - 第二次投出反,这时看第三投,有1/2概率乙胜利(正反反),剩下1/2概率出现正反正,对于甲乙来说胜率相当于第一次投出正的情况,甲为 $$x$$,乙为 $$y$$。\n\n综合\n\n$$x=\\frac{1}{2}\\cdot1+\\frac{1}{2}\\left(\\frac{1}{2}\\cdot0+\\frac{1}{2}x\\right)$$ ,得出$$x=\\frac{2}{3}$$\n\n或者\n\n$$y=\\frac{1}{2}\\cdot0+\\frac{1}{2}\\left(\\frac{1}{2}\\cdot1+\\frac{1}{2}y\\right)$$ ,得出$$y=\\frac{1}{3}$$\n\n两者相加为1,验算通过。", "task_type": {"major": ["问答"], "minor": ["知乎问答"]}, "domain": ["通用"], "metadata": "qid:584880907, aid:2904885349", "answer_from": "human", "human_verified": true, "copyright": "暂无版权及作者信息"} {"instruction": "数论的威尔逊定理已知有哪些证明方法?主要是想问必要性的证明,目前找到的有基于原根逆元的证法,有Lagrange证法,还有哪些证明方法?", "input": "", "output": "威尔逊有三种常见基础证明:\n\n1. 标准教科书证明(稍微拓宽到一般情况),可能就是题主所说的Lagrange证法\n\n设 $$p$$ 为奇素数( $$p=2$$ 明显),设 $$D$$ 与 $$p$$ 互质,讨论同余方程:\n\n$$xy\\equiv D\\bmod p$$\n\n设 $$x,y\\in\\\\{1,2,3,\\cdots,p-1\\\\}$$ ,因为 $$x,y$$ 与 $$p$$ 互质,对于每一个 $$x$$必然存在一个唯一的 $$y$$ 满足以上方程(不然 $$x(y_1-y_2)$$ 为 $$p$$ 的倍数,矛盾)。\n\n如果存在 $$x^2\\equiv D\\bmod p$$ (即 $$D$$ 为 $$p$$ 的二次剩余),那么根据 $$x^2-y^2=(x-y)(x+y)\\equiv 0\\bmod p$$ ,可得 $$y=p-x$$ ,即除了 $$x,p-x$$ 外的数都不可能满足$$x^2\\equiv D\\bmod p$$ 。\n\n所以 $$\\\\{1,2,3,\\cdots,p-1\\\\}$$ 中除 $$x,p-x$$ 外的数可以两两配对满足 $$xy\\equiv D\\bmod p$$,而 $$x(p-x)=px-x^2\\equiv -x^2\\bmod p\\equiv-D\\bmod p$$\n\n因此 $$(p-1)!\\equiv -D^{\\frac{p-1}{2}}\\bmod p$$\n\n代入 $$D=1$$ 就得到威尔逊定理。\n\n此外,代入 $$(p-1)!\\equiv -1\\bmod p$$ 可得 $$D^{\\frac{p-1}{2}}\\equiv 1\\bmod p$$ ,如果$$D$$ 是 $$p$$ 的二次非剩余根据以上步骤可得 $$D^{\\frac{p-1}{2}}\\equiv -1 \\bmod p$$ ,由此得到$$D^{\\frac{p-1}{2}}\\equiv\\pm 1 \\bmod p$$ 可以用来判定 $$D$$ 是否为 $$p$$的二次剩余,即鼎鼎大名的欧拉判定,由此可进一步推导如平方互反等重要结论。\n\n2. 原根\n\n取 $$p$$ 的原根 $$g$$ ,则 $$1,g,g^2,\\cdots,g^{p-2}$$ 除以 $$p$$ 的余数为$$1,2,3,\\cdots,p-1$$ 。则\n\n$$(p-1)!=g^{0+1+2+\\cdots+(p-2)}=g^{\\frac{(p-2)(p-1)}{2}}\\bmod p$$\n\n设 $$p=2k+1$$ ,则 $$(p-1)!\\equiv g^{k(2k-1)}\\bmod p$$\n\n根据费尔马小定律,已知 $$(g^k)^2=g^{2k}=g^{p-1}\\equiv 1\\bmod p$$ ,则 $$g^k\\equiv \\pm 1\\bmod p$$ ,但 $$kf\\left(\\frac{1}{1-x}\\right)=(a, b)(11$$ 时, $$\\frac{1}{1-x}<0$$ , $$0<1-\\frac{1}{x}<1$$ ,因此这个函数每套用一次,就会在$$(1,\\infty)\\rightarrow(-\\infty, 0)\\rightarrow(0,1)$$ 之间循环。\n\n* * *\n\n所以问题的第一个条件就是说,当 $$af(\\frac{1}{1-x})\\leq f(1-\\frac{1}{x})\\leq f(x)$$\n\n当 $$x\\geq b$$ 或 $$1 f(1-\\frac{1}{x})\\geq f(\\frac{1}{1-x})$$ 或者$$f(x)\\geq f(1-\\frac{1}{x})> f(\\frac{1}{1-x})$$\n\n* * *\n\n因此A和C肯定是错的,最小值永远不可能取在 $$x\\in(a,b)$$ 内,根据定义 $$f(x)>f(\\frac{1}{1-x})$$。最大值如果有2个的话,可能发生一种情况:即 $$x\\in (a,b)$$ 和 $$1-\\frac{1}{x}\\notin (a,b)$$ ,且$$f(x)=f(1-\\frac{1}{x})$$ 取到最大值,因此D是错误的。\n\n唯一正确的是B。\n\n* * *\n\n以上都没有考虑0和1,事实上很容易构造满足条件且 $$S=\\\\{0\\\\}$$或者 $$S=\\\\{0,1\\\\}$$ , $$T=\\\\{1\\\\}$$ 或者$$T=\\\\{0,1\\\\}$$ 的情况,这种情况下就连B也可以是错的。", "task_type": {"major": ["问答"], "minor": ["知乎问答"]}, "domain": ["通用"], "metadata": "qid:607407411, aid:3129542311", "answer_from": "human", "human_verified": true, "copyright": "暂无版权及作者信息"} {"instruction": "可不可以理解为所有事件的概率都是二分之一?可以不可以把一个事件最后的结果都翻译成,是与不是,能与不能,会或不会,这样所有事件发生的概率都是二分之一。", "input": "", "output": "首先看“所有事件的概率是二分之一”\n\n从数学角度来说:\n\n概率的数学表达为概率空间,一个概率空间由三样东西组成:样本空间、事件空间和概率函数。\n\n在有限样本空间的简单情况下用人话说就是有各种基本情况,以及一个会把这些情况的排列组合映射到一个0到1之间的数字的函数。\n\n例如,把骰子掷出6和骰子掷不到6看成两个基本情况,并约定概率函数把两种情况都对应于1/2。这是一个完备的概率空间,事实上,你很容易想象存在这样的骰子。\n\n但是概率函数的构造要满足一些基本规则。\n\n在上面的例子里,“掷不到6“的情况包括掷到1,掷到2,掷到3,掷到4,掷到5这些情况。如果你现在也要讨论这些事件发生的概率必须把它们加入概率空间(不然概率讨论就没有意义了),一旦这么做,根据概率函数的定义你会立刻得到“掷不到6“的概率为掷到1,掷到2,掷到3,掷到4,掷到5的概率总和,那么显然这5个概率不可能都是1/2和依然是1/2。\n\n再看“某一件事的概率是二分之一”\n\n这其实没什么毛病,我们说硬币正面向上概率1/2或骰子掷出6概率为1/6并不是基于任何概率论,而是基于对事件底层物理学的假设。所以说到底这也只是一种假设。\n\n例如看“我彩票中奖的概率为二分之一”,这取决于概率的现实意义。\n\n如果你认为自己买彩票的和别人买彩票的机制完全一致,那么买彩票就是一个发生了千百万次的事件,从概率两大学派之一频率学派的角度来看,你可以观察到所有买过这个彩票的人里面有多少中了奖,从而估计你自己彩票中奖的概率,这显然远远小于二分之一。\n\n如果你认为自己买彩票的和别人买彩票的机制并不一致,存在某种无法观察到的属性影响开奖几率(例如人品),那么如果你之前没买过彩票,这就是一个从未验证过是否会发生的事件,你完全可以假设自己彩票中奖的概率为二分之一。这里可以用第二个学派贝叶斯学派的观点,并没有任何逻辑否认你的假设。但是在这种情况下二分之一这个概率的意义也就只是如此,只是一种“信仰”。然后你去买彩票,每次不中(或者中,不影响结果)之后你会逐渐利用这些新信息更新你自己的“信仰”,最终你对于这件事的概率信仰会越来越低,趋近于上面第一种情况(假设人品属性实际不存在),或者趋近于另一个真实值。", "task_type": {"major": ["问答"], "minor": ["知乎问答"]}, "domain": ["通用"], "metadata": "qid:575605415, aid:2827138204", "answer_from": "human", "human_verified": true, "copyright": "暂无版权及作者信息"} {"instruction": "一瓶啤酒要一块钱,两个啤酒瓶能换一瓶啤酒,三个啤酒瓶盖也能换一瓶啤酒。一个人有10块钱,问:不能赊帐,理论上他最多能喝几瓶酒。", "input": "", "output": "如果把酒看作是一个连续的量,那么1个酒瓶可以换1/2瓶酒,1个酒盖可以换1/3瓶酒,所以1瓶酒可以换1/2+1/3=5/6瓶酒,这5/6瓶酒又可以换5/6x5/6瓶酒,以此类推,1瓶酒最终可以化为\n\n$$1+\\frac{5}{6}+\\left(\\frac{5}{6}\\right)^2+\\cdots=\\frac{1}{1-\\frac{5}{6}}=6$$瓶酒。\n\n当然,实际没有那么理想,比如最后剩下一个瓶盖你就不能再换酒了,以上是最后没有剩下任何酒瓶或酒盖情况下的结果,此时所有酒瓶和酒盖都“转化”了没有浪费。这个答案显然也是原题答案的上限,我们知道原题答案无论如何都不可能超过$$6n$$ 。而且我们知道理想状况下1个酒瓶可以换6x1/2=3瓶酒,1个酒盖可以换6x1/3=2瓶酒。\n\n现在重复以下步骤:\n\n把现有全部酒瓶和酒盖尽可能多全部换成新酒,喝光,然后收集酒瓶和酒盖。\n\n如果剩下的总酒瓶不足2个且总酒盖不足3个,停止,不然重复以上步骤。\n\n可见\n\n1 - 每一步结束时至少有1个酒瓶和1个酒盖。\n\n明显,每一步都会换至少1瓶新酒,所以每一步结束都会有至少1个酒瓶和1个酒盖。\n\n2 - 最后剩下1个酒瓶\n\n如1,如果剩下2个或以上还能再换\n\n3 - 最后剩下2个酒盖\n\n如果某一步开始时,酒盖数为 $$3k+2\\geq 2$$ ,则换完之后剩至少2个酒盖。\n\n如果某一步开始时,酒盖数为 $$3k+1\\geq 4$$ ,则酒盖能换至少1瓶新酒,加上剩下多余的1个酒盖,总酒盖数至少2。\n\n如果某一步开始时,酒盖数为 $$3k\\geq 6$$ ,则酒盖能换至少2瓶新酒,得到2个酒盖。\n\n如果某一步开始时,酒盖数为 $$3$$,则光用酒盖能换1瓶新酒,得到1个酒盖,根据1开始时你至少有1个空酒瓶,那么你此时总共有至少2个空酒瓶,循环不会停止,下一步一开始用2个空酒瓶可以再换1瓶新酒,酒盖数增加为2。\n\n所以最后停下来时你必然剩下1个酒瓶,2个酒盖,折合为“浪费”了7瓶酒的量,一共喝 $$6n-7$$ 瓶酒。\n\n因此,如果这个人有10块钱,则他最终能喝啤酒瓶数为$$6n-7 = 6 \times 10 -7 = 53$$", "task_type": {"major": ["问答"], "minor": ["知乎问答"]}, "domain": ["通用"], "metadata": "qid:553387768, aid:2672588607", "answer_from": "human", "human_verified": true, "copyright": "暂无版权及作者信息"} {"instruction": "假设一个人从15岁生了第一个孩子,接下来每五年生一个,四十岁生下最后一胎后不再继续。她的孩子也从15岁开始生孩子,每五年生一个,生到四十岁最后一胎不再继续。那么请问到这个人90岁时,他们家族一共有多少人?不考虑配偶只考虑孩子,中间不会有人死,只作为数学问题。", "input": "", "output": "设 $$F(x)$$ 为 $$x$$ 岁人的总子孙数目,\n\n则可得\n\n$$F(0)=F(5)=F(10)=0$$\n\n$$F(15)=1$$\n\n$$F(20)=2$$\n\n$$F(25)=3$$\n\n$$F(30)=4+F(15)=5$$\n\n$$F(35)=5+F(15)+F(20)=8$$\n\n$$F(40)=6+F(25)+F(20)+F(15)=12$$\n\n从40岁开始,\n\n$$F(x)=6+F(x-15)+F(x-20)+\\cdots+F(x-40)$$\n\n因此当 $$x>40$$ 时有递归公式\n\n$$F(x)=F(x-5)-F(x-45)+F(x-15)$$\n\n所以\n\n$$F(45)=F(40)-F(0)+F(30)=17$$\n\n$$F(50)=F(45)-F(5)+F(35)=25$$\n\n$$F(55)=F(50)-F(10)+F(40)=37$$\n\n$$F(60)=F(55)-F(15)+F(45)=53$$\n\n$$F(65)=F(60)-F(20)+F(50)=76$$\n\n$$F(70)=F(65)-F(25)+F(55)=110$$\n\n$$F(75)=F(70)-F(30)+F(60)=158$$\n\n$$F(80)=F(75)-F(35)+F(65)=226$$\n\n$$F(85)=F(80)-F(40)+F(70)=324$$\n\n$$F(90)=F(85)-F(45)+F(75)=465$$\n\n所以加上本人共466人。", "task_type": {"major": ["问答"], "minor": ["知乎问答"]}, "domain": ["通用"], "metadata": "qid:444371202, aid:1729687048", "answer_from": "human", "human_verified": true, "copyright": "暂无版权及作者信息"} {"instruction": "有五只麻雀偷到了一批大米,准备第二天早上去分。结果第二天第一只麻雀早到了,数了一下,自己吃了一粒,剩下的正好分五份,取走自己那份就先走了。第二只麻雀来了,数了数,也是吃了一粒,剩下的正好分五份,取走自己那份就走了。之后三只麻雀情况一样。那么请问,它们最少偷了多少粒大米?", "input": "", "output": "假设一开始有 $$X$$ 粒米,\n\n第一只麻雀吃剩下 $$\\frac{4}{5}(X-1)=\\frac{4}{5}X-\\frac{4}{5}$$\n\n第二只麻雀吃剩下$$\\frac{4}{5}\\left[\\frac{4}{5}(X-1)-\\frac{4}{5}\\right]=\\frac{4^2}{5^2}X-\\frac{4^2}{5^2}-\\frac{4}{5}$$\n\n以此类推\n\n第五只麻雀吃剩下$$\\frac{4^5}{5^5}X-\\frac{4^5}{5^5}-\\frac{4^4}{5^4}-\\frac{4^3}{5^3}-\\frac{4^2}{5^2}-\\frac{4}{5}$$\n\n以上五项的通项是\n\n$$\\frac{4^n}{5^n}X-\\sum_{i=1}^n\\frac{4^i}{5^i}=\\frac{4^n}{5^n}X-4\\left(1-\\frac{4^n}{5^n}\\right)=\\frac{4^n}{5^n}(X+4)-4$$\n\n很明显要让所有项都是整数, $$X+4$$ 必须是 $$5^5$$的倍数,最小是 $$5^5-4=3121$$ ,这么一来第五只麻雀吃剩下的就是$$4^5-4=1020$$ ,那么五只麻雀总共最少吃掉了 $$3121-1020=2101$$ 粒米。", "task_type": {"major": ["问答"], "minor": ["知乎问答"]}, "domain": ["通用"], "metadata": "qid:458776264, aid:1881240679", "answer_from": "human", "human_verified": true, "copyright": "暂无版权及作者信息"} {"instruction": "C++ 初始化列表展开,详细解释这个代码是什么意思?\ntemplate\nauto printf3(T value, Ts... args) {\n std::cout << value << std::endl;\n (void) std::initializer_list{([&args] {\n std::cout << args << std::endl;\n }(), value)...};\n}", "input": "", "output": "## 代码意图\n```\ntemplate\nauto printf3(T value, Ts... args) {\n std::cout << value << std::endl;\n (void) std::initializer_list{([&args] {\n std::cout << args << std::endl;\n }(), value)...};\n}\n```\n\nprintf3 函数的意图是将参数打印出来,一行一个。比如\n```\nprintf3(1, 2, 3, \"HelloWorld\");\n```\n的 输出结果为\n```\n1\n2\n3\nHelloWorld\n```\n\nprintf3第二行有点难懂,它同时结合了 C++ 的几个语法点\n\n- 初始化列表 initializer_list\n- 可变参数模版,Variadic Template\n- lambda 表达式\n- 逗号表达式\n\n## 初始化列表\n\n初始化列表,还是比较好 懂的,比如下面代码\n\n```\nauto tmp = {1, 2, 3, 4};\nfor (auto& v : tmp) {\n printf(\"%d\\n\", v);\n}\n```\n\ntmp 的类 型就是 std::initializer_list。初始化列表的值不能包含不同类型,下面代码会编译错误\n\n```\nauto tmp = {\"Hello\", 1, 2, 3}; // error\n```\n\n也不能是 void值,下面代码也会编译错误\n\n```\nauto tmp = {void, void, void}; // error\n```\n\n## 可变参数模版\n\n可变参数模版,就是模版中 typename... Ts ,和 Ts... args 这种语法。\n\n```\ntemplate\nauto printf3(T value, Ts... args)\n```\n\n可以将多个输入参数打包(Parameter Pack)到 args 中。比如 printf3(1, 2, 3, 4);,args 就是 2、3、4 三个参数 pack 到 args 中。在模板代码中,可以通过 .... 的语法将参数展开(Parameter Pack Expansion) ,比如下面代码\n\n```\ntemplate \nvoid doSomething(Ts... args) {\n auto tmp = {args...};\n for (auto &v : tmp) {\n printf(\"%d\\n\", v);\n }\n}\n```\n\n调用 doSomething(1, 2, 3);时,tmp 会展开成\n\n```\nauto tmp = {1, 2, 3};\n```\n\nargs...这种只是简单用法。模板参数还可以在更复杂的表达式中展开,比如\n\n```\nauto tmp = { (args * args)...};\nauto tmp1 = { std::sin(args)...};\nauto tmp2 = { (std::sin(args) + args)...};\n```\n\n调用 doSomething(1, 2, 3);时,tmp 会展开成\n```\nauto tmp = {(1 * 1),(2 * 2),(3 * 3)};\nauto tmp1 = { std::sin(1), std::sin(2), std::sin(3) };\nauto tmp2 = { (std::sin(1) + 1), (std::sin(2) + 2), (std::sin(2) + 2) };\n```\n\nC++ 碰到 ...,就会将包含 args 的表达式展开,可以在很复杂的表达式中展开。\n\n## lambda 表达式\n\n下面的 fn 就是先定义了 lambda 表达式,之后再进行调用\n```\nint a = 0;\nauto fn = [&a] {\n std::cout << a << std::endl;\n};\nfn();\n```\n\n也可以将 lambda 表达式定义和调用 写在一起。比如\n\n```\nint a = 0;\nauto b = [&a] {\n return a;\n}();\n```\n\n## 逗号表达式\n\n这个也比较简单。C++ 中 ,假如表达式用逗号分隔,会以此求值,最后一个值就是表达式的值。比如\n\n```\nauto a = (funA(), funB(), funC());\n```\n\n会以此求值 funA()、funB()、funC(),最终 a 为 funC 的值。\n\n## 拆解原始代码\n\n对几个语法点简单了解后,我们现在来拆解\n```\n(void) std::initializer_list{([&args] {\n std::cout << args << std::endl;\n}(), value)...};\n```\n\n首先,里面的复杂表达式\n```\n([&args] {\n std::cout << args << std::endl;\n}(), value)...\n```\n会按照模板参数依次展开,求值后, 生成一个std::initializer_list。\n比如调用为 printf3(1, 2, \"HelloWorld\"); value 值为 1,args 为 2、\"HelloWorld\" 两个参数中的打包。将包含 args 的表达式展开,展开的代码大致为\n```\nvoid printf3(int value, int arg0, const char *arg1) {\n std::cout << value << std::endl;\n (void)std::initializer_list{\n ([&arg0] {\n std::cout << arg0 << std::endl;\n }(), value),\n \n ([&arg1] {\n std::cout << arg1 << std::endl;\n }(), value)\n };\n}\n```\narg0 和 arg1 都分别嵌在逗号表达式中,于是分别调用了 lambda 函数,并 value 作为逗号表达式的值。于是 std::initializer_list放的都是 int 类型。\n各个模板参数展开,依次调用了 lambda 函数,打印出各自的值。\n\n## 一些细节\n\n- 代码中的 (void)std::initializer_list,这个 (void) 是为了防止编译警告,因为 initializer_list 没有被使用。\n- 先定义了 T value,再定义Ts... args,是让 printf3 最少包含一个参数,让 printf3() 调用报编译错误。\n- 使用逗号表达式,是 因为代码中,那个 lambda 函数返回值为 void, 而 std::initializer_list 是不合法的。这里也可以让 lambda 返回一个值,省去逗号表达式。比如\n```\ntemplate\nauto printf3(T value, Ts... args) {\n std::cout << value << std::endl;\n (void) std::initializer_list{([&args] {\n std::cout << args << std::endl;\n return 1;\n }())...};\n}\n```\n\n- 原始代码搞得比较复杂,也为了让 printf3 中的参数可以有不同的类型。比如下面的简单实现,就只能传入相同类型参数。\n```\ntemplate \nauto printf3(T value, Ts... args) {\n auto tmp = {value, args...};\n for (auto &v : tmp) {\n std::cout << v << std::endl;\n }\n}\n```\n\n- C++17 中有个 Pack fold expression 的语法,可以让实现更简单些\n```\ntemplate \nvoid printf3(T value, Ts... args) {\n std::cout << value << std::endl;\n ((std::cout << args << std::endl), ...);\n}\n```", "task_type": {"major": ["问答"], "minor": ["知乎问答"]}, "domain": ["通用"], "metadata": "qid:443285720, aid:1723184923", "answer_from": "human", "human_verified": true, "copyright": "暂无版权及作者信息"} {"instruction": "句柄是什么?", "input": "", "output": "句柄的英文是 handle。在英文中,有操作、处理、控制之类的意义。作为一个名词时,是指某个中间媒介,通过这个中间媒介可控制、操作某样东西。\n\n这样说有点抽象,举个例子。door handle 是指门把手,通过门把手可以去控制门,但 door handle 并非 door本身,只是一个中间媒介。又比如 knife handle 是刀柄,通过刀柄可以使用刀。\n\n跟 door handle 类似,我们可以用 file handle 去操作 file, 但 file handle 并非 file 本身。这个 filehandle 就被翻译成文件句柄,同理还有各种资源句柄。\n\n计算机领域很多英文词,直接从日常词中引申而来。比如 fork,日常用词就是个叉子,在 unix 中引申成创建新进程(进程分叉了)。socket日常用词是插座(连起来用于通电),引申成联网的标记信息(连起来用于通信)。英文是很日常,很容易理解的词,有时翻译成中文反而难以理解了。\n\n句柄这个翻译有点奇怪。据维基百科,句柄 的条目。\n\nDavid Gries所著的《Compiler Construction for Digital Computer》(1971)有句话\n\n> A handle of any sentential form is a leftmost simple phrase.\n\n该书中译本,《数字计算机的编译程序构造》(仲萃豪译, 1976 版)翻译成\n\n> 任一句型的句柄就是此句型的最左简单短语。\n\n这可能是句柄一词最早的出处。\n\n这里确实是在讨论句子。在这里句柄是个意译的合成词,两个字分拆开,“句柄”中的“柄”,用法就类似于,“刀柄”中的“柄”。用在此处是适当的。\n\n但以后将各种资源 handle, 都翻译成句柄时,就有点滥用了。\n\n具体到代码实现,handle 通常是某个数字标记,通过标记操作资源。这个标记在不同的场合有不同的叫法,有时叫 ID,有时叫描述符(descriptor)。在Windows 平台,就叫各种 handle 了。\n\n不要将 handle 简单地理解成编号、索引。比如分配 16 位的索引,再用 8 位密码将 16 位索引加密。之后将 4 位类型、4 位权限、8位密码、16 位加密索引打包成一个 32 位的整数作为 handle。这时说这个 handle 是索引就有点不适当了。\n\n用 handle 如何操作真正的资源,是实现的细节。handle 通常被实现为整数,也可以被实现成其他类型。\n\n广义来说,指针也是某种 handle,可以操作对象。但实际语境中,指针跟句柄是有区别的。初次接触到 handle (或者 id),很多人会有迷惑,为什么要用handle,而不直接用指针呢?\n\n1. 指针作用太强,可做的事情太多。可做的事情越多,就会越危险。接口设计中,功能刚刚好就够了,并非越多权限越好的。\n2. handle 通常只是个整数,实现被隐藏起来,假如直接暴露了指针,也就暴露了指针类型(有时也可以暴露 void* 指针作为某种 handle)。用户看到越多细节,其代码就越有可能依赖这些细节。将来情况有变,但又要兼容用户代码,库内部改起来就更麻烦。\n3. 资源在内部管理,通过 handle 作为中间层,可以有效判断 handle 是否合法,也可以通过权限检查防止某种危险操作。\n4. handle 通常只是个整数,所有的语言都有整数这种类型,但并非所有语言都有指针。接口只出现整数,方便同一实现绑定到各种语言。", "task_type": {"major": ["问答"], "minor": ["知乎问答"]}, "domain": ["通用"], "metadata": "qid:27656256, aid:943130123", "answer_from": "human", "human_verified": true, "copyright": "暂无版权及作者信息"} {"instruction": "为什么会有那么多人坚持黑客精神?", "input": "", "output": "黑客英文为 hacker,先来了解 hacker 一词的出处。\n\n60 年代初,麻省理工学院(MIT)有个学生团体,叫「铁路模型技术俱乐部」(Tech Model Railroad Club,简称TMRC)。俱乐部活动室摆着一个巨大的火车规划模型,俱乐部成员平常活动就是模拟铁路系统。\n\nTMAC 大致分两类成员。一类成员喜欢制作精美的火车、城市、景物模型。另一类成员更关心模型下面的信号和动力系统,比如火车如何调度、前进、停止。后来 MIT购买了计算机,后一类成员慢慢被计算机吸引过去,去研究计算机了。\n\n一群人聚成圈子,会很自然地发明出一些新词,让圈子外的人莫名其妙。\n\n当时 TMAC 的成员,将难题的解决方法称为 hack。巧妙、漂亮的解决方法叫做 cool hack 或者 neathack。笨拙、蛮干、不太优美但又确实有效的解决方法叫做 ugly hack 或者 quick hack。相应地完成 hack 的过程就叫做hacking,从事 hacking 的人就是 hacker,也就是中文的黑客。\n\n黑客一词本意指解决问题的人。因为 TMAC 的人后来很多都去了玩计算机,hack 就经常被特指解决计算机难题。\n\n黑客精神就是去解决问题,纯粹地、出于兴趣、不为功利地去解决问题。拥有黑客精神的就是黑客。按照这种宽泛的定义,黑客在各行各业中无处不在,人类的进步很大程度上就依赖这种精神。\n\n不为物质利益,只是纯粹地想知道原因,想解决某个问题,对于某些人可能是很难理解的。\n\n后来 hacker 一词被媒体误解了,媒体将非法盗窃信息,欺诈用户等等的计算机罪犯都叫成hacker。信奉黑客精神的那些人,不会承认那些搞破坏的为hacker,而是将他们叫为 cracker,中文翻译为骇客。\n\nTMRC 的成员后来去了世界各地,MIT 也被誉为黑客精神的发源地。\n\n具体到计算机,有六条黑客价值观:\n\n- 对计算机的访问(以及任何可以帮助我们认识世界的事物)都应该是不受限制,任何人都有动手尝试的权利。\n- 所有的信息都应该可以自由获取。\n- 不要迷信权威——促进分权。\n- 评价黑客的标准应该是他们的技术,而不是那些没有实际用途的指标,比如学位、年龄、种族、性别、职位。\n- 可以在计算机上创造出艺术和美。\n- 计算机可以让生活更美好。\n\n上面六条价值观很理想主义,不理解的甚至觉得有点傻。但在每个行业中想达到顶尖,就需要多少带点傻傻的理想主义。在中国会说到侠义精神,也带点傻气。那为什么有些人这样傻呢,明明知道可以更容易一点,还选这条路。这种牵涉到价值观的问题,很难解释。伟大点说,是这样可以让世界更美好;个人来说,这样遵循本心,觉得自在。\n\n真想知道得更详细,可以看:\n\n- 《黑客,计算机革命的英雄》\n- 《黑客与画家》\n\n以及一系列介绍 Unix 或者开源文化的书。", "task_type": {"major": ["问答"], "minor": ["知乎问答"]}, "domain": ["通用"], "metadata": "qid:30172484, aid:47077724", "answer_from": "human", "human_verified": true, "copyright": "暂无版权及作者信息"} {"instruction": "Map、Dictionary、Hash Table 有哪些异同?", "input": "", "output": "dictionary 跟 map 其实是同一个东西,只是在不同场合叫法不同。\n\ndictionary 的中文是字典,map在中文是映射,也有地图的意思。查字典,查地图,都是通过某个信息,去找到另一个信息。比如通过单词的拼写找到单词的具体含义。\n\n类比查字典过程,单词的拼写为 key, 单词的具体含义为 value。dictionary 就是通过key,找到value,有时也将 dictionary说成是 key-value 结构。只要达到查找目的,都可以叫做 dictionary。具体怎么找,可以有不同实现。\n\n比如,最简单是将 key,value 放在一起,线性排。\n\n```\nk1, k2, k3, k4, k4 ....\nv1, v2, v3, v4, v5 ....\n```\n\n当需要从 key 找到对应的 value 时,就从头到尾遍历过去。依次判断 k1, k2, k3, k4 是不是等于key, 当等于的时候,就找到 key的具体位置,从而也就找到了value。\n\n \n\n但这样从头到尾遍历,速度就太慢了,时间复杂度为 O(N)。N为数据的大小。\n\n为了快速从key找到value。dictionary(或者说map)的通常有两种实现方式。\n\n1. 二叉树\n2. 哈希(hash)表\n\n二叉树查找的时间复杂度为 O(logN),哈希表的时间复杂度大致为 O(1)。二叉树也分红黑树,AVL树等。哈希表的速度很快,很多语言内置的dictionary 都使用哈希表来实现,但它通常会浪费一些存储空间。这部分有兴趣去看数据结构的书。\n\nhash_map 其实就是使用 hash 表实现的 map。注意,二叉树,哈希表仅仅是 dictionary 的实现方式,不能说 hash 就等于dictionary,实现方式可以有多种多样。\n\n比如上面线性存储的实现,可以调整一下。当数据都放进来之后,先根据 key 来排序,再使用二分查找,就可以很快速地从 key 找到 value,这种实现简单快速,并省内存,有时会比 hash 的实现更好。适合于一些数据不会经常变动的情况。\n\nC++ 11 的标准库中有个 unordered_map,就是采用哈希表使用的 map。在C++11之前,没有标准的哈希实现,很多第三方库实现了哈希字典,基本都叫做 hash_map, 并应用广泛。所以C++11的实现就不能叫hash_map了,因为会造成很多现存的程序名字冲突。哈希实现的字典是无序,也就取了个不算太好的名字,unordered_map。", "task_type": {"major": ["问答"], "minor": ["知乎问答"]}, "domain": ["通用"], "metadata": "qid:27581780, aid:37398584", "answer_from": "human", "human_verified": true, "copyright": "暂无版权及作者信息"} {"instruction": "编程的函数和数学的函数为什么都叫函数?", "input": "", "output": "在数学上,最早将 function 一词翻译成函数,是清朝数学家李善兰。数学的 function指一个量随着另外的量的变化而变化,换句话说,一个量包含在另一些量当中。函数,也就是含数。中文的函,有盒子,用来装东西的意思,也可引申成包含。function在数学上翻译成函数也算贴切。\n\n最早研究计算机那批人,很多从数学系转过去的。编程中的 function也直接采用了数学上的翻译,其实有点误导。但也难怪,计算机设计之初就是用来解决数学问题。\n\n英文中 function, 也有完成一定功能,起作用的意思。编程语境中,function 一词更应该理解成,这代码完成某种功能。\n\n数学上的 function,是固定的输入,就对应固定的输出,中间不会改变状态。而编程上的function,只是完成某个功能,中间可以修改状态。假如类似数学上的意义,完全不修改状态,会叫成纯函数(pure function)以作强调。", "task_type": {"major": ["问答"], "minor": ["知乎问答"]}, "domain": ["通用"], "metadata": "qid:305981549, aid:555602200", "answer_from": "human", "human_verified": true, "copyright": "暂无版权及作者信息"} {"instruction": "什么是逆波兰表达式 ?", "input": "", "output": "逆波兰表达式,英文为 Reverse Polish notation,跟波兰表达式(Polish notation)相对应。之所以叫波兰表达式和逆波 兰表达式,是为了纪念波兰的数理科学家 Jan Łukasiewicz。其在著作中提到:\n\n> 我在1924年突然有了一个无需括号的表达方法,我在文章第一次使用了这种表示法。\n\n- 平时我们习惯将表达式写成 (1 + 2) * (3 + 4),加减乘除等运算符写在中间,因此称呼为中缀表达式。\n- 而波兰表达式的写法为 (* (+ 1 2) (+ 3 4)),将运算符写在前面,因而也称为前缀表达式。\n- 逆波兰表达式的写法为 ((1 2 +) (3 4 +) *),将运算符写在后面,因而也称为后缀表达式。\n\n波兰表达式和逆波兰表达式有个好处,就算将圆括号去掉也没有歧义。上述的波兰表达式去掉圆括号,变为 * + 1 2 + 3 4。逆波兰表达式去掉圆括号,变成 1 2 + 3 4 + * 也是无歧义并可以计算的 。事实上我们通常说的波兰表达式和逆波兰表达式就是去掉圆括号的。而中缀表达式,假如去掉圆括号,将 (1 + 2) * (3 + 4) 写成 1 + 2 * 3 + 4,就改变原来意思了。\n\n现实中,波兰表达式和逆波兰表达式,具体用于什么地方呢?\n\n波兰表达式(前缀表达式), 实际是抽象语法树的表示方式,比如中缀 (1 + 2) * (3 + 4) 编译时转成的抽象语法树为\n\n```\n *\n / \\\n + + \n/ \\ / \\\n1 2 3 4 \n```\n这个操作符就是根节点,操作数为左右子节点。我们将这棵树用符号表达出来,可以写成 (* (+ 1 2) (+ 3 4))。这实际就是 Lisp 的 S-表达式。S-表达式可看成将整棵抽象语法树都写出来,每层节点都加上圆括号。\n\n至于逆波兰表 示式,可用栈进行计算,天生适合于基于栈的语言。遇到数字就将数字压栈,遇到操作符,就将栈顶的两个元素取出计算,将计算结果再压入栈。比较典型的基于栈的语言为 Forth 和 PostScript。", "task_type": {"major": ["问答"], "minor": ["知乎问答"]}, "domain": ["通用"], "metadata": "qid:41103160, aid:452481026", "answer_from": "human", "human_verified": true, "copyright": "暂无版权及作者信息"} {"instruction": "iOS开发中,block与代理的对比,双方的优缺点及在什么样的环境下,优先使用哪一种更为合适?", "input": "", "output": "block 和 delegate 都可以通知外面。block 更轻型,使用更简单,能够直接访问上下文,这样类中不需要存储临时数据,使用 block的代码通常会在同一个地方,这样读代码也连贯。delegate更重一些,需要实现接口,它的方法分离开来,很多时候需要存储一些临时数据,另外相关的代码会被分离到各处,没有 block 好读。\n\n应该优先使用 block。而有两个情况可以考虑 delegate。\n\n1. 有多个相关方法。假如每个方法都设置一个 block, 这样会更麻烦。而 delegate 让多个方法分成一组,只需要设置一次,就可以多次回调。当多于3 个方法时就应该优先采用 delegate。\n\n比如一个网络类,假如只有成功和失败两种情况,每个方法可以设计成单独 block。但假如存在多个方法,比如有成功、失败、缓存、https验证,网络进度等等,这种情况下,delegate 就要比 block 要好。\n\n在 swift 中,利用 enum, 多个方法也可以合并成一个 block 接口。swift 中的枚举根据情况不同,可以关联不同数据类型。而在 objc就不建议这样做,objc 这种情况下,额外数据需要使用 NSObject 或者 字典进行强转,接口就不够安全。\n\n2. 为了避免循环引用,也可以使用 delegate。使用 block时稍微不注意就形成循环引用,导致对象释放不了。这种循环引用,一旦出现就比较难检查出来。而 delegate的方法是分离开的,并不会引用上下文,因此会更安全些。\n\n假如写一个库供他人使用,不清楚使用者的水平如何。这时为防止误用,宁愿麻烦一些,笨一些,使用 delegate 来替代 block。\n\n将 block 简单分类,有三种情形。\n\n* 临时性的,只用在栈当中,不会存储起来。\n\n比如数组的 foreach 遍历,这个遍历用到的 block 是临时的,不会存储起来。\n\n* 需要存储起来,但只会调用一次,或者有一个完成时期。\n\n比如一个 UIView 的动画,动画完成之后,需要使用 block 通知外面,一旦调用 block 之后,这个 block 就可以删掉。\n\n* 需要存储起来,可能会调用多次。\n\n比如按钮的点击事件,假如采用 block 实现,这种 block 就需要长期存储,并且会调用多次。调用之后,block也不可以删除,可能还有下一次按钮的点击。\n\n对于临时性的,只在栈中使用的 block, 没有循环引用问题,block 会自动释放。而只调用一次的 block,需要看内部的实现,正确的实现应该是block 调用之后,马上赋值为空,这样 block 也会释放,同样不会循环引用。\n\n而多次调用时,block 需要长期存储,就很容易出现循环引用问题。\n\nCocoa 中的 API 设计也是这样的,临时性的,只会调用一次的,采用 block。而多次调用的,并不会使用 block。比如按钮事件,就使用target-action。有些库将按钮事件从 target-action 封装成 block 接口, 反而容易出问题。", "task_type": {"major": ["问答"], "minor": ["知乎问答"]}, "domain": ["通用"], "metadata": "qid:29023547, aid:109570584", "answer_from": "human", "human_verified": true, "copyright": "暂无版权及作者信息"} {"instruction": "组合数公式的递归实现,如何分析其时间复杂度?\n递归形式实现的组合计数$C_n^r=C_{n-1}^r+C_{n-1}^{r-1}$\nint C(int n,int r){\n\tif(n==r||r==0) return 1;\n\telse return C(n-1,r)+C(n-1,r-1);\n}\n请问这个时间复杂度是如何计算的?", "input": "", "output": "答案是$$O(C_n^r)$$。\n\n要得到这个答案,并不需要解递归方程。注意到递归只有在值为1的时候才结束,而最终的结果$$C_n^r$$由许多个这样的1相加而来,那么递归树是一棵有$$C_n^r$$个叶子结点的二叉树,总递归次数是树中所有结点的个数$$2C_n^r-1$$,即$$O(C_n^r)$$。\n\n当然你要解递归方程也可以。设计算$$C_n^r$$需要的递归次数为$$T(n,r)$$,则有\n\n$$T(n,r) = \\left\\\\{ \\begin{array}{ll} 1, & \\text{if } n=r \\text{ or } r=0 \\\\\\T(n-1,r) + T(n-1,r-1) + 1, & \\text{otherwise} \\end{array} \\right.$$\n\n然后你发现这跟组合数的递推式本身非常相似,只是第二种情况中多了一个加1。\n\n稍加变换,设$$S(n,r) = T(n,r) + 1$$,则有\n\n$$S(n,r) = \\left\\\\{ \\begin{array}{ll} 2, & \\text{if } n=r \\text{ or } r=0 \\\\\\S(n-1, r) + S(n-1, r-1), & \\text{otherwise} \\end{array} \\right.$$\n\n那个1就被吸收掉了。容易看出$$S(n,r) = 2C_n^r$$,故$$T(n,r) = 2C_n^r - 1$$。", "task_type": {"major": ["问答"], "minor": ["知乎问答"]}, "domain": ["通用"], "metadata": "qid:38474425, aid:76641611", "answer_from": "human", "human_verified": true, "copyright": "暂无版权及作者信息"} {"instruction": "有哪些违背直觉的数学问题?", "input": "", "output": "例一、布丰投针问题\n\n在画有间隔为d的水平线的纸上投长为l的针(l 1972 年 1 月 26 日,22 岁的维斯娜在南斯拉夫航空公司一架 DC-9> 型客机上执勤,客机在捷克山区上空突然爆炸坠毁,只有维斯娜生还。维斯娜当时离爆炸中心较远,连人带椅被爆炸气浪掀出飞机外,从约 10058> 米的高空摔向地面,落在了一片雪堆中。维斯娜的头盖骨被撞裂成两半,三根脊椎骨粉碎,手腿也全部骨折,失去了知觉,但奇迹般地没有死亡,当救援人员发现她时,她仍然穿着空姐制服,不省人事地坐在航空座椅里。维斯娜在医院中昏迷了整整> 3 周,她的腰部以下也发生了暂时性瘫痪。后来她重新站起来,活到 66 岁。", "task_type": {"major": ["问答"], "minor": ["知乎问答"]}, "domain": ["通用"], "metadata": "qid:608007182, aid:3102040142", "answer_from": "human", "human_verified": true, "copyright": "暂无版权及作者信息"} {"instruction": "遗传病是不能治愈的吧!人类在疾病面前太渺小?", "input": "", "output": "不一定。个人在某些疾病面前可能显得渺小,但人类整体几乎不可能被任何疾病“战胜”。许多遗传病只影响千分之一或更少的人口。\n\n截止 2023 年,人群中已知的遗传病有 6000 多种,其中 600多种可以治疗。一部分治疗方案可以让特定遗传病患者的预期寿命、预期健康寿命、预期后代数量与常人无异。\n\n你可以在 [Treatments for geneticdisorders](https://www.rx-genes.com/) 等由医生和科研人员维护的网站查找已知范围内可治疗的遗传病及其治疗方案。\n\n饮食管理可控制近百种对特定分子代谢不良的遗传病,患者可在避免食用这类分子的同时保持与常人相仿的健康水平。\n\n酶替代疗法可控制数十种缺失特定酶的遗传病。\n\n营养补充剂可控制上百种对特定营养物质吸收率不足或缺失特定代谢产物的遗传病。\n\n免疫球蛋白可控制数十种遗传性免疫异常。\n\n药物、疫苗、血液制品可控制数百种特定遗传病。\n\n手术可通过“为一百多种遗传性造血干细胞缺陷患者移植正常造血干细胞、为一些遗传性骨硬化症患者移植正常骨髓、为一些缺失特定酶的遗传病患者移植正常骨髓等组织、为遗传性特定脏器异常的患者移植正常器官、切掉家族性结肠息肉病患者的结肠、摘掉高风险乳腺癌相关基因携带者的乳腺、为遗传性耳聋患者植入人工耳蜗、为布鲁格达氏综合征患者植入除颤器”之类手段来对付基因层面的遗传病。\n\n人们已经知道数千个与遗传病有不同程度关联的基因,可以针对其中一部分进行基因治疗。\n\n- “消除患者的突变基因”不是治愈患者的遗传病的唯一标准。基因治疗将患者的一部分细胞的基因型变正常到一定地步或引入其他有用基因,就可能让患者的症状大幅减退乃至性状与常人无异。\n- 针对 β-地中海贫血、脊髓性肌萎缩症、腺苷脱氨酶缺乏症、RPE65 相关的莱伯先天性黑蒙等的基因治疗正在进行人体临床试验。\n\n一些人将发育性先天缺陷也当成了“遗传病”,它们经常可以被手术充分修复。例如手术可以矫正腭裂、修复一些先天性心脏缺陷。", "task_type": {"major": ["问答"], "minor": ["知乎问答"]}, "domain": ["通用"], "metadata": "qid:609625492, aid:3102802321", "answer_from": "human", "human_verified": true, "copyright": "暂无版权及作者信息"} {"instruction": "把受精胚胎植入无子宫的小白鼠体内,为什么会在发育一半的时候停止生长?", "input": "", "output": "未必。\n\n题目里的“受精胚胎”未说清楚是受精卵还是胚胎,可能来自糟糕的媒体或转述错误。\n\n题目里的“发育一半”所指不明。小白鼠的正常孕期为 18 到 20 天。如果题目的意思是这胚胎能发育到 9 至 10天的水平,那么你将这胚胎手术取出、以现代技术支持,就有一定概率能将其培育成存活个体了。这主要受你的医疗技术影响。\n\n- 截止 2023 年 3 月,人类早产存活领域的吉尼斯世界纪录保持者是 Curtis Zy-Keith Means,他于 2020 年 7 月 5 日出生在美国阿拉巴马,胎龄 148 天,早产 132 天而存活。\n- 目前,小鼠的极早产护理缺少成熟的医疗经验,可预期死亡率更高。\n\n胎盘从周围组织取得营养的效果受具体的组织特性影响,并非什么组织都合适,但确实不限于子宫内膜。\n\n无子宫的母体可能不擅长分泌妊娠过程所需的孕酮等分子,可以人工添加或将动物连接在正常怀孕的雌性个体身上来获取相关分子——对于无免疫排斥的特定实验鼠,可以手术连接循环系统;对于有免疫排斥的动物,可以由透析机间接连接循环系统。\n\n不限于小白鼠,人体内特定部位的宫外孕允许足月妊娠、产生活后代。\n\n- 近年来,宫外孕占人所有妊娠的 1% 到 2%;\n- 人 95% 的宫外孕发生在输卵管;\n- 腹腔妊娠占宫外孕的 1% 左右,21.4% 至 40% 的腹腔妊娠胎儿畸形、取出后能存活一周的不到一半,腹腔妊娠导致的孕妇死亡率约 0.5% 至 8%;\n- 2003 年,约旦研究人员报告一名妇女通过剖腹产取出腹腔妊娠的正常后代;\n- 2008 年,一名加纳妇女通过剖腹产取出腹腔妊娠的正常后代,其胎盘广泛粘附于母体的大肠、大网膜、子宫左角。\n- 2009 年,一名印度妇女通过剖腹产取出腹腔妊娠的正常后代。产妇患有单角子宫并发生子宫破裂,部分胎盘附着在破裂的子宫角上,大部分胎盘附着在腹膜上。这比没有子宫更危险。\n- 2011 年,沙特阿拉伯研究人员报告一名妇女通过剖腹产取出腹腔妊娠的正常后代,胎盘从附着的左侧阔韧带上剥离时引起产妇大出血并需要输血。", "task_type": {"major": ["问答"], "minor": ["知乎问答"]}, "domain": ["通用"], "metadata": "qid:609902920, aid:3103182372", "answer_from": "human", "human_verified": true, "copyright": "暂无版权及作者信息"} {"instruction": "地衣裂开石头需要多久?", "input": "", "output": "地衣附着在石头上后 1 到 4 年可测到少量剥蚀,效果是维管植物的十分之一或更弱,与岩石类型、地衣类型、当地气候和天气事件等因素有关。\n\n- McCarroll 和 Viles 于 1995 年在挪威南部测得地衣引起的岩石剥蚀速率至少约 0.0012 毫米每年。这是当地气候本身引起的风化速率的 25 到 50 倍。相关研究可能低估了某些地衣造成的影响。\n- 夏威夷火山爆发形成的玄武岩被地衣覆盖后 50 年间剥蚀了约 0.5 毫米。这不完全是地衣造成的。\n- 地衣影响下的砂岩每 100 年剥蚀约 3 毫米。这也不完全是地衣造成的。\n\n至少一部分地衣的菌丝可渗透到岩石中的显微缝隙,在空气湿度和降水·结冰影响下反复胀缩、释放盐和有机酸,引起机械破坏并加快化学风化。实地考察显示一些地方的地衣渗入岩石的深度约1.12 到 3.21 毫米。这些渗透至少要花数百年来真正“裂开”少量岩石。\n\n在潮湿地区,蜗牛取食石头上的苔藓时造成的剥蚀比地衣更快。", "task_type": {"major": ["问答"], "minor": ["知乎问答"]}, "domain": ["通用"], "metadata": "qid:611032138, aid:3114321689", "answer_from": "human", "human_verified": true, "copyright": "暂无版权及作者信息"} {"instruction": "人体能感觉到的温差最小是多少?", "input": "", "output": "实验显示,人拇指根部的无毛皮肤可以感知两个冷却脉冲间 0.02 到 0.07 摄氏度的温差、两个加热脉冲间 0.03 到 0.09 摄氏度的温差。\n\n检测皮肤温度变化的灵敏度比这要低、受本身温度和温度变化速率影响。\n\n- 人拇指根部的无毛皮肤在本身温度 33 摄氏度时可检测到的最小升温幅度约 0.2 摄氏度、可检测到的最小降温幅度约 0.11 摄氏度。\n- 皮肤温度每秒变化 0.1 摄氏度以上时可以迅速产生感觉,每分钟变化不到 0.5 摄氏度时 8 到 10 分钟都未必被感觉到。\n\n以上数据引自 Lynette Jones (2009), Scholarpedia, 4(5):7955.\n\n人手掌皮肤在不同的实验步骤下可感知两个冷却脉冲间 0.02 到 0.06 摄氏度的温差,敏感度在手掌皮肤温度 29 到 39 摄氏度时变化不大。", "task_type": {"major": ["问答"], "minor": ["知乎问答"]}, "domain": ["通用"], "metadata": "qid:611196947, aid:3111785055", "answer_from": "human", "human_verified": true, "copyright": "暂无版权及作者信息"} {"instruction": "长得好看到底是怎么成为人类择偶标准的?相貌美丑在物种角度有什么作用?", "input": "", "output": "外观可以反映一些遗传要素、发育情况、当前健康状况、生活水平,在人类诞生前就影响着动物的交配策略和种内关系,参与了整个人类演化史。人的文化对此推波助澜。\n\n不过,对长期共同生活的社会化一夫一妻制物种来说,交配策略不等于“择偶标准”,人的婚姻在历史上大部分时间里首先是经济关系,你可以看看现代的离婚诉讼在谈些什么。\n\n- “长得好看”是一些人“理想中的择偶标准”的组成部分,但经常被他们自行放弃。现代社会只有约 7% 到 8% 的人被所处的社会公认为“美丽”,大部分结婚的人找了平均水平或低于平均水平的配偶。\n- 人类女性对男性外表的关注度平均而言低于人类男性对女性外表的关注度。这可以解释为女性被自然选择塑造为会关注长期一起生活的安全性、为生育后代提供资源的能力,也可以解释为文化塑造,或者二者兼而有之。\n- 许多动物会优待它们觉得“长得好看”的个体,对此类个体的生存、繁衍和此类性状的遗传有利;同时,它们还可能攻击它们觉得“长得畸形”的个体。人也如此。\n- 人类女性的丰乳肥臀、嘴唇饱满、高颧骨、大眼睛、下巴纤细、腰部较细可以反映较高的雌激素水平。人类男性的宽肩膀、宽下巴、大胡子等可以反映较高的雄激素水平。“长得好看”,尤其是脸好看,是生存力、生育力的招牌。不过,男性脸上的较大疤痕在不特别可怕的情况下可以显示此人从伤害中恢复的能力,经常被女性评为“有男子气概”,提高他们短期寻求交配对象的成功率。对于长期共同生活,女性通常认为没疤的脸显示此人更温和、相处的安全性略高。\n\n人对相貌“美丑”的定义和态度受人的生物学特性、人的生活经历和文化影响。\n\n- 自古以来,人们普遍认为外观美丽或“有魅力”的人比平均水平更擅长交际、更热情、更有统治力、更聪明、心理更健康。通常,外观美丽的人会在周围的人追捧下潜移默化地变得比平均水平更自信,年收入也平均而言更高——不过,他们也比平均水平更容易拖欠贷款。\n- 外观丑陋的人(尤其是体力也不强的人)经常受到恶劣对待,影响他们的性格,更促进众人形成刻板印象。他们的年收入平均而言比同样工作能力的其他人少 4% 到 13%。\n- 在食物较为短缺的地区,人们可能觉得更胖的身材较美;在食物过剩、到处是胖子的地方则相反。\n- 实验显示,比较对称的脸、接近自己见过的人脸的平均状态的“平均脸”对人更有吸引力。前者可以展示遗传和发育的状况、跟智力也有关,后者的数据库是在生活过程中构建的、可以不断变化。\n- 当你和某人建立了情感联系,你的脑会觉得此人的外貌比过去更具吸引力、更好看。这不要求双向联系。\n- 对近 300 家尼德兰广告公司进行的研究显示,高管长得更好看的公司的年收入更多。\n\n无论如何,人经常高估自己的外貌,2017 年全球范围内 60% 的女性受访者觉得自己很漂亮。", "task_type": {"major": ["问答"], "minor": ["知乎问答"]}, "domain": ["通用"], "metadata": "qid:556125322, aid:2696160834", "answer_from": "human", "human_verified": true, "copyright": "暂无版权及作者信息"} {"instruction": "灵长类动物通过培训为人类服务从事简单的生产制造?", "input": "", "output": "可以,现实中早就有人训练猴子采摘椰子,19 世纪南非有 1只知名的豚尾狒狒从事雇佣劳动,诸如此类。不过,对于复杂到一定地步的工作,雇人来做可能比训练动物还要便宜。\n\n2020 到 2022年,一些欧美动物保护组织和媒体猛烈攻击泰国椰子产业虐待动物、让猴子从事“奴隶劳动”,一些西方企业停止使用来自泰国的椰子。反对这类攻击的人争辩说,泰国人为猴子提供食物、饮用水、按摩、梳理、清洗、清理爬到身上的昆虫,马来西亚、印度、斯里兰卡等地的一些农民也训练猴子采摘椰子,而且古埃及人就曾训练狒狒采摘水果和坚果,这和用牛耕田没有什么区别。\n\n1880 年代,南非开普敦-伊丽莎白港铁路信号员 James Wide在受伤并双腿截肢后教他的宠物狒狒“杰克”操控机器。其他人很快注意到一头狒狒在控制铁路信号,铁路公司随后宣布雇佣“杰克”,每天给它 20美分工资和食物,每周给它半瓶啤酒。9 年间,据说“杰克”从未发生工作错误,火车乘客经常给“杰克”食物。1890 年,“杰克”死于肺结核。\n\n更多的人声称自己的宠物猴、宠物狒狒可以胜任大量的工作,可以教自己的孩子做一些事情、照顾孩子和狗。", "task_type": {"major": ["问答"], "minor": ["知乎问答"]}, "domain": ["通用"], "metadata": "qid:536657383, aid:3048235418", "answer_from": "human", "human_verified": true, "copyright": "暂无版权及作者信息"} {"instruction": "像美国印度这种阶级分明的国家抗疫如此糟糕,那阶级死化的集群生物,蜜蜂蚂蚁是怎么解决传染病问题的呢?", "input": "", "output": "蜜蜂、蚂蚁的不同物种有不同的社会形态,阶级并不一定固化,一部分物种没有阶级分化。这不怎么重要。\n\n在发生流行病的时候,A. custodiens进行隔离,让感染疾病的蚂蚁不再进入繁殖阶级和幼虫·蛹的居住空间,并通常让它们居住在靠近出口(可以通风)的巢室中;进行更频繁、更密集的互动来对密切接触感染者的个体进行清洁和消毒,及时清理废物和尸体。这些是社会性昆虫的典型防疫措施。对疑似感染者进行清洁的个体有被传染的风险,但有更大概率在接触少量病原体后经由无症状感染或轻症感染而获得免疫。\n\n一些蚂蚁的酸性分泌物可以杀死许多病原体,它们会将这类分泌物抹在身上。C. fulvopilosus会保持社交距离并用蚁酸进行消毒,且一个个体使用蚁酸消毒会引起周围数个个体进行同样的行为。\n\n一些蚂蚁会利用树脂来杀菌,或是从培养的一些真菌·放线菌里获取抗生素来杀菌。Formica paralugubris收集的针叶树树脂的杀菌功能已经被实验验证。\n\n大部分社会性昆虫群体里的患病个体会减少与其它个体的接触。当然,一些疾病会阻止个体自由行动,还有一些疾病在表现出症状之前就可以通过口对口交换食物的行为传播,但能够影响的个体数量终究是很有限的。而且,获得免疫力的蚂蚁会增加口对口交换食物的行为,该行为可以引起参与交换的个体获得免疫力。\n\n社会性昆虫群体之间的低互动性使传染病难以长距离蔓延,与原始时代的人类部落相似。\n\n少数社会性昆虫会吃掉容易感染疾病的幼虫,等大流行过去再生一批就是了。你可以预期某些国家的人不需要学就知道这一招。在一般国家,这一招的类似措施是让学校停课、孩子各自回家去——思路都是“不要将大量的幼体堆在一起给病原体来传染”。", "task_type": {"major": ["问答"], "minor": ["知乎问答"]}, "domain": ["通用"], "metadata": "qid:461441504, aid:1906544450", "answer_from": "human", "human_verified": true, "copyright": "暂无版权及作者信息"} {"instruction": "恐龙几十吨重,孵蛋会把蛋压碎吗?", "input": "", "output": "恐龙比你想象中要聪明。\n\n根据已有化石证据,并不支持几十吨的恐龙拥有孵蛋行为,甚至大部分恐龙都不孵蛋,它们倾向于用泥沙或植物把卵埋起来,靠外部热源孵化;一些小型兽脚类恐龙(如偷蛋龙、伤齿龙)产卵后有着与现今鸟类相似的孵蛋行为,用羽毛覆盖或直接坐在蛋上,可能是孵蛋1.0版本,为现今鸟类多样化的繁殖行为打下基础。\n\n## 怎么筑巢?\n\n从1993年起,古生物学家在蒙古戈壁陆续找到正在孵蛋的恐龙化石标本(葬火龙Citipati,属于偷蛋龙科),到20世纪末至21世纪初,关于鸟类演化自恐龙的一系列证据被研究发表,大众在逐渐接受鸟类就是恐龙这一事实的同时,也经常对比两者的生物行为——比如题主所说的孵蛋。\n\n讨论孵蛋行为,往往离不开筑巢行为,因为有孵蛋习惯的恐龙,基本不会选择用泥沙埋卵。\n\n从开篇的总结可知,(非鸟类)恐龙大概有两种筑巢假说:\n\n- 一是偏向龟鳄类的“埋藏式巢窝”,恐龙产卵后,会用泥沙或植物将其掩埋,借助太阳光、植物腐烂等外部热源来提供温度,从而完成孵化。\n- 二是偏向现今鸟类的“露天式巢窝”,恐龙蛋大部分面积会暴露在地表,部分掩埋在沉积物里(可能起到固定、隐蔽性等作用),父母依靠带有羽毛的前肢或腹部来提供温度。\n\n要知道(非鸟类)恐龙历史跨越三叠纪-侏罗纪-白垩纪,长达1.6亿多年,种类之多,所以想确定谁喜欢孵蛋以及怎么孵是十分困难的一件事。\n\n首先想要找到合适的研究材料并不容易,巢穴化石、胚胎化石本就稀有,“连龙带窝一锅端”的化石更是难求;其次尽管恐龙蛋化石记录很丰富(搜一搜河源恐龙蛋新闻就知道了),但也仅是数量维度上,且基本是没法和恐龙种类一一对应上的,恐龙蛋有独立分类体系;最后早期的恐龙蛋是软壳蛋,不稳定,很难形成化石,这也是众多恐龙“无蛋”的原因……\n\n## 谁会孵蛋?\n\n受限于以上客观因素,古生物学家通过恐龙蛋化石来研究筑巢行为的论文里,有不同的声音就再正常不过了,比如有提出一些泰坦龙类(题主所说的几十吨恐龙)孵蛋的观点。\n\n整体而言,主流观点还是倾向于大多数恐龙(如几十吨的蜥脚类、早期兽脚类、部分鸟臀类)更适合用“埋藏式巢窝”,因为它们的蛋壳都具有高孔隙度——蛋壳上有较大、较多的孔,这种结构有利于进行气体交换,恐龙蛋被沉积物掩埋后,胚胎仍能顺利发育;小部分兽脚类恐龙(如偷蛋龙、伤齿龙类)的蛋壳有较低孔隙度,气体扩散能力相对差,因此这更可能是一个适应在露天环境产卵的演化特征。\n\n除了蛋壳孔隙度,恐龙蛋颜色也为研究孵蛋行为提供一定帮助。\n\n现今存活的羊膜动物中,只有鸟类能产下各种颜色的蛋。在过去很长一段时间里,人类一直认为彩色蛋壳这一特征仅存在于鸟类,但2017年和2018年两篇论文提出了鸟类蛋壳颜色起源于其恐龙祖先,也就是身披羽翼的那一群小型兽脚类恐龙。\n\n同时,研究结果更是显示蛋壳色素只存在于兽脚类,比如河源龙(偷蛋龙科)和恐爪龙(驰龙科)就会产下蓝绿色的恐龙蛋,蛋壳色素沉积方式与鸟类也相似。而兽脚类之外的种群,比如慈母龙和泰坦龙类的蛋壳没有发现色素痕迹,习惯于使用“埋藏式巢窝”的鳄鱼也不产彩色蛋。\n\n如果从蛋壳色素分布情况来推断生物行为,那么拥有彩色蛋的兽脚类恐龙将采用“露天式巢窝”,趴在窝里,像现今鸟类一样去孵蛋,彩色的蛋壳能够更好地伪装以躲避真正的“偷蛋龙”,甚至是抵御巢寄生(假设它们那个时代也有杜鹃这种家伙);而拥有无色蛋的鸟臀类、蜥脚类恐龙则倾向于选择把卵埋起来,采用龟鳄类一样的孵化策略。\n\n## 怎么孵蛋?\n\n基于蛋化石的外壳结构和色素,结合恐龙胚胎化石、成年个体化石,可以为了解这群恐龙的孵蛋行为进一步提供更有力的证据支持。\n\n回到开篇提到的经典葬火龙孵蛋化石标本,现实里它保持着鸟类一般的孵蛋姿势:蛋一般排列成三层的同心圆,数量可多达约22颗,它伏在巢中央的大空隙,张开前肢,用羽毛环绕着一颗颗蛋。可这个标本实际上并不够完美,因为蛋壳内部的胚胎化石没有保存下来,你能够质疑它,并对此开展多种解读:这只恐龙在产卵时恰好遭遇沙尘暴寄了、还没来得及掩埋卵就暴毙了、面临危险之际呈现的一种护蛋行为……总之不是在孵蛋就对了。\n\n更完美的证据是来自中国江西赣州的偷蛋龙化石,不仅能看到上述同样孵蛋姿势,更能看到处于不同发育阶段的胚胎,蛋壳孵化温度也不同,这意味着偷蛋龙幼崽会在不同时间破壳,跟现生鸟类的“异步孵化”繁殖特点如出一辙。\n\n这家伙是真的在孵蛋,而且还挺有一套。\n\n当然,也有恐龙会直接坐在蛋上孵化,并且产卵方式也比较奇特。\n\n比如伤齿龙,它产卵时,并不会像偷蛋龙一样把蛋铺一层又一层,而是把蛋立起来,直接插到土里,聚集在中央。孵化时,用腹部直接接触蛋壳,真正用体温来孵蛋。至于压不碎的力学原因,没查过,感兴趣可以自行去搜索相关论文。\n\n看起来,目前恐龙筑巢和孵蛋行为的进化路线就大概是:\n\n- 埋蛋,靠外部热源孵化(源于最早的恐龙及其祖先)\n- 露天筑巢,开始亲自上阵孵蛋(源于兽脚类,并在接近鸟类的恐龙中演化出各种1.0版本)\n- 进一步完善迭代(已灭绝的鸟类)\n- 筑巢选址、孵蛋技巧百花齐放(现代鸟类)\n\n总之,恐龙比想象中聪明,筑巢行为也比想象复杂得多,以上仅为管中窥豹。", "task_type": {"major": ["问答"], "minor": ["知乎问答"]}, "domain": ["通用"], "metadata": "qid:591115526, aid:3027634411", "answer_from": "human", "human_verified": true, "copyright": "暂无版权及作者信息"} {"instruction": "有没有人类保护环境但适得其反的例子?", "input": "", "output": "有很多,例如:\n\n澳大利亚和美国近年来的猛烈野火,部分归因于“一些植被长得太过连续、不利于消防,但环保政策和某些环保人士妨碍了用于控制该状况的小规模砍伐与可控燃烧”。过度受限的小规模砍伐与可控燃烧不但起不到控制野火蔓延的作用,在美国部分地区还导致地表火焰扩散加快,增加人员伤亡的风险。\n\n发展可再生能源对降低人为温室气体排放量很重要,但是人们现有的太阳能电站与风电机大量杀死野生鸟类。许多鸟是自己撞死在太阳能面板或巨型扇叶上的,可能是将反光的面板当成了水面、将白色的扇叶当成更柔软的东西来互动的结果。实验显示,将风电机涂成黑色可以降低鸟的自杀率。你可以看看世界各地有哪个大型风电场将风电机都涂黑了。废旧太阳能电池的无害化处理也还是个问题,废旧风电机的扇叶目前已经有一些“加工成其它产品或打碎并在发电厂里完全燃烧”的方案。\n\n美国西海岸一些地方紫海胆泛滥,造成海藻林退化、大量动物死亡,但当地环保政策禁止大量捕捞或杀死紫海胆。一批科学家游说多年,才得到在数平方千米的狭小海域内无限制消灭紫海胆的许可,这是杯水车薪。\n\n澳大利亚于 1893 年设立赏金来鼓励人们猎杀入侵物种赤狐(该物种被人于 1863 年作为休闲狩猎的射击对象引入澳大利亚后野化),2002年,他们评估了这项政策的效果,发现其在很大程度上没用甚至适得其反——人们希望赤狐继续存在以便持续获取赏金,猎杀强度从未达到消灭赤狐所需。\n\n截止 1995 年,澳大利亚在 80 年间为控制入侵植物马缨丹而接连引入了 28种攻击马缨丹的昆虫,但马缨丹仍在散播,而这些昆虫频繁攻击目标以外的植物。其中最好笑的是第 28 种昆虫——澳大利亚人测试了角蝉科的 Aconophoracompressa 对 62 种常见植物的攻击倾向,认为它很安全,将它释放到环境后才发现它很喜欢并未测试过的观赏植物 Citharexylumspinosum,而这种树在澳大利亚居住区很常见。这动摇了人们对生物防治的信心。\n\n新西兰于 1880年代引入白鼬来捕杀泛滥的兔子,但白鼬很快大规模击杀当地鸟类,新西兰人不得不着手控制白鼬。他们使用了投毒、设置陷阱等多种方式,预计新西兰的白鼬会在本世纪中叶灭绝。\n\n意大利近年来闹野猪,部分归因于“环保政策导致适合野猪增加数量的植被持续增加,动物保护政策和环保人士的抗议让控制野猪数量的行动寸步难行”。\n\n我们并不擅长保护环境。", "task_type": {"major": ["问答"], "minor": ["知乎问答"]}, "domain": ["通用"], "metadata": "qid:495597377, aid:2204321621", "answer_from": "human", "human_verified": true, "copyright": "暂无版权及作者信息"} {"instruction": "如果人类是由鱼进化而来的,为什么我们反而不能在水里呼吸呢?人类进化经历了一段漫长的岁月,从这些进化当中可以得知,人类是在往一个有优势,且更加智能的方向发展,但是为什么却摒弃了一些对于现在的人来说同样是对我们有利的优势呢?", "input": "", "output": "没那么简单。\n\n人的肺不能处理大量的水,但也不能没有水。包括人在内,脊椎动物的肺泡从表层的水基液体(肺泡液)里获取溶解氧,而不是直接暴露在空气中。\n\n- 成年人的肺泡液约 36 毫升,水分随呼吸流失,补充肺泡液所需的水通常是人每天生存所需水量的约四分之一。\n- 在干燥空气中,没有肺泡液保护的肺上皮细胞会在数分钟内因水分流失而受损、死亡。\n\n人的皮肤可以通过扩散作用从水中取得少量溶解氧。这不足以维持整个人体的生命。\n\n- 顺便说,1851 年就知道人的皮肤可以通过扩散作用从空气取得少量氧气;2001 年,实验显示,在皮肤完整、潮湿的条件下,人体表层到 0.25~0.4 毫米深处的细胞可依靠从空气获取的氧生存 。\n\n作为人类祖先的鱼是数亿年前的一些肉鳍鱼,它们起初用鳃呼吸,后来演化至用肺呼吸。\n\n从鳃到肺的转变并不罕见:\n\n- 常见的青蛙、蟾蜍从蝌蚪变态发育而来(一些罕见的小体型青蛙不经过蝌蚪阶段)。蝌蚪的鳃能在水里呼吸。变态发育后,青蛙、蟾蜍的肺不能处理过多的水。\n- 青蛙、蟾蜍通常可以在水中靠皮肤吸收溶解氧来维持生命 4 到 7 小时,在水下剧烈运动会缩短它们的潜水时限。\n- 不少蝌蚪同时具有鳃和肺。在生命早期,蝌蚪没有足够的力量突破水的表面张力将头伸出水面,依靠鳃和皮肤呼吸溶解氧,但有肺的蝌蚪在体长 3 毫米时就能通过制造气泡呼吸空气。长大一些后,许多蝌蚪用肺获得更多氧气,而鳃和皮肤的呼吸能力不足以长时间维持长大的身体。你可以在这里看到某些祖先的影子。\n\n- 蝌蚪的肺还有助于控制浮力。\n\n用于处理溶解氧的鳃在空气里会很快干燥而塌陷,大幅减少可用于气体交换的表面积。攀鲈、弹涂鱼等辐鳍鱼现在进行时地具有在陆地上活动数小时的能力,攀鲈以鳃上呼吸器官处理空气,弹涂鱼以皮肤、口腔、喉咙处理空气,这些结构的正常功能都需要保持湿润。脊椎动物肺的内陷构造有助于防止结构变得干燥,这降低了排出过多液体的能力。\n\n演化过程中,生物的可遗传性状经常减弱或缺失。无论减弱或缺失了什么,只要可育后代的数量没有明显减少,新性状都有概率作为中性性状在种群中长期存续。对一部分在浅水活动的古代肉鳍鱼来说,“更擅长呼吸空气的性状”在结果上支持更快的代谢、有利于捕食·自卫,可能有利于求偶,并在一定程度上允许“更不擅长处理大量水的性状”不影响可育后代的数量。一些古代肉鳍鱼在登陆前就演化至“成体用肺呼吸空气、在浅水·潮间带等处捕食”,这之中的一些种群日后发展出包括人在内的四足形类。\n\n数亿年来,体型较大的四足形类的耗氧量不能在不大幅修改身体的情况下用鳃维持。从陆地返回海洋的鲸豚类、鳍足类等并未切换回靠鳃呼吸。\n\n- 问题出在水中溶解氧的含量明显低于空气中的氧气。\n- 溶解着大量氧气的全氟化碳可维持老鼠之类实验动物呼吸,尽管呼吸远比空气密度大的全氟化碳要消耗相对多的体力。\n\n题目里“人类是在往一个有优势,且更加智能的方向发展”的说法源自过时的定向进化论。在此基础上,由于我国媒体灾难性的表达能力和近年来伸腿瞪眼的语文教育,许多人将“人类是由鱼进化而来的”这句话当成了“人是从某种在水里游的鱼直接发展而来,跟‘古猿’、‘猩猩’、‘猴子’之类无关”,造成更多误会。\n\n过去数十年间,我国生物学义务教育不准确地描述了“一部分鱼演化成两栖类、一部分两栖类演化成爬行类、一部分爬行类演化成哺乳类”。常理而言,用不着“人类是由鱼进化而来”这个情报,提问者也会注意到一部分脊椎动物的呼吸方式在演化过程中发生了变化。大概提问者以为“哇,原来人不是从古猿进化而来的,人应该跟水里游的那些兄弟更像才对”,暂时忘记了青蛙什么的。", "task_type": {"major": ["问答"], "minor": ["知乎问答"]}, "domain": ["通用"], "metadata": "qid:595083819, aid:3010923976", "answer_from": "human", "human_verified": true, "copyright": "暂无版权及作者信息"} {"instruction": "在太空中人类的昼夜节律是否会被改变?为什么?", "input": "", "output": "目前会。近地轨道上的光照规律、航天器内的照明状况、微重力、电离辐射、工作时间安排会影响人类航天员的昼夜节律。睡眠不足、睡眠剥夺、疲劳是航天员最常见的主观抱怨。\n\n在天空实验室、和平号空间站、美国航天飞机、国际空间站上记录到航天员每 24 小时的平均睡眠时间约 6 到 6.5小时,他们有时会发生入睡困难。美国航天员在国际空间站的平均安眠药消耗量是地球上普通人群的 10 到 20 倍。\n\n睡眠不足会影响航天员的情绪、认知能力、工作能力。\n\n- 航天员经常两班倒工作。\n- 航天器上的部件经常发生故障、让航天员面临计划外的高强度操作需求。\n- 航天飞机、联盟号等航天器飞往国际空间站的发射窗口相当窄,参与对接任务的航天员需要在指定发射时间前至少 5 小时醒来,这会影响他们的工作时间安排。\n- 细胞培养实验显示,人细胞在微重力下分泌的褪黑素较少,而且一部分褪黑素会被用于应对辐射损伤。这会影响昼夜节律和骨代谢,与微重力的其他效应一起加速骨质流失。\n\n人工调光可以帮助航天员适应。可以在航天器上用人工光照与人工遮光营造接近 24小时的昼夜循环,在保证散热支持的条件下让航天员适当运动锻炼来减慢骨质流失。现有航天器尚未尝试以足够快的自旋来提供拟似重力。", "task_type": {"major": ["问答"], "minor": ["知乎问答"]}, "domain": ["通用"], "metadata": "qid:593684695, aid:2969227587", "answer_from": "human", "human_verified": true, "copyright": "暂无版权及作者信息"} {"instruction": "像桃花源这样的封闭聚落最终都会因为近亲繁殖的问题而自然消亡吗?\n条件\n完全封闭与外界无接触\n自然资源充足\n人数的话假设一开始有1000村民好了,男女平均", "input": "", "output": "不会,题设的初始村民人数 1000 远远超过处理“近亲繁殖”问题所需。\n\n- 现实中就有不到 1000 人的岛屿部落、偏远村落长期存续。而且,古人类的小型迁徙群体约 160 人。当前人类的基因多样性本就很低,看起来这个水平已经够用了。\n- 胡特尔派在十八到十九世纪几乎被来自外部的暴力攻击灭绝,后来以高度近亲繁殖的方式在 125 年间从约 400 人增加到约 42000 人,现在有数百个居民点。他们的每个居民点包含 10 到 20 个家庭,居民点人口约 60 至 250,在居民点的人数超过 250 时,领导人会找个口实、以抽签方式选出半数成员去开拓新的居民点。你可以参考他们的历史,想想 1000 是不是太保守了。\n\n致力于保存濒危物种的科学家考察了多种动物维持生存所需的最小数量,1980 年代,澳大利亚的 Ian Franklin 与美国的 Michael Soulé针对哺乳类和鸟提出 50/500 原则:有 50 只繁殖个体,可以避免近交衰退;有 500只繁殖个体,可以避免漂变造成的不适应。由于要保护的动物的具体情况参差不齐,他们给出的“比起正常的人口规模,更接近 0”的数字经常被同行吐槽,后来干脆乘以10 来搞定:500 只繁殖个体可以忽略近交衰退,5000 只繁殖个体可以自行适应环境。\n\n此外,近亲繁殖本身没有一些人估计得那么危险。普通人的基因组通常有 1 到 2 个纯合时可导致致死疾病的隐性基因。从 1933 年到 1970年在捷克斯洛伐克出生的儿童显示,父母是彼此的一级亲属的儿童有近 40% 患有严重残疾,14%早死。在“躲避战乱、逃到桃花源”之类情景里,逃难前的人群死亡率可能不止如此。对哺乳类家畜育种的经验是,近亲繁殖会快速淘汰若干严重的致病基因,在数代后明显降低遗传负荷,在维持20 代(对古人来说约 320 到 400 年)以上后基本稳定、不再有近交衰退的危险。\n\n自然灾害、传染病、偶然发生的出生率波动等因素对封闭聚落的影响相对大一些。", "task_type": {"major": ["问答"], "minor": ["知乎问答"]}, "domain": ["通用"], "metadata": "qid:594009656, aid:2971457729", "answer_from": "human", "human_verified": true, "copyright": "暂无版权及作者信息"} {"instruction": "分子遗传学推翻了哪些传统观点?", "input": "", "output": "传统认为细菌的基因组结构比别的细胞生物都简单是因为原始,但是通过对核糖体RNA等一系列超保守细胞器组分遗传分子钟的构建,遗传关系显示一部分细菌的基因组结构比古细菌和真核生物简单可能是特化的结果。\n\n常见细菌所属于的一些细菌域后期分支,其基因翻译率高、遗传冗余低、基因调节结构简单的特点,并不是原始特征,反而可能是特化的结果。\n\n细菌域一些早期分化的分支的遗传结构的特征更接近古细菌和真核生物的领域,有比较多的无法翻译的冗余遗传物质和基因表达层面的调节结构。\n\n真核生物域的基因翻译率低、遗传冗余高、基因调节结构复杂,反而是保留了更多LUCA的原始特征。\n\n仔细想想这个结果倒是完全符合RNA世界假说的预测,从RNA世界过渡到DNA+蛋白质世界,越原始的特征应该保留越多的核酶、核酸相互作用和冗余基因,基因可翻译为蛋白质的比例更低。\n\n实际上这些细菌在进化史上特化的过程中丢失了很多遗传结构和细胞结构。这种基因翻译率高、遗传冗余低、基因调节结构简单的特点,也导致这些细菌在面对病毒和环境变化时只能大量死亡,靠数量和积累变异应对自然选择压力,在形成针对病毒的特异性免疫与其他细胞互动的方面都采取了与古细菌真核细胞及更早期细菌分支不同的策略,遗传修复机制较少,而是筛选出一些专门作用于特定病毒基因组的具有更高特异性的剪切酶和直接整合噬菌体的结构,也为发展CRISPR/Cas9基因编辑技术提供了天然的原材料。\n\n有趣的是,细胞特化的过程往往也都伴随遗传物质和细胞结构的丢失,在多细胞真核生物中,通过测序生物体内不同来源的细胞,体细胞分化过程经常伴随着随机的遗传物质丢失,只有体内生殖细胞和多能干细胞能保持遗传完整。并且研究已经发现这些随机的遗传物质丢失,竟然和细胞相互识别、免疫识别、组织结构和神经突触的维持都有关系。但同时细胞这种遗传物质随机丢失,也导致已分化的细胞一但被调动,发生去分化并脱离组织环境重新被赋予多能性,这个过程很容易导致癌变,病毒感染、慢性炎症、纤维化等过程由于频繁调用已分化体细胞重新转变为非干细胞的非附着化间充质多能细胞,成为多细胞生物肿瘤发生和肿瘤转移的重要机制。", "task_type": {"major": ["问答"], "minor": ["知乎问答"]}, "domain": ["通用"], "metadata": "qid:593595482, aid:2969060391", "answer_from": "human", "human_verified": true, "copyright": "暂无版权及作者信息"} {"instruction": "你知道蚊子有多少种吗?", "input": "", "output": "知道。\n\n截止 2023 年 3 月 9 日,“蚊子”是蚊科 3618 个已知有效物种的通称。\n\n在此基础上,摇蚊科的超过 5000 个物种也经常被人误认为是蚊子。\n\n一些科学家会在网络上更新蚊科的分类情况,例如:\n\n[Mosquito Taxonomic Inventory](https://mosquito-taxonomic-inventory.myspecies.info/valid-species-list)\n\n算上未知物种,地球上当前可能有超过 1 万个物种会被目击者称为“蚊子”。\n\n截止 2023 年 3 月初,不同的科学家将蚊科分成 43 个到 113 个属。\n\n截止 2023 年初,已经观测到约 90 种蚊子能传播疾病,它们主要来自伊蚊属、按蚊属、库蚊属。\n\n截止 2023 年初,已经记录了至少 28 种蚊子化石,每个都可能代表一个历史上存在过的属级分类。", "task_type": {"major": ["问答"], "minor": ["知乎问答"]}, "domain": ["通用"], "metadata": "qid:591320006, aid:2949995186", "answer_from": "human", "human_verified": true, "copyright": "暂无版权及作者信息"} {"instruction": "生孩子那么痛苦,为什么人类在过去几百万年中都没有因为不愿生孩子而灭绝呢?", "input": "", "output": "首先,题目的前提“生孩子那么痛苦”的可信度是不稳定的,有时是可疑的。\n\n- 对双胞胎的研究显示,人类女性的骨盆宽度受生活环境的影响似乎超过基因造成的影响。农业时代的饮食变化可能比骨盆宽度的遗传因素造成了更多的产科问题。\n- 在过去几百万年的大部分时间里,人的生育没有现在这么痛苦;现代人生育时的痛苦程度也因人而异、在一生中会多次改变,通常二胎或更多胎的痛苦程度低于头胎。\n\n在此基础上,现代的一些宣传是极度歪曲的,夸大了分娩的痛苦和死亡风险。你可以在这个问题下的某些回答里看到千奇百怪的夸张模因。\n\n- 美国麻醉医师协会的全国性调查显示,46% 的新母亲表示,生第一个孩子时经历的痛苦低于她们的预期。询问“分娩过程中最痛的时候有多痛”时,45% 的回答是“像极重的痛经”,16% 的回答是“像严重的背痛”,15% 的回答是“像骨折”。无论使用麻醉还是辅助性的疼痛缓解手段,90% 的参与者表示疼痛缓解有效。\n\n其次,在过去大部分时间里,人们没有足够的认知能力去理解孩子是怎么搞出来的,没有足够的认知能力与技术去进行有效避孕或堕胎。你完全无法指望分娩时的痛苦去阻止历史上的大部分人进行交配、怀孕、分娩。\n\nSherwood Lerned Washburn 于 1960年提出“产科困境”假说,试图将人类女性那看起来异常困难的分娩说成“演化产生的解决方案”。古尔德过去附和过“产科困境”,将其推得很广。这是个看起来很好懂的模因,被各路媒体和他们的受众到处传播。但这不是事实。提出“产科困境”假说时使用的统计数据可能不反映历史上大部分时候的状况。\n\n- 类人猿的足月妊娠所需时间是灵长类里最长的,黑猩猩、倭黑猩猩的正常妊娠期约 39 周,人的正常妊娠期约 37 到 41 周。黑猩猩、倭黑猩猩的新生儿像人新生儿一样缺乏独立生存能力,需要母亲提供大量照顾。但这两种类人猿并没有人这么高的难产率,它们的产道比新生儿尺寸明显更宽。\n- 正常人类新生儿的大脑体积是成年时的 30% 左右,正常黑猩猩新生儿的大脑体积是成年时的 40% 左右,卷尾猴新生儿的大脑体积是成年时的 50% 左右。黑猩猩新生儿那 40% 并没有改变其缺乏独立生存能力的表现。所谓“人类新生儿都是早产儿”的说法是错误的。\n- 人大脑生长速度的峰值出现在出生前的几个月间,与生下来就能行走的动物类似(Halley, 2017 年)。\n- 将人类女性的骨盆再加宽几厘米以扩大产道,不会和“产科困境”假说设想的那样妨碍双足行走。非洲南部石器时代晚期的人群似乎搞出过身高很低而骨盆很宽的女性,当时他们经常长距离迁徙,化石并未显示这些人在行走方面有什么困难。生物力学也不支持“加宽骨盆会增加运动成本”的想法。\n- 农业时代以来,生活方式的改变让人分娩的难度上升。约 12000 年的农业社会生活尚未让人的骨盆宽度重新优化到普遍适合新生儿的大头与宽肩膀(人类新生儿的颅骨可动性减少了大头的影响,肩膀有时比头更不好处理),看起来这造成的影响在种群层面并不重要。当前状态可以算是略有不利的中性性状,能在种群中长期传递。\n- 人类胎儿的能量需求和母体能安全地提供的代谢率与散热能力的关系似乎更直接地决定了胎儿能达到的体重。这方面的统计相关性也出现在没有骨头限制产道的鲸类身上(Sacher 和 Staffeldt,1974 年)。", "task_type": {"major": ["问答"], "minor": ["知乎问答"]}, "domain": ["通用"], "metadata": "qid:579025680, aid:2848919116", "answer_from": "human", "human_verified": true, "copyright": "暂无版权及作者信息"} {"instruction": "都说全球变暖导致一些珊瑚礁的死亡消失,那其他地方有产生新的珊瑚礁吗?有的话在哪里?", "input": "", "output": "有。珊瑚正在死去的珊瑚留下的位置及其周围生长,一些热带珊瑚的分布区域朝两极移动,另一些相对耐热的珊瑚留在原地。地质史上全球大规模暖化时留下的珊瑚化石也有类似的迹象。\n\n- 二十世纪三十年代以来,科学家在北半球一些地区研究的 9 种珊瑚中有 4 种以每年 14 千米的速度向北扩张。从 1930 年到 2011 年,日本温带领海的冬季表层水温上升了约 2.4 摄氏度,允许更多珊瑚生存在比历史上更靠北的海域并造礁。\n- 从 1974 年到 2012 年,赤道海域的珊瑚密度有所下降,亚热带海域的珊瑚密度有所增加。在南北纬 20 度以上海域,珊瑚密度增加了 78% 以上。大量珊瑚出现在北纬 33 度、南纬 30 度附近,这里将会形成新的珊瑚礁。\n\n2022 年,大堡礁受损区域内出现大范围的珊瑚复辟,大堡礁北部的硬珊瑚覆盖率达到约 36%,是自人类开始监测其覆盖率以来的最高水平,大幅超过 2017 年的13%;大堡礁中部的硬珊瑚​​覆盖率估计为 33%,同样是有记录以来的最高水平,大幅超过 2019 年的 14%;大堡礁南部的珊瑚覆盖率估计为 34%,比2021 年下降了 4%,主要原因是棘冠海星爆发。\n\n珊瑚的共生微生物群可以在温度变化下发生变化并适应。\n\n对珊瑚来说,形成“全新的珊瑚礁”通常要花 25 年,有些物种造礁速度更慢或不造礁。人为投放活珊瑚碎片可以加快一些地方的珊瑚生长。", "task_type": {"major": ["问答"], "minor": ["知乎问答"]}, "domain": ["通用"], "metadata": "qid:587349256, aid:2921063063", "answer_from": "human", "human_verified": true, "copyright": "暂无版权及作者信息"} {"instruction": "生命科学的第一性原理是什么?\n第一性原理的首创者是古希腊著名哲学家亚里士多德。他认为:“任何一个系统都有自己的第一性原理,是一个根基性命题或假设。它不能被缺省,也不能被违反。”", "input": "", "output": "一部分学者认为生命科学的第一性原理是 Erwin Bauer 于 1920 年发表的《生物科学的基本原理》中提出的稳定非平衡原理:\n\n- 生命系统,且只有生命系统,永远不会处于平衡状态。借助自由能,它们持续地做功来避免根据物理和化学定律应当在给定的外部条件下达到的平衡。\n\nMikhail Volkenstein 于 2009 年给出了更简短的版本:\n\n- 生命系统永远不会处于平衡状态,这以它们的自由能为代价。\n\n在上述命题中,所谓“永远不会处于平衡状态”是靠“一旦某个生命系统不可逆转地达到了平衡状态,该系统就不再算是生命系统”来实现。这不仅在逻辑上相当难看,而且不可避免地遇到了台风之类耗散系统是不是生命系统的问题。\n\n- 已知的生命系统似乎都是耗散系统,生命系统和其他耗散系统的边界要么不存在要么十分模糊。\n\n薛定谔于 1943 年 2 月在都柏林三一学院的公开讲座谈论生命时用“负熵”表示“热力学自由能”,思路跟稳定非平衡原理相似。讲座内容在 1944年出版为《生命是什么》——并不出色的作品,但你可以参考里面的一部分思路。\n\n此外,你可以考虑道金斯的“生命是自然选择塑造的信息”。虽然形式上看起来跟上面的命题有很大差异,但是这个也允许谈论黏土结晶、计算机程序之类的生命特征。", "task_type": {"major": ["问答"], "minor": ["知乎问答"]}, "domain": ["通用"], "metadata": "qid:587873807, aid:2925764210", "answer_from": "human", "human_verified": true, "copyright": "暂无版权及作者信息"} {"instruction": "大理石纹螯虾是如何进行“自我复制”来繁殖的?", "input": "", "output": "大理石纹鳌虾,有学者建议命名为“处女鳌虾”,是自然产生的三倍体雌性龙纹鳌虾,进行单性生殖(孤雌生殖):\n\n- 2007 到 2008 年的两项研究认为,大理石纹鳌虾的卵母细胞不进行减数分裂;\n- 2016 年,日本的一项研究显示,大理石纹鳌虾的卵母细胞不进行第一次减数分裂,但会进行类似第二次减数分裂的分裂并自行恢复染色体组数;\n- 无论如何,大理石纹鳌虾的孤雌生殖不包含“卵细胞互相融合或卵细胞与极体融合”之类过程,是无融合生殖的孤雌生殖。它们产生的三倍体卵子无需受精,可以被批量排出体外、在适宜的水温下开始发育。\n- 这种方法产生的不同个体可以携带各自不同的基因突变,并在和环境的互动过程中配备不同的表观遗传修饰。\n- 实验显示,雄性龙纹鳌虾的精子不能让大理石纹鳌虾的三倍体卵子受精。\n\n目前推测大理石纹鳌虾于二十世纪九十年代(最晚 1995年)起源于水族馆中,减数分裂发生的随机错误或是水温变动的影响产生了一颗二倍体卵子,该卵子受精并发育成三倍体个体,单独建立了种群。这是在一代间建立了生殖隔离、形成了新物种。\n\n \n\n人们对孤雌生殖物种有很多误解,例如大谈“容易感染病原体而灭绝”之类,需要纠正。\n\n其中,有人用绿水螅的图片表示“菌”,在他人指出后声称“为了美观牺牲客观”,很需要纠正。\n\n2021年,实验发现采集自不同地区的大理石纹鳌虾的单核苷酸多态性较低;与此同时,大理石纹鳌虾明显在对新入侵地区存在的高盐度的水做出适应,扩张至祖先未到之地,并在体色、花纹、体型大小上表现出地理差异。大理石纹鳌虾对欧洲气温的适应可能主要来自表观遗传,这是因为实验显示其基因组甲基化程度比龙纹鳌虾显著降低。\n\n大理石纹鳌虾携带鳌虾瘟疫真菌 Aphanomycesastaci,可以传染给世界各地对此敏感的鳌虾(主要是欧洲的鳌虾物种),自身保有北美鳌虾对该病的抗性。讽刺的是,十九世纪以来在该真菌影响下濒临灭绝的欧洲鳌虾均为有性生殖物种。\n\n基因组中的大部分遗传多样性是中性的,既不会提高也不会削弱个体生存或产生后代的能力。生物的环境耐性(包括疾病抗性)相关的基因,影响个体生存或产生后代的能力的遗传多样性,存在于基因组内非常特定的区域,与全基因组的遗传多样性完全无关。快速传代、大量繁殖的无性生殖物种或孤雌生殖物种完全可以拥有压倒有性生殖物种的环境耐性。\n\n不限于无性生殖或孤雌生殖,许多入侵物种抵达新地区时只有数个个体,基因多样性暴跌,但迅速发展为庞大的新种群,并在特定的基因上显示出迅速适应。2021年,科学家在入侵物种“绿蟹”身上再次确认了这件事。\n\n大理石纹鳌虾有 276 条染色体,龙纹鳌虾为 184 条。但是,大理石纹鳌虾似乎存在一些基因丢失,其总碱基对数约为龙纹鳌虾的 140% 而不是150%,这也有助于二者产生生殖隔离。在这样的丢失过程中大量基因会重新排列,与拷贝数的变化一起改变生物的蛋白质合成规律和性状。大理石纹鳌虾的最大体长、最大寿命、单次产卵数都大幅超过龙纹鳌虾,性成熟所需的时间相近。\n\nBdelloid rotifer 和 Darwinulid ostracods,通称“进化丑闻”,在没有经历有性重组的情况下持续了约 8000 万年到 1亿年,分别产生了 360 个和 28 个物种(Butlin,2002 年;Mark Welch 等,2009 年;Schön 等,2009年)。几个现存的孤雌生殖竹节虫谱系在没有经历有性重组的情况下持续了超过一百万代(Schwander 等,2011年)。孤雌生殖的长期持续存在可以解释为有害突变被表观遗传沉默、表观遗传多样性可以执行序列多样性的功能(例如抑制寄生虫、适应环境),随着时间流逝,表观遗传多样性可能被碱基变化固定为序列多样性,最终产生新物种(Vogt,2015年)。", "task_type": {"major": ["问答"], "minor": ["知乎问答"]}, "domain": ["通用"], "metadata": "qid:266920159, aid:2128466682", "answer_from": "human", "human_verified": true, "copyright": "暂无版权及作者信息"} {"instruction": "听说食肉动物的皮肤具有游离性,有一定的防御能力,想知道防御能力如何,贴吧虎吧有人那这个吹老虎的体防?", "input": "", "output": "食肉目皮肤的防御能力一般。狮子、老虎身上容易与其他生物接触的部位的皮肤厚度可达 2~3厘米,能防御一些攻击,但远远没有到值得吹的地步。狮子皮已经被古人证实不适合作为衣服,挂在墙上或放在座椅上当装饰还可以。\n\n老虎皮当衣服的性能同样堪忧,主要用于地毯、壁挂、坐垫。\n\n跟大猫的皮比起来,熊、河狸、海獭、狼獾、渔貂的皮要耐用得多。\n\n在防御穿刺方面,已经在野外观测到狮子·老虎被豪猪刺死、被毒蛇咬死。并非所有毒蛇都有足够长的毒牙来攻击这些动物。\n\n在防御打击方面,“皮肤具有游离性”并不能抵御长颈鹿、大象之类大型植食性动物的踢打。\n\n老虎像其他猫科动物一样有护毛和底毛。护毛比底毛长而粗,有一定的防水性能,可以遮挡紫外线并防御擦伤、轻度切割伤和某些昆虫。底毛就是绒毛,在皮肤周围约束空气来保暖。狮子、老虎的毛比很多人想象的“硬如铁丝或铜丝”要柔软,但没有家猫那么软。\n\n若要抵抗步枪之类武器,体型比虎皮更有用。小口径子弹的击倒能力不足以立即制动成年老虎。你可以使用较大的子弹。", "task_type": {"major": ["问答"], "minor": ["知乎问答"]}, "domain": ["通用"], "metadata": "qid:524272343, aid:2412936353", "answer_from": "human", "human_verified": true, "copyright": "暂无版权及作者信息"} {"instruction": "用其他物种的花粉刺激三倍体西瓜会使其结果吗?", "input": "", "output": "一部分非西瓜物种的花粉可以让三倍体西瓜结果,例如葫芦品种“Kachidoki2”的花粉。这种葫芦花粉亦可诱导二倍体西瓜产生符合商业无籽标准的果实,从而省略搞三倍体的过程。\n\n- 实验显示,在上述无籽果实启动发育时,葫芦花粉管根本就没有到达西瓜子房的胚珠。\n- 这样搞的结果率约 57.1%,比用二倍体西瓜花粉为二倍体西瓜雌花授粉的结果率 65.0% 略低。\n- 用葫芦花粉搞出来的西瓜果实,横截面有变成三角形或长方形的趋势,果长与果宽之比有明显变化,但果实重量、果皮厚度、果肉颜色、糖度与对照组几乎相同。在这个场合,形状变化可能是优势。\n\n顺便说,二倍体西瓜、四倍体西瓜与三倍体西瓜的生殖隔离是不完全的。三倍体西瓜仍有概率产生正常配子,市面上的一些无籽西瓜含有少量正常种子。\n\n- 美国农业部规定,等分的切面露出 10 个或更多成熟种子的“无籽西瓜”不能算做无籽西瓜,一批“无籽西瓜”里超过 10% 的瓜不算数会导致这批“无籽西瓜”评不上美国等级 1。\n\n历史上,生产无籽西瓜的方法主要有用植物生长调节剂刺激二倍体西瓜雌花、用二倍体西瓜花粉为三倍体西瓜雌花授粉、用软 X射线辐照过的二倍体西瓜花粉(配子无活性)为二倍体西瓜雌花授粉、用四倍体西瓜的低活性花粉为二倍体西瓜雌花授粉。使用葫芦花粉的技术跟历史上的其他方法明显不同,产生的果实完全没有正常种子。", "task_type": {"major": ["问答"], "minor": ["知乎问答"]}, "domain": ["通用"], "metadata": "qid:582942644, aid:2884830051", "answer_from": "human", "human_verified": true, "copyright": "暂无版权及作者信息"} {"instruction": "动物能通过观看影视作品获得乐趣吗?动物喜爱观看的影视作品可能是什么样的?会提出这个问题是因为的前些天看到一个黑猩猩看视频的视频。黑猩猩眼中的视频是什么样的?它能理解它观看的视频吗?能通过观看视频获得乐趣吗?人类为什么能通过观看视频获得乐趣?其他动物如海豚,章鱼,猫狗等呢?", "input": "", "output": "在播放的内容波长、亮度、刷新率适当的时候,视力不太差的动物可以观看影视作品;在播放的声音频率、响度适当的时候,听力不太差的动物可以听到音频。“能不能理解”“有无乐趣可言”在子非鱼的层面上无法判断,不深究这个的话,看起来动物可以根据互动视频做出人们预期的部分反应,至少能对视频达到和对周围的事物相似的理解程度——原本就看不懂的东西,例如偏微分方程,写在纸上和显示在屏幕上都不懂。\n\n看起来,黑猩猩、猫狗之类可以从看视频和/或听音频中获得一定的乐趣。人们并没有给海豚和章鱼看过什么正经玩意,但章鱼跟水族馆里的摆设玩得看起来不错,这包括而不限于砸水箱、喷水来短路水箱上方的灯。\n\n2013年的一项研究表明,狗可以根据视觉线索选出屏幕上的狗的图像并将它们分组,狗辨认屏幕上的狗的能力高于辨认非狗之物——这可能是长期演化的繁殖相关识别能力。在音频方面,狗对电视发出的狗的声音、友好的人声、玩具的响声做出的反应多于一般杂音。狗的品种可能会影响它对电视的反应:嗅觉敏锐的猎犬对视觉效果不那么感兴趣,而牧羊犬喜欢看到移动的物体。\n\n历史上,低帧率的显像管电视的图像被狗当做简单的运动或光线闪烁。二十一世纪初,在 HDTV 上出现了针对狗的电视频道,称为DogTV,播放面向狗的高帧率视频信号(狗眼适合的帧率约 75赫兹),按照狗能识别的色彩(黄色、蓝色及其组合)着色,包含狗耳能听到的高频声音,节目内容有放松(例如狗在草地上伸懒腰)、刺激(例如狗在南加州冲浪)、生活(狗对响起的门铃做出反应并服从主人的命令)等,每段节目长3~6 分钟以适应狗的观看习惯。\n\n猫也经常在人家里看电视,表现出可以识别电视屏幕上的不同轮廓和动作并对电视发出的声音做出反应。猫眼适合的频率约 70~80赫兹,并对红色识别能力较弱。猫有时会试图攻击电视上的虫子和老鼠。\n\n你可以预期黑猩猩喜欢看黑猩猩生活相关的节目,其余动物大抵也是关注同物种胜过对它们来说“不知所云”的人类肥皂剧。\n- 在斯图加特动物园进行的实验证明,播放倭黑猩猩打架的视频会让观看视频的倭黑猩猩感到紧张,播放倭黑猩猩进食、玩耍、交配的视频则无此影响。\n\n他人如何获得乐趣、有无乐趣可言,在子非鱼的层面上同样无法判断。不深究这个的话,人们通常会对看到的东西进行模式识别、匹配、预测并以镜像神经元等触发共情,在看到符合预期的内容时触发奖励机制,在看到严重不符合预期的内容时触发战斗反应和/或镇痛机制(并随后因并无真正的危险而出现回调),诸如此类。视频·音频在你那极度复杂的大脑皮层神经连接网络里激起的更多反应是目前不能精确解析的。\n\n节肢动物基于逐行扫描的复眼在观看显像管电视时获得的图像,根据帧率的差异,可能跟节目内容出现极大的偏差。", "task_type": {"major": ["问答"], "minor": ["知乎问答"]}, "domain": ["通用"], "metadata": "qid:479746147, aid:2060165705", "answer_from": "human", "human_verified": true, "copyright": "暂无版权及作者信息"} {"instruction": "养虚拟宠物(任天狗、拓麻歌子、机械玩具)能否和养真宠物一样有助于催产素释放?", "input": "", "output": "能,具体效果在很大程度上取决于虚拟宠物的外形、行为、互动能力。有些时候,机器宠物带来的好处可以超越活动物。\n\n人看到可爱的小狗/小猫的静止图片,会释放一些多巴胺和催产素。这由哺乳动物幼体的视觉线索触发,检测内容包括占身体比例较大的头、占面部比例较大的眼睛、圆润而不分节的身体形状、柔软的身体表面(尤其是绒毛)。这种反应有助于哺乳动物照顾幼体,不限于同物种的幼体。成体仍保有上述特征的小狗、小猫等动物和虚拟宠物可以利用上述机制。机器蛇、机器蝎子、机器蜘蛛、机器苍蝇宝宝、机器蛔虫之类东西要触发上面的机制就难得多了。\n\n动态图、立体影像、可触摸的毛绒玩具、能主动接触人的活动物/机器带来的效果比静止图片更强。动物做出的意想不到的滑稽动作会在神经学层面上吸引人:当你不期望看到可爱的东西却看到了,会触发大脑的奖励机制。不过,同样的行为重复太多次就不会触发这机制了,拓麻歌子(万代电子鸡)那样老式的电子宠物经常让小孩失去兴趣。\n\n与活宠物互动减轻人们压力的前提是宠物行为正常、照顾的难度可承担。机器宠物可以让一部分人觉得足够可爱、能做的事超出预期、照顾起来不难,另一些人(尤其是老年人)会觉得别人送自己机器宠物是在敷衍自己、将自己当小孩对待、用玩具哄自己,后者在人群中占比通常较低。\n\n- 2014 年的一项研究显示,约 10% 的老年人拒绝使用机器海豹 PARO,这比自称不喜欢动物的老年人的比例低。一些老年人可能不是真的不喜欢动物,而是觉得自己没有足够的钱、体力和精力来照顾动物。\n\n针对任天狗,一些研究人员做过调查,发现人们确实觉得从它那里获得了陪伴(这需要背后的一系列化学物质释放与神经活动),但人们反馈的陪伴程度低于真的狗或猫。\n\n世界范围内,人口正在老龄化,老年人和痴呆症患者面临社会孤立和孤独的风险。社交机器人,包括机器宠物,可以提供陪伴、改善情绪、减少焦虑、促进社交互动,产生积极影响,同时避免活动物带来的大部分过敏、传染病、咬伤、抓伤、夜间噪音,还不会像活动物那样受伤或死亡。\n\n- 售价 110 到 130 美元、使用的技术相对简单的“Joy for All”机器猫/机器狗在一定程度上能代替活宠物和更贵的机器产品(例如 PARO 售价 6000 美元)。研究人员分析了美国 15 家购物网站上 1327 条关于买给老年人或痴呆症患者的 Joy for All 机器宠物的反馈,发现大多数人报告对机器宠物的外观与交互性的积极看法,认为机器为用户提供了“照顾”它并将其融入日常生活的机会。\n- 一些负面反馈表示老年人拒绝使用机器宠物,或者老年人试图给机器吃饭、喝水而让人困扰,或者老年人在机器关机/维修时担心它“死掉”了、心情不好,或者大半个养老院的老年人都想来摸机器宠物而造成混乱,或者产品不够结实、使用寿命短于 8 个月(往往与被灌水、从床上意外掉落、抱得太紧等有关),或者产品的喵喵叫声太假、转头时有电动机的嗡嗡声之类。\n- 总体而言,消费者希望机器宠物容易清洁、耐用、可定制、可远程控制音量和动作幅度,尤其是不要搞出太多不必要的喵喵叫来(老年人会担心猫有什么需求未得到满足)。\n- 上述机器猫会对人的触摸和运动做出缺乏可预测性的随机反应,被一些用户解释为类似活猫的行为、值得表扬。你可以联想到《The Last Guardian》里经常无视玩家命令的大鹫(设计初衷就是这更像真的动物)。不过,用不了多久,老年人就可能更需要认知高度发达的机器来照顾他们。\n- 机器海豹 PARO 相关的研究就更多了,它在世界多地的养老院获得正面反馈。2017 年在美国养老院进行的研究显示,与听音乐、参加体育活动、接受精神刺激的对照组痴呆症患者相比,每 3 天一次以持续 20 分钟的小组会议的形式与 PARO 互动 3 个月的痴呆症患者表现出更低的压力和焦虑水平、更少的行为问题、更轻的疼痛水平,氧合水平和心脏状态得到改善,可以减少一些精神药物和止痛药。\n- 与活狗相比,PARO 持续吸引痴呆症患者注意力的能力较低,这可能与竖琴海豹本身不像狗那样长期与人类互动(痴呆症患者偶尔会说“你应该住在大海里”“你不像是应该在家养的动物”)、PARO 的外观和举动不如真的竖琴海豹幼体可爱有关。\n- 一些研究比较了多种机器宠物,发现老年人更喜欢毛茸茸的机器猫、机器狗,他们希望机器动物做出像真实动物的举动来互动,希望机器动物摸起来是温暖的、身体内部摸起来是有弹性的。不切实际的外观、塑料或橡胶的质感、较大的速度和重量、声音太大等特征会让老年人有些反感,但互动能力可以抵消外观与质感的问题。\n- 一些媒体声称用机器宠物(尤其是 PARO)陪伴痴呆症患者有“伦理问题”。他们并没有很好地说明到底哪里有伦理问题。\n\n看起来,对于老年人(尤其是痴呆症患者),PARO 具有一定学习能力的人工智能、多样化的传感器(触摸、声音、光、位置)、个体化定制的反应并不比 Joy forAll 机器猫·机器狗的简单设计有用。后者只对触摸和声音做出响应,自带的动作也简单得多。当然,这可能是因为 PARO的海豹造型对老年人来说过于非日常。老年人期望的机器宠物的行为似乎主要是看向用户、翻身、摇尾巴、呼吸、在被抚摸时发出咕噜声、在用户腿上行走之类。", "task_type": {"major": ["问答"], "minor": ["知乎问答"]}, "domain": ["通用"], "metadata": "qid:544263925, aid:2630375183", "answer_from": "human", "human_verified": true, "copyright": "暂无版权及作者信息"} {"instruction": "鱼类和海洋里面的哺乳动物,会放屁吗?", "input": "", "output": "将放屁定义为有肛门的动物从肛门排出消化道内的气体(尤其是消化食物产生的气体),那么至少一部分鱼和海洋哺乳类会放屁。\n\n- 生活在浅水中的一些鱼会放屁。健康的金鱼通常不放屁,而是打嗝。消化出现问题的金鱼会放屁。\n- 南美洲的淡水鱼灰鳉食用藻类后在消化道里产生的气体跟身体的规模比起来相当多,由此导致的胀气会让它们浮在水面上难以下潜。它们通过放屁排气,放不出足够多的屁会高概率被捕食者攻击。\n- 生活在深水中的鱼从口和/或肛门持续排出消化道里的气体。持续排气算不算屁是个哲学问题,建议算。\n- 鲸的屁曾数次被拍摄下来。海豚们在海洋馆里放的屁就更多了。\n- 根据现场调查,海豹的屁有一股鱼味,很臭。\n- 绿色和平的工作人员曾比较 60 种动物的屁味,认为海狮的屁是这之中最臭的。\n\n00:17\n\n许多水生动物开发了屁在降低消化道内压力之外的用途,例如鲱鱼可以通过放屁产生高频率的声音进行交流。\n\n- 海牛以消化道中的气体提供部分浮力,在潜入水下前会从肛门排气。你可以称这种获取浮力的手段是憋屁。当海牛便秘时,屁会阻止它有效潜入水下,造成海牛头朝下尾巴朝上在水中摇摆。\n- 沙虎鲨使用和海牛相似的憋屁提供部分浮力,使它们可以悬浮在水中——与大部分鲨鱼不同。其他鲨鱼可能持续排出消化道里的气体。\n\n尚未发现水生动物用屁进行攻击。陆生动物在这方面走得很远。例如串珠草蛉幼虫可以用屁里的化学物质击倒白蚁并捕杀,释放位置合适时一屁可以击倒 6只白蚁,击倒的有效时间可长达 3 小时,通常就这样将白蚁熏死。\n\n \n\n鱼和海洋哺乳类之外的水生动物在放屁方面的情况不一而足。\n\n- 多孔动物、栉水母动物、扁盘动物、刺胞动物、水生扁形动物、水生线形动物、水生环节动物、棘皮动物等似乎不会放屁。\n- 水生软体动物似乎不会放屁。不过,有学者认为头足类的喷水推进是一种类似放屁的行为,可以叫做伪屁。\n- 现存的水生无颌类和海鞘会不会放屁还有待研究。\n- 小体型两栖类消化道里的气体会较为持续地从肛门排放出来。牛蛙之类体型较大的可以产生恶臭的屁。由于括约肌较弱,两栖类的屁通常无声。\n- 至少一部分海蛇、龟、蜥蜴、鳄鱼会放屁。蜥蜴通过放屁排出的不少气体是用嘴吞下去的。一部分陆生蛇类用放屁产生的噪音惊吓其他动物来自卫。\n- 现代水鸟在健康状态下不会放屁,因为它们的消化道菌群里没有产生大量气体的微生物。\n\n目前不知道鱼龙、蛇颈龙、沧龙等活着的时候会不会放屁,因为你无法期待屁的痕迹化石保留下来。非水生的非鸟恐龙和翼龙的情况也不清楚。大概至少有一些大型植食性恐龙能够产生消化道胀气。\n\n在水生动物之外,蝙蝠、树懒不会放屁。树懒消化道里产生的含甲烷气体会吸收入血并从呼吸系统排出。\n\n包括人在内,许多会放屁的动物也可以通过打嗝和呼吸排出消化道里的部分气体。", "task_type": {"major": ["问答"], "minor": ["知乎问答"]}, "domain": ["通用"], "metadata": "qid:523872349, aid:2405924211", "answer_from": "human", "human_verified": true, "copyright": "暂无版权及作者信息"} {"instruction": "没有线粒体的真核生物能合成血红蛋白吗?", "input": "", "output": "不用那么多前提,任何有核糖体的生物在适当条件下都可以合成血红蛋白。即使是本身基因组里没有血红蛋白相关基因的物种,也可以被转入相关基因,或是更简单地整点血红蛋白mRNA来。\n\n血红蛋白和类血红蛋白分子本来就广泛存在于自然界,在没有血液的植物、真菌、细菌们身上也有应用。没有线粒体的真核生物合成的血红蛋白往往跟你身上的血红蛋白有颇多的氨基酸差异,没有血液也没有肌肉的生物合成的血红蛋白与肌红蛋白往往无法区分,因此“类血红蛋白分子”更准确。\n\n- 根瘤菌与大豆组成的共生构造用根瘤血红蛋白降低游离氧到固氮酶可以接受的程度,供根瘤菌展开固氮活动。根瘤血红蛋白是根瘤菌和植物根系一起分泌的,根瘤菌那都别说线粒体了,压根就是个原核生物。\n- 在中学生物试卷里,你可能见过“将兔血红蛋白mRNA放进大肠杆菌提取液,合成出了兔血红蛋白”的题目,通常是选择题,让你选“原核生物和真核生物使用同一套遗传密码子”的。“大肠杆菌提取液”提供的主要是大肠杆菌的核糖体,这混合物里什么细胞核、线粒体都没有。\n- 在深海热泉生态系中,巨型管虫用极其复杂的血红蛋白运输二氧化碳、氧气、硫化氢,供其共生的细菌使用。血红蛋白还可以为厌氧生物运输或清除一氧化氮。\n- 我们已经发现厌氧古菌Methanosarcina acetivorans拥有类血红蛋白分子,用来隔绝氧气。\n\n因此,血红蛋白的起源可能能一直追溯到LUCA,那比线粒体的出现要早若干亿年。", "task_type": {"major": ["问答"], "minor": ["知乎问答"]}, "domain": ["通用"], "metadata": "qid:442469572, aid:1713035392", "answer_from": "human", "human_verified": true, "copyright": "暂无版权及作者信息"} {"instruction": "如何看待运动、健身加快了人体新陈代谢和身体损耗,导致寿命缩短?", "input": "", "output": "没那么简单。\n\n30 岁后,人的最大摄氧量通常每十年下降约10%,这和多种慢性疾病的患病率与死亡率、丧失独立生活能力的风险增加直接相关。缺乏运动是慢性病的主要原因之一。适度体育锻炼可以在一定程度上增加最大摄氧量,降低人死于高血压、2型糖尿病、高血脂、冠心病、中风、癌症的概率,可将全因死亡率降低约 30% 至 35%。与不定期进行锻炼的人相比,每周累计进行 150分钟以上适度体育锻炼(可以是 5 天间每天 30 到 45 分钟)的人的预期寿命更长,锻炼形式可以是简单的步行。\n\n- 13 项研究的结果表明,有规律的体育锻炼与预期寿命延长 0.4 至 6.9 年有关。\n- 11 项研究在此基础上考察了死亡率的混杂风险因素,显示有规律的体育锻炼可使预期寿命延长 0.4 至 4.2 年。\n\n“身体损耗”在材料供应充足、细胞功能正常时可以被身体自行修复。\n\n- 研究人员考察了 5823 名 20 到 84 岁成年人的 DNA,发现每周至少有 5 天进行半小时以上慢跑的女性或每周至少有 5 天进行 40 分钟以上慢跑的男性的端粒长度较大,相当于比自己年轻 9 岁的人的平均水平。\n- 端粒不是决定性因素,但可以在一定程度上显示细胞水平的氧化应激或炎症强度。\n- 研究人员比较了多个年龄段的人的心脏、肺活量、骨骼肌,发现经常锻炼身体的 75 岁老年人的心血管健康水平、肺活量、肌肉力量相当于 40 到 45 岁人群的平均水平。\n\n超重或多年不运动的人也可以通过每天步行 30 到 60 分钟来延长预期寿命。\n\n限制摄入的热量(包括减少每日摄入量和较长的饮食间隔)或增加运动量到刺激 Sirtuins蛋白活跃的程度,可以改善细胞的表观遗传衰老状况,延长多种实验动物的寿命。\n\n另一方面,过度锻炼,尤其是沉迷于减肥或增肌,有概率缩短人的寿命。这通常是循环系统出问题的概率上升所致。11项关于前运动员的预期寿命的对照研究显示,有氧耐力运动员的预期寿命较长,其他运动员的预期寿命有长有短。", "task_type": {"major": ["问答"], "minor": ["知乎问答"]}, "domain": ["通用"], "metadata": "qid:505535815, aid:2501159062", "answer_from": "human", "human_verified": true, "copyright": "暂无版权及作者信息"} {"instruction": "生物体可能产生金属单质吗,或者产生半导体,构建逻辑电路?", "input": "", "output": "那不是“可能”的程度,地球上的生物早就可以产生金属单质、半导体、逻辑电路了:\n\n- 地面上就广泛存在将金属氧化·还原来获取少许能量的微生物。你肚里大量存在的大肠杆菌就可以将汞单质氧化为二价汞离子并取得些微的能量,土壤里有多种细菌可以将二价汞离子作为电子传递链的电子受体,还原为汞单质。\n- 将金属离子还原为金属单质的细菌相对人类冶金工业来说非常分散,在短时间内不会形成肉眼可见的大量金属单质矿物,但随着时间可能积累并参与矿床的形成。异化金属还原菌能将溶液中的二价钯离子·三价金离子·一价银离子还原为钯、金、银纳米颗粒,对镉、锶等也有效,已经用于微生物燃料电池、污染治理和回收贵金属等研究。\n- 生物体内的半导体更是普及到好笑的地步,你身上就有的黑色素就是半导体。人们业已开发出基于姜黄和靛蓝(两种由植物合成的天然染料)的单层天然半导体二极管。\n- 硅藻、海绵、水稻等各式各样的生物体可以运用二氧化硅,海绵的二氧化硅骨针是高性能的生物光纤。\n- 你身上就有许多神经细胞能执行复杂逻辑电路元件的功能。人大脑皮层第 2 层和第 3 层的锥体神经元的树突臂中的每个微小区室都可以执行复杂的数学逻辑操作,允许单个神经元的树突进行异或运算。\n\n人类制造的各种金属物品可以被细菌腐蚀。二十世纪中叶,研究人员建立的模型是这样的:\n\n- 水中的氢会夺取金属表面的电子形成氢气,在金属表面形成“氢膜”阻碍金属的溶解。附着在金属表面的微生物可以利用体内的氢化酶将硫酸根还原成硫化氢、除去金属表面的氢气,让金属继续溶解侵蚀,硫化氢还能与多种金属发生反应。\n\n2016 年的新理论认为,附着在金属表面的细菌生物膜可以提取金属材料的电子来还原体内的硫酸根。这套系统非常古老,在人类出现之前就已经将二价铁-三价铁循环转换了数十亿年,支持着深海热泉•冷泉和地下的大片生态系。\n\n多细胞生物可以在身上运用金属离子,例如你的骨骼含有大量的羟基磷灰石,铁元素是血红蛋白、固氮酶、叶绿体蛋白等的金属中心。结合共生的细菌或从细菌那里转移的基因,就可以期待将金属单质附着在身体部件上。\n\n- 这不限于脊椎动物。2008 年,研究人员在 136 种昆虫、30 种蜘蛛、12 种环节动物(包括蚯蚓)、4 种蜈蚣的爪子尖端·颚部·尖牙中找到金属离子。这些结构很小,但在与环境接触时可以承受很大的应力。这些金属组成的不是像钉子那样有组织的原子簇,而是单个金属离子分别固定在适当的位置,种类主要是铁、铜、锰、锌。\n\n在切叶蚁的下颚上,成年后短时间内迅速积累的锌离子可以大幅提高其抗压强度。蜘蛛的尖牙也是在成年后一段时间内迅速积累金属离子的。", "task_type": {"major": ["问答"], "minor": ["知乎问答"]}, "domain": ["通用"], "metadata": "qid:483402010, aid:2092425142", "answer_from": "human", "human_verified": true, "copyright": "暂无版权及作者信息"} {"instruction": "人的神经信号也是电信号,为什么电磁干扰影响不到人?", "input": "", "output": "您的问题存在错误,电磁干扰是可以影响到人的。\n\n医疗扫描和经颅磁刺激使用的强磁脉冲经常引发副作用,包括而不限于睡眠障碍、头痛、头晕、抑郁、疲倦、发痒、注意力不集中、短暂的轻度躁狂发作、认知改变、暂时性听力损失、味觉变化(典型的是口腔出现金属味)、暂时性工作记忆受损,少量患者会晕厥。\n\n电磁场对人体的影响不限于神经系统。10特斯拉的磁场可以影响血液流动。随时间迅速变化的强电磁场可以在人体内产生热效应,例如微波炉可以很简单地杀人。磁星的超强磁场可以将人体拆成一条单原子线、加速到亚光速并在它的表层撞得剧烈爆炸。\n\n显然,人们经常被文艺作品里“只影响机器、不杀人”的“EMP”误导。其实,有封闭金属外壳或法拉第笼式构造并良好接地的机器比人更不怕电磁脉冲。\n\n很弱的磁场就能影响神经细胞的门控离子通道,还能上调大鼠脑神经细胞的活性氧生成水平,提高 DNA 断裂概率,引起更多神经细胞凋亡或坏死。\n\n磁脉冲照射可在脑组织中引起微电流,这微电流可以激活神经细胞,人已经用磁脉冲照射影响过许多实验动物和人的行为。较强的照射可以干扰实验动物或人的平衡感和方向感、诱发幻觉、产生强烈而不受控的恐惧和焦虑等情绪、导致神经调节紊乱和上面谈到过的各种副作用。\n\n静磁场可以影响人脑、可以干扰动物的神经功能。经颅静磁场刺激可以降低人运动皮层的兴奋性并暂时改变皮质内抑制系统的活动。在视觉皮层、顶叶皮层、前额叶皮层等处进行静磁场刺激或磁脉冲照射可以改变人的行为。强静磁场可诱发果蝇体细胞DNA 突变,这效应长期持续当然也可以致癌以下略。\n\n电磁场可以引起癫痫患者的癫痫发作,严重时可导致猝死。", "task_type": {"major": ["问答"], "minor": ["知乎问答"]}, "domain": ["通用"], "metadata": "qid:555222929, aid:2689385106", "answer_from": "human", "human_verified": true, "copyright": "暂无版权及作者信息"} {"instruction": "为什么灵长类的嗅觉都不强?", "input": "", "output": "“灵长类的嗅觉都不强”是个以偏概全的刻板印象,原猴亚目的嗅觉能力强,而人类的嗅觉能力其实并不弱。\n\n- “人类的嗅觉很差”是一条19世纪模因,来自Paul Broca。实情是,人类的嗅球只是“占整个脑的比例较小”,而不是比其它动物小,在实验室里测试特定的气味时,人类检测一些气味的能力强于老鼠和狗,三者各有自己擅长检测和不擅长检测的气味。\n\n试图解释“灵长类嗅觉退化”的早期学说是“嗅觉退化与视觉进化同步进行”。2004年,来自德国和以色列的一群科学家试验发现,灵长类嗅觉感受基因的缺失和三色视觉的获得之间存在某种联系。\n\n- 嗅觉感受基因是嗅觉的分子基础,截止2004年已经测序的人类·非人灵长目动物·老鼠有数量相近的嗅觉感受基因,不过人类的嗅觉感受基因中有60%不具活性,通称“假基因”。无尾猿的嗅觉基因有30%是假基因,老鼠的嗅觉基因有20%是假基因。科学家们调查了包括人在内的19种灵长目动物,结果发现欧洲猴子与无尾猿的嗅觉感受基因中假基因比例基本相同,而在美洲除吼猴外的灵长目动物里,嗅觉感受基因中的假基因比例少得多。\n- 这表明,欧洲猴类、无尾猿、吼猴的嗅觉各自出现了衰退。三者并没有很接近的共同祖先,但有一个共同的感觉特征:三色视觉。这需要三种视蛋白吸收不同波长的光线。无尾猿和欧洲猴类拥有全部三种视蛋白基因,大部分美洲猴类只有两种视蛋白基因。三色视觉的出现在灵长目中似乎发生了两次,一次发生在无尾猿和欧洲猴的共同祖先身上(约2300万年前),另一次发生在吼猴身上(约700万至1600万年前)。研究人员认为,三色视觉的出现与嗅觉的退化发生在同一时期,当动物的嗅觉退化时,听觉或视觉更发达的性状成为优势。\n\n后来,随着对原猴亚目的实验次数增多,“灵长类嗅觉不灵敏”的刻板印象受到了质疑。2018年,一些学者研究了来自24个灵长类物种的嗅觉感受基因,结果发现原猴亚目的嗅觉感受基因多达600~800个、平均约22.5%是假基因,而类人猿亚目的嗅觉感受基因是200~400个、平均约47.5%是假基因。\n\n- 在纳入了擅长嗅觉的原猴亚目灵长类的数据之后,动物的活动模式(昼间活动/夜间活动)与三色视觉并没有表现出“跟功能性嗅觉感受基因的数量在统计学上显著相关”(这和刻板印象里“夜行性动物更依赖视觉以外的感官”不一致),系统发育、鼻子结构(原猴亚目的鼻子有弯曲的构造)则显示出和功能性嗅觉感受基因数量的统计学关联。\n- 类人猿祖先分支的嗅觉感受基因的丢失率比灵长类的平均水平要高、和敏锐的视觉同步演化。\n- 嗅觉感受基因丢失率最高的是叶食性疣猴亚科,这可能与从吃水果到吃树叶的食性转变有关,气味信息对搜寻水果并判断其品质比较有用,对搞点树叶的重要程度有所下降。\n\n2012年的一项研究显示,吃水果的狐猴物种靠嗅觉识别其首选食物,吃树叶的狐猴物种依靠视觉和嗅觉结合来选择首选食物,杂食性的狐猴物种则靠视觉与嗅觉中的任何一个就能选中其首选食物。如果所有食物的品质都不错,吃树叶或杂食性的狐猴物种可以只靠视觉选出最爱吃的食物,吃水果的狐猴物种则仍然要靠嗅觉加视觉来判断。", "task_type": {"major": ["问答"], "minor": ["知乎问答"]}, "domain": ["通用"], "metadata": "qid:441129858, aid:1697299277", "answer_from": "human", "human_verified": true, "copyright": "暂无版权及作者信息"} {"instruction": "为什么鸟冬天要迁徙?", "input": "", "output": "一些鸟在冬天迁往气温较高的地区,可以获得比不迁徙的鸟更多的食物。在食物充足的情况下,许多候鸟会暂停迁徙。现存候鸟物种往往处于一部分种群迁徙、另一些种群不迁徙的状态。\n\n对于体型较大的鸟,成群飞行可以节约能量。组成 V 形编队的大雁比单独飞行节省 12% 到 20% 的能量。组队迁徙比零散地飞行去寻找食物更高效。\n\n当然,鸟本身未必明白这些情况。\n\n在食物总量不足的情况下,触发具体的鸟迁徙的因素通常是每天的白昼时长缩短到一定水平引起的激素水平变化。也有检测气温变化的鸟,这预计会受到气候变化的影响。\n\n鸟的迁徙可能首次出现于约八千万年前,在鸟类演化史上多次独立出现。在全球范围内、在鸟纲的五个主要的目级分类群中,迁徙行为总体上与更高的物种形成率、更低的物种灭绝率相关,从而导致更高的净多样化率。\n\n迁徙的鸟并不都会飞。许多企鹅能通过游泳迁徙超过 1000 千米。Dendragapus obscurus主要通过步行进行高度迁移。澳大利亚的鸸鹋在旱季会长途步行去寻找食物。", "task_type": {"major": ["问答"], "minor": ["知乎问答"]}, "domain": ["通用"], "metadata": "qid:566110887, aid:2755009503", "answer_from": "human", "human_verified": true, "copyright": "暂无版权及作者信息"} {"instruction": "为什么大部分动物没有彩色视觉,只有黑白视觉?动物进化出彩色视觉的意义在哪里?", "input": "", "output": "事实上,以已知的物种数量、个体数量、生物量计算,大部分动物是具有彩色视觉或检测颜色的感光能力的。\n\n- 物种数主要来自具有多色视觉的昆虫,个体数主要来自没有眼睛但能检测颜色的线虫,生物量主要来自具有多色视觉的鱼。\n- 以个体数量计算,有图像视觉的动物以彩色视觉为多;\n- 所有动物的大部分数量归于线虫。秀丽隐杆线虫可在白光或一定比例的蓝黄光下检测铜绿假单胞菌等细菌反射的光的波长比例并避免摄入这类对它们有毒的细菌,这涉及的分子机制与其他动物的视蛋白完全无关,涉及的基因至少有 jkk-1 和 lec-3。从其他线虫到人的多种动物有这些基因,但它们在人身上似乎不用于检测光波长的比例。\n\n对图像视觉来说,色觉来自多种具有不同吸收峰的视蛋白分子。在陆生动物中,只有节肢动物和脊椎动物有基于视蛋白的色觉,但这就足够用物种数量淹没其他动物分类群。\n\n- 节肢动物的共同祖先至少有 3 种视蛋白。现代螯肢亚门(蜘蛛、蝎子、蜱、螨等)和泛甲壳动物(甲壳亚门、六足亚门)的大部分物种有视觉且有色觉。\n- 大多数现存昆虫有两种视蛋白,一种主要对应绿光和黄光(峰值波长 550 纳米),一种主要对应蓝光和紫外线(峰值波长小于 480 纳米),可有效地与开花植物互动,尤其是带有紫外线标记的花朵。\n- 1914 年,卡尔·冯·弗里希通过实验证明蜜蜂可以区分蓝色和任何深度的灰色,这是人们第一次知道非人动物有色觉。此后,他继续研究蜜蜂,证明蜜蜂使用象征性的舞蹈传递信息。1973 年,弗里希因“对比较行为生理学的贡献与对昆虫沟通方式之领导性的研究”而和康拉德·洛伦兹、尼古拉斯·廷贝亨一同获得诺贝尔生理学或医学奖。\n- 脊椎动物有 5 种视蛋白,其中 4 种在距今约 3.5 亿年前已经存在。两栖动物与羊膜动物的共同祖先(约 3.5 亿年前)具有四色视觉。现代脊椎动物的大部分物种有三色视觉,少数有二色视觉,单色视觉和无视觉是极少数。\n- 一些人以为猫狗是单色视觉,其实它们是二色视觉,猫可以区分红色与蓝色,狗可以区分红色与紫色。\n\n \n\n光在水中传播时,波长较长的红黄光被吸收的速度快于波长较短的蓝绿光,环境光的平均波长随水深增加而变短。不同种类的视蛋白可以对应不同深度的常见光线波长,一旦突变出来就可以在一定范围内提供更清晰、更灵敏的视觉,对捕食、自卫、求偶产生一定好处,容易在种群中延续并蔓延。感光细胞使用不同波长的视蛋白,经神经系统处理,在某个阶段就会产生色觉。\n\n色觉有助于更好地识别不同材质的表面,有助于检测捕食者、食物、配偶,获取的信息可用于战斗、逃跑、觅食、躲藏等,可以被广义自然选择青睐。许多现存动物对波长450 纳米以下的光(尤其是直射光)和波长 450 纳米以上的光(尤其是反射光)有不同反应。\n\n线虫的光波长比例检测能力有类似的结果。", "task_type": {"major": ["问答"], "minor": ["知乎问答"]}, "domain": ["通用"], "metadata": "qid:564843295, aid:2746174303", "answer_from": "human", "human_verified": true, "copyright": "暂无版权及作者信息"} {"instruction": "哪些蜘蛛具有群居习性,其中具备社会性的又是哪几种?", "input": "", "output": "在已知的约 50500 种蜘蛛里有至少数百种群居,其中至少 23 到 25 种有一定的社会性。人工环境可以让更多蜘蛛物种建立群体。阿格纳森等估计蜘蛛 18到 19 次独立演化出社会性。\n\n姬蛛科有至少 11 到 12 个具有一定社会性的群居物种,可能涉及 8 到 9 次独立演化出社会性。姬蛛科 Anelosimus 属有至少 8个具有一定社会性的群居物种,主要分布在美国和马达加斯加。\n\n- 生活在南美洲的 Anelosimus eximius 是人最了解的社会性蜘蛛物种,它的网可以覆盖整个树冠、包含数千到五万个体,这种网合作捕猎的效率在 1000 只左右的群体里分摊到个体的效费比最高。\n\n- 在圈养环境,更多的姬蛛科物种可以形成小的群体、共享网和部分食物。\n\nAchaearanea disparata、Achaearanea vervortii、Aebutina binotata、Agelenaconsociata、Australomisidia ergandros、Australomisidia inornata、Australomisidiasocialis、Delena cancerides、Mallos gregalis、Parasteatoda wau、Parawixiabistriata、Theridion nigroannulatum 是群居的,有不同程度的社会性或准社会性。\n\n- Agelena consociata 可以 25 到 40 只合作捕杀较大猎物并一起修复被大雨损坏的巢穴。\n- Delena cancerides 可以组成至多约 300 只的群体合作捕猎,群体内可分享猎物,群体间则经常厮杀。\n- Mallos gregalis 可以数千只共同建造片状的网来捕猎。\n- Parawixia bistriata 可以单独活动,也可以形成数十到约 500 只的群体并合作捕猎,通常在干旱环境里合作较多。在群体内存在争夺设置网的空间、无网者尝试夺取有网者的猎物等现象。\n\nPhryganoporus candidus 的受精雌性可以建立约 100 只后代组成的巢穴,其信息素可以让幼体合作捕猎并分享猎物。成体之间不进行合作。\n\nPhiloponella Republicana 可以形成数只到上百只不等的群体,这种群体很容易解散。\n\n主要吃植物的 Bagheera kiplingi 通常单独活动,也能组成数百只的小群体,一些个体会尝试保护卵和幼体。这物种有时也吃昆虫或同类相残。\n\nLeucauge 属的多种蜘蛛可以形成松散的群体,这种群体缺少合作防御之类。\n\nStegodyphus 属的 S. lineatus,S. mimosarum,S. dumicola 是群居、社会性的,群体的约 88% 是雌性,雌性中约40% 的个体负责繁殖,其余雌性参与照顾同一巢穴的幼体。\n\n- 群体的所有成年个体都会参与巢穴的维护修理,可以合作捕捉猎物或抵御蚂蚁等捕食者的攻击,可以将消化道内容物吐给同一巢穴的其他个体。\n- 个体的体型和性格会影响其参与合作行为的频率。\n- 雌性个体的寿命约 1 年,寿命将尽的雌性会被同一巢穴的幼体食用,无论那是不是自己的后代。一些成年雌性会在交配后离开巢穴、在附近建造新的巢穴。\n- 雄性个体的寿命比雌性短,在一生中大部分时间里不会离开出生的巢。在繁殖季节,雄性会彼此竞争,少数雄性会离巢并加入附近其他雌性建立的新巢穴。\n\n诸如此类。蜘蛛这样给人孤独印象的生物演化出社会性的难度看起来其实不高。", "task_type": {"major": ["问答"], "minor": ["知乎问答"]}, "domain": ["通用"], "metadata": "qid:562915197, aid:2740441374", "answer_from": "human", "human_verified": true, "copyright": "暂无版权及作者信息"} {"instruction": "污染物沿着食物链传递,其浓度在较高营养级中一定增大吗?\n19.生物富集的效率可以用生物富集系数 (bio-concentration factor, BCF) 进行量化, 计算公式如下:BCF = 生物体中某种物质的浓度/环境中同种物质的浓度, 以下有关叙述错误的是\nA. 生物富集的过程一般是沿着食物链和食物网进行的\nB. 生物富集效率会受 到生物特性、有害物质性质及环境因素的影响\nC. BCF 数值越大,说明该物质越容易在生物体内分解\nD. 污染物沿着营养级传递过程中, 其浓度在较高营养级中不一定增大\n19题D 哈尔滨2022市统考生物试卷", "input": "", "output": "不一定增大。\n\n选项 B 已经概括性地解释了选项 D。生物富集的效率受生物的特性、有害物质的性质、环境因素影响,效率可以低到生物富集系数(BCF)等于或小于1,高营养级生物体内的污染物浓度不变或减少。\n\n一些生物很擅长排出特定污染物,或是很不擅长吸收·储存特定的污染物。一些生物能将污染物分解为无害产物。一些生物不同部位的污染物浓度有明显差异、吃这生物的其他生物并不吃高浓度的部位。这些情况都能实现较高营养级的污染物浓度不变或减小的效果。\n\n例如有机磷农药乙拌磷亚砜及其有毒代谢产物在欧亚鲤(常见的养殖鲤鱼)体内的 BCF 小于 1,六氯环己烷(bhc,666)在真无须魮的鳃的 BCF 为0.076、在肌肉为 0.043、在肠道为 0.132,实验所用的六氯环己烷浓度已经接近对这动物的 LD50。\n\n讲生物富集而不讲这些情况的生物教育是个谜。", "task_type": {"major": ["问答"], "minor": ["知乎问答"]}, "domain": ["通用"], "metadata": "qid:550614820, aid:2651645242", "answer_from": "human", "human_verified": true, "copyright": "暂无版权及作者信息"} {"instruction": "坚持跑步为什么会提高大脑认知能力,有什么科学依据吗?", "input": "", "output": "科学依据很多。不限于跑步,坚持体育锻炼似乎可以改善人一生中的认知健康水平。\n\n- 神经影像学数据显示,健康的心肺功能可以延缓衰老过程中与年龄相关的脑组织(灰质和白质)损失,有概率改善特定脑结构的健康程度 ;\n- 有氧运动可以增加流过大脑的血量,尤其是在涉及信息检索和执行功能的脑区,可能有助于降低患上抑郁症或神经退行性疾病的风险;\n- 有氧运动可能影响表观遗传修饰和分子通路、线粒体的行为、涉及脑源性神经营养因子 (BDNF) 的分子系统。通常,在半年间定期跑步和/或进行其他有氧运动,就能让所有年龄段的平时不怎么运动的人自己体验到一些改善;\n- 有氧运动可以改善前额叶和顶叶在认知挑战任务中的表现;在年轻人身上,这不止能提高快速反应过程中行动检测的效率,还能在需要灵活改变策略的任务期间改善传统上称为“注意力分配”的过程;\n- 电生理学研究也支持“心肺健康或积极的生活方式可以改善认知相关的神经活动”。\n- 限制摄入的热量(包括减少每日摄入量和较长的饮食间隔)或增加运动量到刺激 Sirtuins 蛋白活跃的程度,可以改善细胞的表观遗传衰老状况,延长多种实验动物的寿命。\n\n要注意的是,跑步在“提高大脑认知能力”方面的作用并不比其他有氧运动优越。跑得太多、配套设施不合适、姿势不对、运气不好等因素可能损伤人的膝盖、脚踝等部位,有概率引起严重的问题。每天步行30 分钟就足以让 50~55 岁的女性抑郁症患者更有活力和信心,大概对其他年龄段、其他性别的抑郁症患者也有效,亦能帮助预防抑郁症。\n\n- 血液中的脑源性神经营养因子水平与海马体积有关,有氧运动上调血清中的脑源性神经营养因子水平,在统计上与儿童和老人的海马体积增加有关,在儿童身上与更好的记忆表现有统计学关联。但其数值还没到值得将全国的小学生都拉去每天跑十圈的地步。\n- 一项研究考察了 1206 名健康人的步行耐力与认知功能的关系,受试者平均年龄 28.8 岁,45.5% 为男性。结果显示较高水平的步行耐力与整体认知功能增强有统计学关联。性别、年龄、教育程度、BMI、HBA1c、动脉血压等变量并没有改变结果的整体模式。\n- 一项研究调查了 206 名成年人在六个月内每周至少锻炼四天的情况。受试者平均年龄 66 岁,在实验开始时、三个月后、六个月后各接受思维和记忆测试,并以超声波监测流过大脑的血量。有氧运动计划从每天 20 分钟开始,在六个月间增加到 40 分钟。实验结束时,受试者在执行功能测试中的成绩改善了 5.7%,在语言流畅性方面的成绩改善了 2.4%,大脑血流量增加 2.8%~\n- 埃里克森等调查了 165 名 59 至 81 岁之间的、认知健康的老年人,通过最大分级运动测试评估他们的心肺健康水平,让他们执行空间记忆任务,并使用功能磁共振成像 (fMRI) 测量他们的海马体积。结果表明,心肺健康水平较高的受试者双侧海马体积都较大,并与更好的空间记忆表现统计学相关。\n- 一项研究对 120 名老年人进行随机对照设计,发现运动组的海马体积相比对照组增加了 2%,并表现出更高水平的血清脑源性神经营养因子水平和更大幅度的空间记忆改善。\n- 一项研究将 124 名年龄在 60 至 75 岁之间的老年人随机分配去接受为期 6 个月的步行训练(有氧运动)或柔韧性训练(非有氧运动)。结果表明,步行组在一系列认知任务中的表现改善,而柔韧组没有这种现象。\n\n总之,有氧运动对正常衰老的人和患有阿尔茨海默症、精神分裂症等各种脑部疾病的人群的认知能力和大脑健康有相似的益处。\n\n历史上,人们曾经担忧跑得太多会损伤大脑。针对从意大利到挪威的赛程 64 天的 4487千米长跑比赛的参与者的一项研究显示,这样的超级长跑似乎会导致人的大脑暂时缩小,受影响最大的是视觉皮层(可能是因为铺装路面过于千篇一律),但参与者的大脑在比赛结束后半年内恢复原体积。参与者的膝盖等部位的软骨在前2500 千米赛程内呈现一些损伤和退化,那之后徐徐修复。\n\n动物的脑可以在和外界或自身互动的过程中改变结构和功能。过去一百多万年间,基于身体运动的战斗、逃跑、探索、觅食、自卫、求偶、发明、仪式、舞蹈等行为,在人的生存、适应、发展过程中发挥了重大的作用,狩猎在很大程度上塑造了人。二十世纪的理论认为,涉及上述行为的脑区,尤其是在记忆处理中有基本作用的海马体,受身体运动的影响十分明显;其余脑区出于能量效率,在演化过程中也会受到身体运动的影响。近年来,更多的研究显示人的“看起来很简单的身体运动”会激活巨大范围的神经细胞,旋转眼球会激活约一半神经元,那么身体运动的影响就更没有特别的边界了。", "task_type": {"major": ["问答"], "minor": ["知乎问答"]}, "domain": ["通用"], "metadata": "qid:496662091, aid:2229944422", "answer_from": "human", "human_verified": true, "copyright": "暂无版权及作者信息"} {"instruction": "为什么海洋中动物的生物量大于植物?", "input": "", "output": "海洋植物只占海洋生产者的一小部分,已知海洋动物生物量不止超过海洋植物生物量,更超过海洋生产者的已知生物量。Bar-On 等估计消费者占海洋生物量的 80%以上,生产者占陆地生物量的 95% 以上。不计尚未搞清楚的部分,当前陆地生物量约为海洋生物量的 80 倍,陆生植物占地球总生物量的约80%,主要是大型维管植物(海洋里稀缺的类型)。超过 70% 的动物生物量存在于海洋。\n\n这可能有以下原因:\n\n### 一、海洋单位面积的光合生产力较低\n\n统计显示,在海洋和陆地环境中,当地光合生产力越高,当地生产者与消费者的比率越大。\n\n2018 年估计的海洋总初级生产力(485 亿吨碳每年)与陆地总初级生产力(564亿吨碳每年)数量级相同,而海洋的表面积是陆地的两倍以上,海洋单位面积的生产力较低。这可以归因于水的透光性差、光合生物需要的多种矿物质在远海海水中的浓度低,海洋的大部分面积不利于大型光合生物(尤其是植物)生长。\n\n那么,当地光合生产力平均而言明显低于陆地环境的海洋环境理应具有较少的总生物量和较高的消费者比例。\n\n### 二、海洋光合生产量更大比例地输入消费者\n\n平均而言,大约 10% 的能量从海洋生产者转移到各级消费者,大约 1% 的能量从陆地生产者转移到各级消费者。\n\n- 这部分归因于陆地大型维管植物木质化的茎。对消费者来说,木质素不易分解,高大的茎还让树叶之类结构也变得不易靠近。陆地木本植物的周转速度很慢,它们的生物量代表数年、数十年乃至千百年间初级生产的积累。\n- 相比之下,海洋生产者的周转速度要快得多。洋流使有限的营养物质在海洋中高度流动,而且以单细胞生物为主的海洋生产者很容易被消费者取食。结果,海洋生产者用每一刻较小的生物量产生了庞大的年均初级生产。\n\n### 三、海洋环境有更多的变温动物、更大的捕食者-猎物体型比\n\n海洋基础消费者主要是节肢动物、软体动物等变温动物。变温动物不需要损耗多少能量来维持体温。另一方面,海洋哺乳动物为保暖而需要更大的体型。\n\n浮力降低了大型动物支撑自身重量的难度、有助于优化大型捕食者的运动速度,长距离移动的能耗也较低。\n\n理论支持这产生“头重脚轻”的营养金字塔。", "task_type": {"major": ["问答"], "minor": ["知乎问答"]}, "domain": ["通用"], "metadata": "qid:559053771, aid:2716731910", "answer_from": "human", "human_verified": true, "copyright": "暂无版权及作者信息"} {"instruction": "人类更适合吃肉还是吃植物?", "input": "", "output": "现代人适合吃肉并搭配一些植物,植物可提供膳食纤维、维生素 C 等。\n\n古人类更适合吃肉。\n\n总体上,人类的饮食倾向是高度可变的。住在江河湖海附近的古人类吃了很多鱼和贝类。生活在森林等植被丰富地区的古人类吃了很多水果、块茎和蜂蜜。现代狩猎采集者哈扎人吃的东西随季节改变,总体上生存所需的能量有一半左右来自动物肉,另一半来自植物和蜂蜜。现代人亦可在很大范围内选择饮食——但是,突然改变长期适应的饮食习惯会影响消化道菌群,有概率造成疾病。\n\nMiki Ben-Dor 博士和他的同事从遗传学、新陈代谢、生理学、形态学、古生物学等领域的约 400 篇科学论文中收集了 25条重要证据,支持石器时代的古人类是肉食性动物。\n\n例如:\n\n- 与杂食动物和大部分肉食性动物相比,人胃液的酸性很强。产生和维持这样的强酸胃液需要大量的能量,其功能主要是消灭不新鲜的肉里的常见细菌。古人类狩猎的大型动物体重远超整个小群体每日所需的量,可以吃上几天甚至几周,使得古人类经常食用含有大量细菌的不新鲜肉类,演化青睐强酸胃液。\n- 人的脂肪细胞结构是肉食性动物的典型配置。杂食动物的脂肪储存在数量相对较少的大型脂肪细胞中,而包括智人在内的肉食性动物将脂肪分散在大量小型脂肪细胞内。\n- 人类基因组的一些基因被关闭来对应富含脂肪的动物性饮食,黑猩猩基因组里的同种基因则激活来对应富含碳水化合物的植物性饮食。\n- 史前人类骨骼中的同位素比例和古人类遗迹中的武器、动物残骸化石表明,古人类经常猎杀脂肪含量很高的大中型陆生动物,他们生存所需的能量可能有 70% 以上来自大中型动物。\n- 化石证据显示,古人类的猎杀对全新世大灭绝做出了贡献。\n\n历史上,人们经常幻想人类的演化归功于杂食的灵活性。现在看来,人类恐怕是狮子和鬣狗的同路人。植物在石器时代末期才成为人类食物的主要组成部分,那时人类居住区域周围动物的平均体重早已从约500 千克暴跌到不足 50 千克。\n\n遗传变化的证据和用于加工植物的独特石器的出现显示,约 85000 年前的非洲人、约 40000 年前的欧洲人和亚洲人逐渐增加了植物性食品的消费。\n\n相比之下,人类以动物性食品为主的岁月持续 140 万到 200万年。那段时期,无论当地的生态条件如何,在石器中都观察到长时间的相似性和连续性:用于杀戮和处理肉类。\n\n显然,虔诚的素食者不愿相信他们的祖先是地球上从未有过的疯狂掠食者,这个物种不但捕猎大中型动物,还在猎物的平均体型缩小后使用弓箭与陷阱继续猎杀、通过刀耕火种对植物进行大规模摧毁。从人类崛起到现在,野生植物的生物量至少减少了一半。\n\n当然,他们大概可以从古人类也吃植物里获得些许安慰。", "task_type": {"major": ["问答"], "minor": ["知乎问答"]}, "domain": ["通用"], "metadata": "qid:559625647, aid:2719278840", "answer_from": "human", "human_verified": true, "copyright": "暂无版权及作者信息"} {"instruction": "生物会对遗弃的巢穴重复利用吗,比如蚂蚁;或者生物会利用其他生物的巢穴吗?", "input": "", "output": "会,很常见。\n\n老鼠经常在感到危险或巢穴受损时逃走,并在不久之后返回、对受损的巢穴进行修补、重新住下。老鼠选择筑巢地点时会考虑安全、保温、获取食物和水的难度,会在巢穴里使用报纸、破布、塑料、树叶等材料保温。为觅食等活动暂时离开巢穴时,老鼠会沿途留下气味标记,以便原路返回。鉴于合适的筑巢地点有限,老鼠改变筑巢地点时优先选在旧巢穴附近;一窝老鼠被赶走后,新来的其他老鼠(无论是不是同一物种)有概率选择同一位置筑巢。老鼠经常在人的“巢穴”里筑巢。\n\n在雨季,许多蚂蚁会将新热带白蚁 Nasutitermes corniger (Motschulsky)的巢穴当作庇护所,可能顺便吃掉一些白蚁工蚁(不论死活),同时为白蚁提供富氮营养并改善巢穴的防御水平,与白蚁共生的最常见的蚂蚁物种是 Monadsbispinosa (Oliver),它们不攻击活的白蚁。\n\n一些地下白蚁会利用蚂蚁的部分巢穴。白蚁经常在人的“巢穴”里筑巢。\n\n许多蜥蜴和一些蜘蛛会利用蚯蚓挖的地洞。\n\n地势好的山洞·水下洞穴可被许多不同生物运用,体型显著不同的动物可共用较为宽敞的洞穴。\n\n大部分洞穴是黑暗、阴冷、潮湿的,历史上可能有熊、洞熊、洞狮、洞鬣狗等动物居住,经常易主(尤其是在其中一些物种灭绝之后)。一万多年前,古人类可能为了制造屋顶、毯子、地毯之类击杀洞狮,与气候变化一同将其抹消。", "task_type": {"major": ["问答"], "minor": ["知乎问答"]}, "domain": ["通用"], "metadata": "qid:559636235, aid:2719363915", "answer_from": "human", "human_verified": true, "copyright": "暂无版权及作者信息"} {"instruction": "生物的进化应该是趋利避害才能延续下去,可为什么人类在惊恐的时候会僵住任人宰割,而不是撒腿就跑?\n为什么惊恐的时候明明撒腿就跑比僵住动不了任人宰割存活的几率要大得多,为什么依然进化成动不了呢???", "input": "", "output": "在危险情况下难以行动,吓呆(冻结行为/freezing behavior, 冻结反应/freezeresponse),是应对捕食者的一系列行为的组成部分,比人类、比哺乳纲更古老。各种动物经常将其拿来应对捕食者之外的有害事物、同类攻击等生存威胁。\n\n演化青睐有助于提前发现捕食者并降低被捕食者发现的概率的行为。动物检测到远处出现疑似捕食者的迹象时,“吓呆”会减少自己的动作(许多捕食者能识别远处的物体移动和声音)、促使自己的感官关注该迹象、增强收集信息的敏感度。有复杂感官的动物通常会朝疑似捕食者所在的方位瞪大眼睛、竖起耳朵之类。这是古老、简单而有效的被动防御策略,经常搭配其他适应性反应,例如啮齿类在检测到蛇时会竖起毛发增加散热,这略微有助于降低被红外线检测发现的概率。\n\n动物检测到捕食者正在对自己发动攻击或距离靠近到危险的程度,会触发装死、尖叫、战斗或逃跑反应等。“吓得完全无法移动甚至昏倒”有助于装死。具体什么样的距离算是危险、要不要装死,取决于自身的物种和身体状况、捕食者的物种和个体特性(形状、体型、移动方向、速度等)、自身的生活经验等。战斗或逃跑是常见的主动防御策略。\n\n- 实验显示,将老鼠放在离猫很远的盒子里,老鼠会“吓呆”或逃跑,将老鼠放在离猫较近的盒子里,老鼠会惊慌失措、显示出战斗或逃跑反应(布兰查德等,1990 年)。\n- 装死适合对付对自己不熟悉、有一定认知能力来辨认猎物死活、没有足够的认知能力来识破装死、不爱补刀、不喜欢死猎物(因为有大量腐生微生物和/或腐败产生的有害物质等)的捕食者。\n- 尖叫可以提醒同类并吸引捕食者的注意力,在群居动物身上尤其有用:可以提高血亲逃生的概率,这会提高相关基因在种群中长期存续的概率。\n\n动物被捕食者抓住或咬住会触发强效镇痛,抑制在平时会导致跛行或倒地的疼痛造成的影响、支持全力抵抗或逃跑。也有一些动物会在这阶段再次触发“吓呆”,这有助于装死。\n\n人的“吓呆”关系到基底外侧杏仁核复合体与海马体,这些结构(或其类似构造)受损或被抑制的动物会不容易吓呆,乃至无法吓呆。\n\n一些学者建议将冻结反应和战斗或逃跑反应合称“战斗、逃跑或冻结反应/fight-flight-or-freeze response”。\n\n题目里“人类在惊恐的时候会僵住任人宰割,而不是撒腿就跑”的说法并不适用于触发战斗或逃跑反应的人,后续的“惊恐的时候明明撒腿就跑比僵住动不了任人宰割存活的几率要大得多”同样有适用范围。站着不动或缓慢后退有概率让捕食者放弃攻击。许多捕食者,包括持有远程武器的早期智人之类,擅长从背后击杀逃走的大中型动物。而且,在面对难以抵抗的捕食者时,让少量个体吸引其注意力、帮助其他个体逃跑是可行的策略——高声尖叫可以警告同伴并吸引捕食者的注意力,你可以看到人类中的一部分个体有这种行为。\n\n题目里“趋利避害才能延续下去”的说法是对进化的古老误会之一。其实进化是很凑合的。地球上数量最繁盛、生殖最成功的类群(以个体数是病毒、细菌,以生物量是维管植物,在动物范围内以个体数是线虫)普遍缺少自主快速运动的能力,比起在环境不适合或出现敌害时逃走,它们更多地改变自己体内的生化反应和化学平衡·自己身体的一部分材料及其性能来对抗环境变化与敌害,以单纯的繁殖速度补上被环境和敌害消灭的部分。现代海洋里的细菌每天被噬菌体消灭约40%,其年均数量的波动跟这几乎无关。", "task_type": {"major": ["问答"], "minor": ["知乎问答"]}, "domain": ["通用"], "metadata": "qid:559888069, aid:2722374344", "answer_from": "human", "human_verified": true, "copyright": "暂无版权及作者信息"} {"instruction": "为什么大自然要让人类女性生殖时要承受巨大的痛苦,而没有将这种痛苦进化消失掉?", "input": "", "output": "大自然不负这个责任。历史上大部分时候,古人类分娩没有现在这么痛苦,选择压不强。最近数百年里,站着、坐着、蹲着分娩的难度及其带来的痛苦仍低于仰卧分娩,尤其是站着分娩允许重力协助、骨盆带自然活动(Dundes,1987年;Gupta 与 Nikodem,2000 年;Michel 等,2002年)。古埃及壁画、古欧洲浮雕、阿兹特克艺术品都曾描绘女性站着分娩。十九世纪末到二十世纪初的粗糙医学一度造成产妇死亡率上升。\n\n对双胞胎的研究显示,人类女性的骨盆宽度受生活环境的影响似乎超过基因造成的影响。农业时代的饮食变化可能比骨盆宽度的遗传因素造成了更多的产科问题。约12000 年的农业社会生活尚未让人的骨盆宽度重新优化到适合新生儿的大头,显示当前状态可以算是略有不利的中性性状,能在种群中长期传递。\n\n将人类女性当前的骨盆再加宽几厘米以扩大产道,并不会和“产科困境”假说设想的那样妨碍双足行走,声称现在的宽度有任何演化上的好处,都很可疑。非洲南部石器时代晚期的人群似乎搞出过身高很低而骨盆很宽的女性,当时他们经常长距离迁徙,化石并未显示这些人在行走方面有什么困难。生物力学也不支持“加宽骨盆会增加运动成本”的想法。\n\n现代人掌握解剖学与遗传学知识的时间太短,还不够对自己进行人工筛选,目前更多地以剖腹产解决难产问题,这又会影响筛选的效率。现代人对抗分娩痛苦的有效办法包括麻醉,这比自然界的大部分动物更好地缓解了疼痛,风险相对可控。\n\n- 类人猿的足月妊娠所需时间是灵长类里最长的,黑猩猩、倭黑猩猩的正常妊娠期约 39 周,人的正常妊娠期约 37 到 41 周。黑猩猩、倭黑猩猩的新生儿像人新生儿一样缺乏独立生存能力,需要母亲提供大量照顾。不过,这两种类人猿并没有现代人这么高的难产率,它们的产道比新生儿尺寸明显更宽。人的祖先在数百万年间也如此。\n- 正常晚期智人新生儿的大脑体积是成年时的 30% 左右,正常黑猩猩新生儿的大脑体积是成年时的 40% 左右,卷尾猴新生儿的大脑体积是成年时的 50% 左右,人脑已经因成年时的大体积而做出妥协。\n- 人大脑生长速度的峰值出现在出生前的几个月间,与生下来就能行走的动物类似(Halley,2017 年),这方面仍有为分娩让步的空间。\n- 人类胎儿的能量需求和母体能安全地提供的代谢率与散热能力的关系似乎更直接地决定了胎儿能达到的体重。这方面的统计相关性也出现在没有骨头限制产道的鲸类身上(Sacher 和 Staffeldt,1974 年)。\n- 提出“产科困境”假说时使用的统计数据可能不反映历史上大部分时候的状况。", "task_type": {"major": ["问答"], "minor": ["知乎问答"]}, "domain": ["通用"], "metadata": "qid:558684347, aid:2722377297", "answer_from": "human", "human_verified": true, "copyright": "暂无版权及作者信息"} {"instruction": "运动会加快细胞新陈代谢,那是不是意味着越运动越容易狗带?", "input": "", "output": "不是。“加快新陈代谢”跟寿命或死亡率没有简单的线性关联。\n\n缺乏运动是慢性病的主要原因之一。30 岁后,人的最大摄氧量通常每十年下降约10%,这和多种慢性疾病的患病率与死亡率上升、丧失独立生活能力的风险增加直接相关。适度体育锻炼可以在一定程度上增加最大摄氧量,降低人死于高血压、2型糖尿病、高血脂、冠心病、中风、癌症的概率,可将全因死亡率降低约 30% 至 35%;与不定期进行锻炼的人相比,每周累计进行 150分钟以上适度体育锻炼(可以是 5 天间每天 30 到 45 分钟)的人的预期寿命更长,锻炼形式可以是简单的步行。\n\n- 13 项研究的结果表明,有规律的体育锻炼与预期寿命延长 0.4 至 6.9 年有关。\n- 11 项研究在此基础上考察了死亡率的混杂风险因素,显示有规律的体育锻炼可使预期寿命延长 0.4 至 4.2 年。\n\n“运动带来的身体损耗”在材料供应充足、细胞功能正常时可以被身体自行修复。\n\n- 研究人员考察了 5823 名 20 到 84 岁成年人的 DNA,发现每周至少有 5 天进行半小时以上慢跑的女性或每周至少有 5 天进行 40 分钟以上慢跑的男性的端粒长度较大,相当于比自己年轻 9 岁的人的平均水平。\n- 端粒不是决定性因素,但可以在一定程度上显示细胞水平的氧化应激或炎症强度。\n- 研究人员比较了多个年龄段的人的心脏、肺活量、骨骼肌,发现经常锻炼身体的 75 岁老年人的心血管健康水平、肺活量、肌肉力量相当于 40 到 45 岁人群的平均水平。\n\n超重或多年不运动的人也可以通过每天步行 30 到 60 分钟来延长预期寿命。\n\n限制摄入的热量(包括减少每日摄入量和较长的饮食间隔)或增加运动量到刺激 Sirtuins蛋白活跃的程度,可以改善细胞的表观遗传衰老状况,延长多种实验动物的寿命。\n\n另一方面,过度锻炼,尤其是沉迷于减肥或增肌,有概率缩短人的寿命。这通常是循环系统出问题的概率上升所致,主要原因不是代谢率高,而是心脑血管压力大,次要原因是肠道的应激反应,这些问题可以被饮食、药物等控制。11项关于前运动员的预期寿命的对照研究显示,有氧耐力运动员的预期寿命较长,其他运动员的预期寿命有长有短。", "task_type": {"major": ["问答"], "minor": ["知乎问答"]}, "domain": ["通用"], "metadata": "qid:561342429, aid:2725438586", "answer_from": "human", "human_verified": true, "copyright": "暂无版权及作者信息"} {"instruction": "第一个产生抗体的物种是那个?", "input": "", "output": "分子生物学证据显示,第一种产生抗体的动物可能是某种目前尚未找到化石或找到了化石但未确定免疫功能的有颌鱼,可能是盾皮鱼或软骨鱼,生活在距今超过 4亿年前。估计这种有颌鱼的体型较大、单次繁殖产生的后代数量较少,较强的免疫功能在一定程度上有助于可育后代的数量。\n\n- 一个次要证据是,某些盾皮鱼似乎采用体内受精和胎生 。\n- 七鳃鳗的适应性免疫系统不产生抗体、产生可变淋巴细胞受体。盲鳗之类似乎也如此。估计抗体的出现不早于脊椎出现。\n- 现存软骨鱼的抗体的一些特征看起来比硬骨鱼的抗体更不像哺乳类的抗体。现存硬骨鱼的抗体显示,远古硬骨鱼可能首创了现存哺乳类的抗体的一些特性(尤其是铰链区) ,并创造了一些看上去后来被四足形类随机丢弃的特性。哺乳类有已知最复杂的铰链区的结构。\n\n类似抗体的东西比“距今 4 亿年前”更古老。\n\n- 原核生物的表面蛋白的免疫球蛋白样结构域显示出类似真核生物免疫球蛋白的 β 链二级结构,可粘附在特定的宿主细胞上、抵御特定抗生素、对特定宿主致病。贝特曼等在 1996 年提出细菌的此类表面蛋白是抗体的起源。一些病毒也有类似的蛋白,目前不知道是从细菌身上取来还是起初由病毒转给细菌的。\n- 海绵、海鞘等非脊椎动物有不同类型的·结构与免疫球蛋白相似的蛋白质。海鞘物种 Ciona intestinalis 等拥有带可变区的几丁质结合蛋白,这是一种免疫识别分子,结构和功能在一定程度上类似抗体,能粘附在细菌上并促进血细胞吞噬细菌 。\n- 文昌鱼、海胆、软体动物等有一些细胞能进行类似 VDJ 重组的体细胞突变。七鳃鳗和多种鱼类有一些细胞能进行体细胞超突变。这些是 B 淋巴细胞产生各种各样的抗体的基础。", "task_type": {"major": ["问答"], "minor": ["知乎问答"]}, "domain": ["通用"], "metadata": "qid:561794736, aid:2727989918", "answer_from": "human", "human_verified": true, "copyright": "暂无版权及作者信息"} {"instruction": "人类的神经细胞为什么无法再生?", "input": "", "output": "事实上,人的神经细胞可以再生,死亡的神经细胞可以被新的神经细胞取代,尤其是在海马体;即使不产生新的神经细胞,已经存在的神经细胞也可以改变功能来代偿受损或死亡的细胞的功能,半脑切除之类手术和一些严重脑水肿案例显示人脑可代偿看起来不可思议的神经缺失。\n\n这和所谓意识的连续性完全无关。新产生的神经细胞可以接替旧细胞形成同样模式的突触连接来储存一样的信息、执行一样的功能。况且,你不觉得在“随着年龄增长,死掉了大量神经细胞、没有新细胞补充”的情况下“维持主观连续的意识”难度更高吗?\n\n包括人在内,成年哺乳动物脑仍有两个区域包含神经干细胞:侧脑室的脑室下区(SVZ)、海马体齿状回,这些神经干细胞在许多动物身上制造新的神经元。室管膜细胞和放射状胶质细胞可以分裂、可以分化为其他类型的细胞。人的侧脑室壁有神经胶质细胞迁移形成的带状结构,包含新产生的神经元、星形胶质细胞、少突胶质细胞,但其流量在年满18 个月后明显下降。\n\n- 1998 年,癌症患者遗体展示了成年人中枢神经再生的首个证据。他们在世时接受过溴脱氧尿苷注射,该物质能标记新分化的细胞。科学家发现该物质出现在这些患者大脑内的海马体(和记忆、学习相关的海马状结构)中。\n- 2013 年,来自瑞典卡罗林斯卡研究所的 Jonas Frisén 团队对 55 名已去世患者大脑组织中的单个神经元进行了碳年代测定,根据细胞年龄推测,人类每天在编码记忆的海马体齿状回区更新 700 个神经细胞。\n- 后来有其他研究认为这再生速度在人幼年时较快、30 多岁时基本陷于停滞。其他脑区的神经细胞补充速度比这要慢或更早停滞。\n\n在人体发生脑损伤(例如中风)后,病灶周围的神经元会上调促进轴突生长和突触形成的信号通路(Dancause 和 Nudo,2011年),树突棘的周转增加,以便形成新的连接。病灶周围组织中的神经元可以将新的轴突延伸数毫米,在附近的皮质区域形成新的功能性突触连接(Nagappan等,2020 年)。\n\n人外周神经系统的轴突断裂后可以用每天 1 到 2 毫米的速度缓慢地生长恢复。断肢再植或移植四肢时,患者可以体验到触觉一点点朝肢体末端延伸。\n\n- 在人身上进行的完整双臂移植手术于在 2008 年在德国农民 Karl Merk 身上首次取得成功。切断的轴突每天再生约 1 毫米。\n- 新神经细胞延伸出轴突的速度在大神经中每天约 5 毫米,在小神经中每天约 2 毫米。\n\n实验显示,半年间每天进行步行、骑自行车、爬楼梯等有氧运动,可以提高各年龄段的人的思维能力,观察到左侧额叶区域的大脑皮层厚度增加。这主要来自突触数量上升而非更多的神经细胞生成。\n\n胎儿细胞可以进入母亲的血液循环并在心脏、肝脏、甲状腺等处分化成新细胞。对人类来说,这些细胞可以在循环系统里维持约 27年。根据在老鼠身上进行的实验,胎儿细胞能在母亲大脑受损时大量透过血脑屏障去进行修理。不过,在人身上也有胎儿细胞引起的自身免疫性疾病。\n\n无论如何,在大脑发达的人类身上,上述自然修复速率无法和脑神经细胞的总数相比,脑细胞大量死亡的时候不能靠这种机制恢复。现在也还缺少注射神经干细胞或拟似胎儿细胞并主动促使其大规模修复人类大脑的技术,少量干细胞主要产生神经胶质细胞,将其诱导为神经元的规模也很有限。脑部或脊髓损伤通常会导致神经胶质细胞形成瘢痕组织。\n\n- 胶质瘢痕由密集的神经胶质细胞组成,缺乏功能性神经元,长期被视为神经再生的障碍,但其分泌的生长因子又是神经再生所需要的,因此处理起来十分麻烦。目前研究的方向是利用病毒载体调高神经胶质细胞中的神经转录因子水平,诱导其转化为功能性神经元。\n\n已经有对神经干细胞进行基因改造来在大鼠身上提高效率地修复脑梗死等的成功实验。\n\n此外,2020 年 6 月 25 日,《自然》杂志封面文章称,研究人员在体外的人类细胞和小鼠中找到了只需一个步骤就将星形胶质细胞转化为神经元的方法:星形胶质细胞产生的 RNA 结合蛋白 PTBP1(多聚嘧啶道结合蛋白)阻止了这种细胞变成神经元,去除 PTBP1 就可以解决该问题。\n\n- 研究人员首先从小鼠和人类脑中分离出星形胶质细胞,降低这些细胞中 PTBP1 基因的表达水平。经过四个星期,不同样本中 50%~80% 的星形胶质细胞转化为神经元,来自中脑的星形胶质细胞所转化的神经元表达了多巴胺神经元特异性表达的基因。\n- 然后,研究人员为小鼠注射可降低 PTBP1 基因表达的病毒。经过 12 个星期,感染病毒的星形胶质细胞有超过 30% 转化为多巴胺神经元、具备正常生理功能,将患有帕金森病的小鼠纹状体的多巴胺含量从(正常值的)25% 提高到 65%,改善了症状。\n- 最后,研究人员尝试向小鼠中脑注射可干扰 PTBP1 的反义寡核苷酸,短暂抑制其水平,发现也有治疗效果。这比病毒更适合临床使用,不过该疗法在用于人类之前仍需开展大量研究。", "task_type": {"major": ["问答"], "minor": ["知乎问答"]}, "domain": ["通用"], "metadata": "qid:560846671, aid:2729424583", "answer_from": "human", "human_verified": true, "copyright": "暂无版权及作者信息"} {"instruction": "海豚、淡水豚会被鱼刺卡住吗?", "input": "", "output": "会。从 1997 年至 2011 年,科学家记录到美国佛罗里达州印第安河泻湖沿岸搁浅的 350 条宽吻海豚里有 14 条(12 条成年、2条幼年)是被鱼卡住喉咙而死。\n\n- 相关鱼主要是背鳍中有硬刺的罗非鱼、羊头鲷、Diapterus plumieri(条纹莫哈拉鱼),其中 5 条带有鱼钩、鱼线之类让海豚更难处理的人类产品。\n- 海豚吃大型猎物时喉部脱位到食道,被猎物身上出乎意料的结构卡住的概率上升。淡水豚有基本上同样的结构,可以预期有类似海豚的噎死概率。\n- 历史上,海豚一般会避开羊头鲷这样危险的猎物,但近年来的环境变化可能让海豚缺少传统猎物。\n\n若题目想问的是鱼的肌间刺,对海豚来说那不是问题。生鱼的肌间刺没有人熟悉的熟鱼身上那么硬,海豚也不怎么将鱼扯成露出肌间刺的碎块。", "task_type": {"major": ["问答"], "minor": ["知乎问答"]}, "domain": ["通用"], "metadata": "qid:561624687, aid:2732311357", "answer_from": "human", "human_verified": true, "copyright": "暂无版权及作者信息"} {"instruction": "哪些蜘蛛可以飞行?", "input": "", "output": "蜘蛛靠丝线飞起来的方法称为“飞航”,一般不叫飞行。已记录飞航行为的蜘蛛物种分布在 27个科,常见的飞航蜘蛛来自皿蛛科、蟹蛛科、园蛛科、猫蛛科、逍遥蛛科。漏斗蛛科、近管蛛科、地蛛科、管巢蛛科、圆颚蛛科、螲蟷蛛科、卷叶蛛科、石蛛科、隆头蛛科、平腹蛛科、栅蛛科、光盔蛛科、狼蛛科、拟态蛛科、米图蛛科、蜜蛛科、类球蛛科、盗蛛科、跳蛛科、肖蛸蛛科、球蛛科、妩蛛科亦有飞航的物种。\n\n一些科学家实验研究过皿蛛科的 Erigone atra、园蛛科艾蛛属的 Cyclosa turbinata、球蛛科希蛛属的 Achaearaneatepidariorum、隆头蛛科 Stegodyphus 属的 S. dumicola 和 S.mimosarum(大型蜘蛛,放出多条丝线)、狼蛛科豹蛛属的 Pardosa amentata、P. lugubris、P. monticola、P.nigriceps、P. palustris、P. prativaga、P. pullata、P. purbeckensis、P. ramulosa、P.tuoba 等物种的飞航行为。\n\n- 蜘蛛飞航行为受温度、湿度、风速、上升气流、光照强度、食物供应等影响,不同季节有不同物种的蜘蛛在飞,大部分飞航个体的体型较小(可以是幼体也可以是体型小的成体)。\n- 飞航过程中,调整蛛丝长度和姿态能在一定程度上影响飞航距离和降落点,必要时可以切离丝线并降落。除风力和上升气流外,飞航可能利用了大气电场。\n- 新蛛亚目蜘蛛飞航前通常会爬到高处,然后释放多条细丝或用丝线垂挂一定距离来获得初速度再释放多条细丝并切断垂挂用的丝线。某些猛蛛亚目蜘蛛从高处跳下、在空中放出多条细丝。\n- 许多蜘蛛一次只飞航数米远,可以连续飞航。天气条件适宜时,一些蜘蛛能在 6 小时内飞航 30 千米,高度可达 5 千米以上(Foelix,2011 年。飞航蜘蛛偶尔会飞到平流层),低空飞行时可无视 0.5 米高的海浪等障碍。某些连续飞航的蜘蛛能在水平方向上移动 3200 千米以上(Gressitt, 1965 年)。\n\n此外,某些叶螨、至少 31 种鳞翅目昆虫的幼虫也可以飞航。", "task_type": {"major": ["问答"], "minor": ["知乎问答"]}, "domain": ["通用"], "metadata": "qid:561364476, aid:2727585344", "answer_from": "human", "human_verified": true, "copyright": "暂无版权及作者信息"} {"instruction": "在自然界的演变中,RNA为什么没有被单链DNA所取代?大部分生物的遗传物质是DNA,显然DNA较RNA有更高的稳定性,那么为什么RNA仍然在进化中保留下来?RNA的部分功能是否可由单链DNA来执行?", "input": "", "output": "DNA 和蛋白质不能执行 RNA 的某些催化功能,例如在核糖体里发挥催化作用的是且只是 RNA,过去 38 亿到 45亿年的演化都没有改变这状况。剪接体也如此。\n\n不过,题目错的地方比这更基本:\n\n在热力学稳定性上,双链 RNA 胜过双链 DNA,双链 DNA 胜过 DNA-RNA 杂交链。\n\n- 人们其实直到公元 2013 年才能实验测定生物体内 RNA 的结构。真核生物体内大量的小 RNA 是双链的,有很多非编码 RNA 是通过碱基互补跟 mRNA 配对形成双链再去发挥功能的,mRNA 也可能形成双链。\n- “RNA 病毒突变快”是因为不进行纠错,碱基错配概率高,而不是“RNA 链不稳定”。\n\n日常所说的“RNA 不稳定”,部分原因是单链 RNA 在高 pH 下 2' 羟基会丢失质子、容易水解,主要原因是地球生物大量使用 RNA 酶,单链 RNA容易被切断。这允许细胞生物降解现有的 mRNA 来调整合成的蛋白质的类型、数量、比例,处理掉带有严重错误的 mRNA。\n\n对储存遗传信息来说,稳定性没有那么重要。\n\n- DNA-RNA 杂交链能维持生物的正常功能。现实中人类自己制造过使用 DNA-RNA 杂交链的大肠杆菌、酵母菌等生物。2018 年 Angad P. Metha 等构建并表征了一种含有 DNA-RNA 杂交链的大肠杆菌菌株,其基因组含有约 40% 到 50% 的核糖核酸。\n- 即使考虑单链,人类也早就知道 L-RNA 比自然界的生物使用的 D-RNA 对 RNA 酶稳定得多。\n- tRNA 以甲基化修饰和假尿嘧啶、次黄嘌呤核酸等特殊碱基提高稳定性,人们亦可这样修饰 mRNA 来在体外保存并使用。Moderna 和 BioNTech 的 mRNA 疫苗引入假尿嘧啶来提高 mRNA 的稳定性。\n\n热力学稳定性不是用什么做遗传物质的重点。\n\n双链 RNA 的性质较接近 A-DNA,不如 B-DNA 适合与蛋白质进行序列特异性的相互作用。\n\nRNA 不但可以在局部折叠成双链结构,还可以用相距甚远的两段序列互补配对形成双链,比 DNA更容易出现四链结构,可以和多种金属离子、辅助因子互动。在需要执行催化功能而不止储存一些信息时,RNA 胜过 DNA。\n\n- 在自然界,人们主要观察到 RNA 催化两大类反应:磷酸转移,肽键形成。没有这些就没有细胞生物。\n- RNA 使用的催化策略相当广泛,可以拿去催化其他反应。RNA 上的一些位点可以结合水合金属离子或蛋白质作为酶时使用的几种辅因子,可以催化光合作用、丙酮酸氧化、固氮、酶促氧化还原反应、碳-碳键的形成与切割(磷酸戊糖途径、卡尔文循环、三羧酸途径)、糖苷键形成、厌氧发酵、乙醇脱氢、自由基生成、异构化、烷基化、Diels-Alder 环加成、金属转移等。\n- 尽管 RNA 的官能团有限,在原则上,你可以用 RNA 催化维持生命所必需的亿万种化学反应。\n- 已经从自然界的细菌中分离出与 NAD+连接的 RNA。在实验室,已经造出使用包括 NAD 和 CoA 在内的辅因子的核酶 。\n\n“适者生存所以能”不怎么适合当做答案。“适者”体现在“经历自然选择并生存下来”,不解释“为什么能生存下来”。\n\n无论如何,你看到许多现存的无危物种携带某个长期流传的遗传特征,就可以预期“那个遗传特征并不明显减少可育后代的数量”。", "task_type": {"major": ["问答"], "minor": ["知乎问答"]}, "domain": ["通用"], "metadata": "qid:557690605, aid:2711096495", "answer_from": "human", "human_verified": true, "copyright": "暂无版权及作者信息"} {"instruction": "为什么胸腺嘧啶一般只能在DNA中?尿嘧啶一般只能在RNA中?", "input": "", "output": "这取决于核酸的功能。\n\nDNA 使用的胸腺嘧啶 T 是甲基化修饰的尿嘧啶 U,不易被核酸酶降解,不会在 DNA 修复机制纠正胞嘧啶 C 突变为尿嘧啶 U的偶发状况时被错误修理,有利于稳定记录遗传信息。\n\n- 尿嘧啶 U 可以摇摆配对腺嘌呤 A 和鸟嘌呤 G,在多次复制中容易突变。\n- 胞嘧啶 C 容易自发脱去环外氨基、变为尿嘧啶 U,改变核酸存储的信息并降低下一次复制后的核酸分子的稳定性(CG 之间有三个氢键,而 AU 之间有两个氢键),造成各种错误。\n- DNA 要长期使用并传给后代,没有特殊目的的话,应当处理掉序列里出现的 U。\n\n大部分 RNA 没有 DNA 那么高的稳定性需求,不需要进行这种甲基化修饰。\n\ntRNA 使用大量经过甲基化修饰的碱基,包括胸腺嘧啶。这有利于维持 tRNA 的稳定、有助于和密码子配对、有助于氨基酸与受体臂连接。", "task_type": {"major": ["问答"], "minor": ["知乎问答"]}, "domain": ["通用"], "metadata": "qid:535943435, aid:2516264783", "answer_from": "human", "human_verified": true, "copyright": "暂无版权及作者信息"} {"instruction": "自然界中的生物,雄性参与养育后代的物种占比多少?", "input": "", "output": "你想问的大概局限于动物。动物中不到 1% 的物种会尝试照顾后代,其中雄性参与照顾后代的不到这 1% 的10%,能叫做“养育”大概起码提供了一点食物(而不止是“孵蛋、击退敌害”),其比例接近 0%~\n\n- “自然界中的生物”在个体数量和物种数量上绝大部分是病毒、细菌、古菌,动物在生物的个体数量和物种数量中占比接近 0%,是绝对的非主流。\n- 几乎没有能通过“额外排放一点有机物”之外的方法去照顾后代的线虫,而线虫在动物的数量里占比极高。照顾后代的节肢动物很少,而节肢动物在动物的物种数里占比很大。这意味着在动物内部,照顾后代的占比也会非常低。\n\n哺乳纲的所有物种都有雌性照顾后代的行为,约 5% 到 6% 的物种存在雄性照顾后代的行为,主要出现在啮齿目、灵长目、食肉目犬科。\n\n- 哺乳纲约 3% 的物种采用社会化一夫一妻制,这些物种里约 59% 存在雄性照顾后代的行为。\n- 灵长目约 15% 的物种采用社会化一夫一妻制。\n- 看起来,只在照顾后代产生的收益大于“多找几个雌性交配”的时候,雄性哺乳动物才会倾向于照顾后代。这受“是否确定后代是自己的”“自己的食物是否过剩”“自己的性能力如何”等多种因素影响。在人类中,睾丸较小的父亲往往会更多地参与照顾孩子(Jennifer S. Mascaro 等,2013 年)。\n\n鸟纲约 75% 到 81% 的物种有雌雄合作照顾后代的行为,但不是所有个体都会参与。鸟纲约 9% 的物种跨血缘关系合作照顾幼鸟。\n\n鸟纲约 1% 的物种完全由雄性照顾后代,约 8% 到 9% 的物种完全由雌性照顾后代。\n\n- 鸟纲超过 90% 的物种采用社会化一夫一妻制,其中约 81% 的物种存在父母合作照顾后代的行为。\n- 鸟纲约 9% 的物种使用一妻多夫制,雄性参与孵蛋。\n- 鸟纲不到 1% 的物种使用一夫多妻制,雄性提供大部分照顾。\n- 照顾后代可能是鸟的祖传代码,来自非鸟恐龙。\n\n30%的鱼纲科级分类存在照顾后代的行为,其中大半是雄性单独照顾后代,约五分之一是雌雄一起照顾后代。鱼的雄性照顾后代与体外受精明显正相关,大概是因为能确定起码有一部分是自己的后代。海马由雄性负责“怀孕”。\n\n约 6% 的青蛙、约 18% 的蝾螈、约 5% 的蚓螈会照顾后代(或至少看起来有这样做的尝试),其中雄性单独照顾和雌性单独照顾各一半左右,极少合作照顾。\n\n爬行类极少由雄性单独照顾后代。\n\n无脊椎动物极少照顾后代,雄性尤其少。它们提供的照顾以保护为主。", "task_type": {"major": ["问答"], "minor": ["知乎问答"]}, "domain": ["通用"], "metadata": "qid:512851109, aid:2321244229", "answer_from": "human", "human_verified": true, "copyright": "暂无版权及作者信息"} {"instruction": "是否发现某类基因决定了动物的社会性?\n比如某些动物有明显的社会关系,而另一些动物则偏向独居。", "input": "", "output": "成百上千个基因与社会性有关,未发现哪个是决定性的。\n\n- 精氨酸加压素受体、催产素受体等受体参与从田鼠到人类的许多物种的社会行为,这显然关系到编码它们的基因、参与调节基因表达水平的基因与非编码序列;\n- 既然是神经系统要参与的事,突触蛋白、离子通道蛋白、昼夜节律相关蛋白显然是绕不过去的;\n- 转录因子 egr1 (在不同物种中被称为 ngfi-a、 zif-268、krox-24、tis8 、 zenk 等)参与鸟对声音的反馈,前脑的一些细胞在听到鸟叫声等声音后调高编码该转录因子的基因的表达水平,调高情况在听到白噪音、陌生的鸟叫、熟悉的鸟叫时显著不同,在鸟周围有同类时更与单独行动时不同;编码 egr1 的基因还参与伯氏妊丽鱼对社会地位的识别,此鱼雄性个体会随社会地位变化改变自己的外观,地位越高越鲜艳,求偶的效果也越好,在群体内高地位的个体被人为移除后,其他雄性下丘脑中含有促性腺激素释放激素的神经元的 egr1 表达迅速上升,激素水平变化,进而改变体色;实验显示,egr1 表达可以在 20 到 60 分钟内出现巨大变化,可以立即增强或抑制其他基因的转录,在不同的细胞类型中可以和不同的蛋白质互动。这样的转录因子还有许多;\n- 实验显示,在蜜蜂的一生中,脑部数千个基因的表达水平有可测量的差异;蜜蜂感知信息素后会在几天到几周内改变脑中数百个基因的表达。\n- 大鼠照顾后代的行为可以在后代海马体引发糖皮质激素受体基因的启动子的表观遗传修饰,影响长期持续。\n\n诸如此类。\n\n注意基因不能直接指定动物的行为。细胞用基因序列和表观遗传修饰存储着构建身体并调控身体功能的各种分子的信息。脑发育、脑活动、动物行为是遗传·环境·生活经历·力学机制·随机扰动等共同创建的。社会信息会和其他内外信息一样影响一些脑细胞的基因表达与突触连接网络,进而影响生物体的一些行为。在完全不修改基因序列的情况下,胚胎细胞受力状况的变化可以彻底改变动物胚胎细胞发育成的东西的身体计划。\n\n此外,社会继承可以在很大程度上解释动物社会网络的结构。", "task_type": {"major": ["问答"], "minor": ["知乎问答"]}, "domain": ["通用"], "metadata": "qid:549951899, aid:2645507678", "answer_from": "human", "human_verified": true, "copyright": "暂无版权及作者信息"} {"instruction": "科学家认为,人类都是早产儿,那如果真的按照推测孕育21个月是不是会产生更出色的新人类?", "input": "", "output": "不是那样。\n\n“人类都是早产儿”是拿来解释“为什么人类新生儿的生存能力极度低下”的不严谨说法,历史上曾经由阿道夫·波特曼整合到 Sherwood LernedWashburn 于 1960 年提出的“产科困境”假说中,试图将人类女性那异常困难的分娩说成“演化产生的解决方案”。“21个月”是历史上按人大脑发育速度估算的“人新生儿的大脑发育到体积占成年大脑 40%的程度(和黑猩猩新生儿一样)”需要的时间。这是基于过时知识计算的,按新一些的数据,需要的时间更接近 16 个月(DeSilva 和 Lesnik, 2006年)——你有了这程度的知识,就足以意识到,无法指望这产生什么“更出色的新人类”。\n\n古尔德过去附和过“产科困境”,将其推得很广。这是个看起来很好懂的模因,被各路媒体和他们的受众到处传播。但这不是事实。\n\n- 类人猿的足月妊娠所需时间是灵长类里最长的,黑猩猩、倭黑猩猩的正常妊娠期约 39 周,人的正常妊娠期约 37 到 41 周。黑猩猩、倭黑猩猩的新生儿像人新生儿一样缺乏独立生存能力,需要母亲提供大量照顾。但这两种类人猿并没有人这么高的难产率,它们的产道比新生儿尺寸明显更宽。你觉得黑猩猩、倭黑猩猩的新生儿是早产儿吗?\n- 正常人类新生儿的大脑体积是成年时的 30% 左右,正常黑猩猩新生儿的大脑体积是成年时的 40% 左右,卷尾猴新生儿的大脑体积是成年时的 50% 左右,早产儿的标准要划定在什么地方?\n- 黑猩猩新生儿那 40% 并没有改变其缺乏独立生存能力的表现。\n- 人大脑生长速度的峰值出现在出生前的几个月间,与生下来就能行走的动物类似(Halley, 2017 年)。\n- 对双胞胎的研究显示,人类女性的骨盆宽度受生活环境的影响似乎超过基因造成的影响。\n- 将人类女性的骨盆再加宽几厘米以扩大产道,真的会和“产科困境”假说设想的那样妨碍双足行走吗?非洲南部石器时代晚期的人群似乎搞出过身高很低而骨盆很宽的女性,当时他们经常长距离迁徙,化石并未显示这些人在行走方面有什么困难;农业时代的饮食变化可能比骨盆宽度的遗传因素造成了更多的产科问题。生物力学也不支持“加宽骨盆会增加运动成本”的想法。\n- 约 12000 年的农业社会生活尚未让人的骨盆宽度重新优化到适合新生儿的大头。\n- 人类胎儿的能量需求和母体能安全地提供的代谢率与散热能力的关系似乎更直接地决定了胎儿能达到的体重。这方面的统计相关性也出现在没有骨头限制产道的鲸类身上(Sacher 和 Staffeldt,1974 年)。\n- 提出“产科困境”假说时使用的统计数据可能不反映历史上大部分时候的状况。\n\n人的胎盘在超过 42 周妊娠后会出现明显的功能劣化,血管变细、局部钙化、输送氧和营养物质的能力下降,对胎儿的生命安全构成威胁,胎儿死亡率上升到 40周妊娠的 3 到 6 倍。随着时间流逝,胎儿死亡或畸形的概率还会上升。以当前的人类基因,人的胎盘无法支持“孕育 21 个月”。\n\n人胎盘是怀孕 10 周后由胎儿的合体滋养细胞发育而成,不是子宫的一部分。人造子宫不解决胎盘的问题。\n\n另外,在人类子宫中,超过 42 周的过期妊娠会引起羊水量减少和一些胎儿尺寸过大,难产率上升。\n\n机械胎盘尚未研制出来,可以想象要移除胎儿的胎盘、插几根管子、泡点类似羊水的液体,这可能将胎儿搞死搞残。\n\n人造子宫尚未研制成功,哺乳类幼体很脆弱,目前对 120 天左右的早产儿使用的支持技术(恒温箱、ECMO之类)有概率导致早产儿发生脑出血等自灭。延长机械支持时间会增加出问题的概率。\n\n“在足月后继续将婴儿摆在恒温箱里”类似对重度先天性免疫缺陷患者的措施(无菌隔离),实践不支持那对人的身心发育有任何好处。", "task_type": {"major": ["问答"], "minor": ["知乎问答"]}, "domain": ["通用"], "metadata": "qid:545738250, aid:2599016608", "answer_from": "human", "human_verified": true, "copyright": "暂无版权及作者信息"} {"instruction": "假如从世界上第一个人诞生开始,一直往宇宙深处走,到了自然死亡就有下一个人接力。我们可以走多远?", "input": "", "output": "看起来,本题目只是想问全人类累积能走多远的路程,不考虑人体有没有能力走到天上去。那么,太阳系的尺寸比不过人类走过的路程。\n\n以尚未观测清楚的奥尔特云计算,太阳系的半径约 1 光年。\n\n1 光年是 9460730472580800 米,精确值。\n\n当前世界上有约 79 亿人,设其中 50 亿人在 1 年间每天拿出 1 小时用 1.5 米每秒的速度走路,那么一年的总路程是 1.04光年多一点。根本就不用出动“从世界上第一个人诞生开始”那么夸张的阵容。\n\n你可以看出,人们对规模巨大的东西是没有概念的,哪怕那个东西是人们自以为最熟悉的“人类”也一样。\n\n \n\n目前累计出生过的人数约 1170 亿。这之中有许多个体早死,已经死去的全人类的平均寿命约 35 年。\n\n- 旧石器时代人出生时的预期寿命约 22 到 33 年,新生个体活到 15 岁的概率约 60%,15 岁个体的预期总寿命约 54 年。\n- 由于早期农业很菜,新石器时代与青铜时代早中期不少地区的古人类预期寿命比旧石器时代还短数年。\n- 古代世界在这方面的改善程度不大,人出生时的预期寿命长期在 25 到 40 年震荡。\n- 从 1750 年到 2000 年,新生个体的预期寿命从约 33 年急剧增加到约 67 年。\n\n设 1170 亿人平均每人在 15 年间每天拿出 1 小时用 1.5 米每秒的速度走路,那么总路程将超过 365 光年。", "task_type": {"major": ["问答"], "minor": ["知乎问答"]}, "domain": ["通用"], "metadata": "qid:524724780, aid:2414813875", "answer_from": "human", "human_verified": true, "copyright": "暂无版权及作者信息"} {"instruction": "如何用定义证明1/x是双曲线?(F1-F2=2a)?", "input": "", "output": "将曲线 $$\\mathscr{H}:y=\\frac{1}{x}$$ 顺时针旋转 $$\\frac{\\pi}{4},$$由于旋转变换并不改变曲线本身形状,我们只需证明这旋转后的曲线 $$\\mathscr{H'}$$ 是双曲线即可。\n\n考虑极坐标化, $$\\mathscr{H}$$ 的极坐标方程为 $$r^2\\sin\\theta\\cos\\theta=1.$$ 假设$$(r,\\theta)$$ 是 $$\\mathscr{H'}$$ 上任意一点,则将其逆时针旋转 $$\\frac{\\pi}{4}$$ 还原后的位置$$\\left(r,\\theta+\\frac{\\pi}{4}\\right)$$ 必落于 $$\\mathscr{H}$$ 上,这坐标将满足$$\\mathscr{H'}$$ 方程,即有$$r^2\\sin\\left(\\theta+\\frac{\\pi}{4}\\right)\\cos\\left(\\theta+\\frac{\\pi}{4}\\right)=1,\\\\\\$$也就是 $$\\frac{1}{2}r^2\\cos^2\\theta-\\frac{1}{2}r^2\\sin^2\\theta=1,\\\\\\$$ 再化到直角坐标系,有$$\\frac{x^2}{2}-\\frac{y^2}{2}=1,\\\\\\$$ 这就得到了 $$\\mathscr{H'}$$ 的方程,显然表示着一条等轴双曲线。", "task_type": {"major": ["问答"], "minor": ["知乎问答"]}, "domain": ["通用"], "metadata": "qid:376164481, aid:2058925663", "answer_from": "human", "human_verified": true, "copyright": "暂无版权及作者信息"} {"instruction": "请问能否不使用微积分的知识证明伯努利不等式?", "input": "", "output": "首先指出,问题不严谨。不使用任何微积分知识,要证明实数次幂型的 $$\\text{Bernoulli}$$不等式是不可能的,因为事实上,连这个实数次幂本身都需要用微积分的实数理论去定义。于是,假定当前问题只是想要一种不涉及导数的证法,这当然是办得到的。\n\n现在的任务是要证明\n\n> 设 $$x>-1,$$ 则有 > $$(a)$$ 不等式 $$ (1+x)^r\\le 1+rx$$ 对所有 $$0 $$(b)$$ 不等式 $$ (1+x)^r\\ge 1+rx$$ 对所有 $$r>1$$ 成立; > $$(c)$$ 不等式 $$ (1+x)^r\\ge 1+rx$$ 对所有 $$r<0$$ 成立。\n\n## 一、先来证明 $$r$$ 是有理数时的情形\n\n$$\\color{blue}{\\bold{\\text{Case 1}}}$$ 当 $$00.$$ 同时,因$$x>-1,$$ 也有 $$1+x>0.$$ 于是依 $$\\text{AM-GM}$$ 不等式,成立\n\n$$\\begin{align*}(1+x)^r&=(1+x)^{\\frac{p}{q}}=\\sqrt[q]{\\underbrace{(1+x)\\cdots(1+x)}_{p~~\\text{terms}}\\cdot\\underbrace{1\\cdots\\cdot1}_{q-p~~\\text{terms}}}\\\\\\&\\le\\frac{p\\cdot(1+x)+1\\cdot(q-p)}{q}=1+\\frac{p}{q}\\cdot x=1+rx, \\end{align*}\\\\\\$$于是 $$(a)$$ 得证。\n\n$$\\color{blue}{\\bold{\\text{Case 2}}}$$ 当 $$r>1.$$\n\n此时又分两种情况:\n\n * 若 $$1+rx\\le0,$$ 则 $$(1+x)^{r}>0\\ge 1+rx;$$ * 若 $$1+rx>0,$$ 则 $$rx>-1,$$ 同时注意到 $$0<\\frac{1}{r}<1,$$ 于是直接依 $$(a)$$ 成立 $$(1+rx)^{\\frac{1}{r}}\\le 1+\\frac{1}{r}\\cdot rx=1+x,$$ 即 $$(1+x)^r\\ge1+rx.$$ \n\n综上, $$(b)$$ 得证。\n\n$$\\color{blue}{\\bold{\\text{Case 3}}}$$ 当 $$r<0.$$\n\n此时总能求得充分大的 $$n\\in \\mathbb{N_+}$$ 使得 $$0<-\\frac{r}{n}<1$$ 以及$$-1<\\frac{rx}{n}<1$$ 同时成立。\n\n于是依 $$(a)$$ 成立\n\n$$0<(1+x)^{-\\frac{r}{n}}\\le1-\\frac{r}{n}\\cdot x\\le\\left(1+\\frac{r}{n}\\cdotx\\right)^{-1},\\\\\\$$ 同时做 $$-n$$ 次幂,将有\n\n$$(1+x)^r\\ge \\left(1+\\frac{r}{n}\\cdot x\\right)^n\\ge1+n\\cdot \\frac{r}{n}\\cdotx=1+rx,\\\\\\$$ 于是 $$(c)$$ 得证。\n\n## 二、再来证明 $$r$$ 是无理数时的情形\n\n此时总能求得一个有理序列 $$\\\\{r_n\\\\}$$ 收敛于 $$r,$$ 比如可以将 $$r_n$$ 直接取为 $$r$$ 无尽小数表达式的 $$n$$位小数近似值。\n\n为了证 $$(a),$$ 利用前已证知的结论,并结合函数的连续性和序列极限的有关性质,将有\n\n$$(1+x)^r=\\lim_{n \\to \\infty}(1+x)^{r_n}\\le \\lim_{n \\to \\infty}(1+r_nx)=1+rx,\\\\\\$$ 从而 $$(a)$$ 得证。完全类似地,可证 $$(b),(c).$$\n\n至此,我们已避开导数完成了实数次幂型的 $$\\text{Bernoulli}$$ 不等式的全部证明。", "task_type": {"major": ["问答"], "minor": ["知乎问答"]}, "domain": ["通用"], "metadata": "qid:458277550, aid:1874108827", "answer_from": "human", "human_verified": true, "copyright": "暂无版权及作者信息"} {"instruction": "这道积分题目挺经典的,该怎么处理来着?\n设函数 $f$ 在 $[0,1]$ 上有二阶连续导数, 且 $f(0)=f(1)=f^{\\prime}(0)=0, f^{\\prime}(1)=1$. 求证:\n$$\n\\int_0^1\\left(f^{\\prime \\prime}(x)\\right)^2 d x \\geqslant 4\n$$\n先取特殊函数x^3-x^2,二阶导平方后积分值为4,后面怎么说明 任意函数都比它大来着?", "input": "", "output": "这里给出一种低门槛、容易理解的做法,更本质的做法需要利用变分法中的 $$\\text{Euler-Lagrange}$$ 方程,在此不做讨论。\n\n很显然,对所有的 $$a,b\\in\\mathbb{R}$$ 都有\n\n$$\\int_{0}^1 (f''(x)+ax+b)^2{\\rm d}x\\ge0.$$\n\n将左端展开,就有\n\n$$\\begin{align*} &\\int_{0}^1 (f''(x)+ax+b)^2{\\rm d}x\\\\\\ =&\\int_{0}^1a^2x^2+2abx+b^2{\\rm d}x+\\int_0^1(2ax+2b)f''(x){\\rm d}x+\\int_0^1f''(x)^2{\\rmd}x,\\\\\\ \\end{align*}$$\n\n其中\n\n$$\\int_{0}^1 a^2x^2+2abx+b^2{\\rm d}x=\\frac{a^2}{3}+ab+b^2,$$\n\n$$\\int_0^1(2ax+2b)f''(x){\\rm d}x=\\int_0^1(2ax+2b){\\rm d}f'(x)=2a+2b,$$\n\n所以\n\n$$\\int_0^1f''(x)^2{\\rm d}x\\ge -\\frac{a^2+(3b+6)a+3b^2+6b}{3},$$\n\n这式子要对一切 $$a,b$$ 成立,必须\n\n$$\\int_0^1f''(x)^2{\\rm d}x\\ge \\max_{a,b\\in \\mathbb{R}}\\left(-\\frac{a^2+(3b+6)a+3b^2+6b}{3}\\right)=4,$$\n\n这最值在 $$a=-6,b=2$$ 时取得,由此一举得到待证不等式的取等条件 f''(x)-6x+2=0,\" eeimg=\"1\"> 结合题设初值条件,即$$f(x)=x^3-x^2.$$", "task_type": {"major": ["问答"], "minor": ["知乎问答"]}, "domain": ["通用"], "metadata": "qid:442878394, aid:1715483130", "answer_from": "human", "human_verified": true, "copyright": "暂无版权及作者信息"} {"instruction": "是否存在在R上严格递增且处处不连续的函数?", "input": "", "output": "今将证明\n\n> $$\\color{blue}{\\text{Theorem}}$$ 单调函数的间断点至多可列。\n\n设 $$f(x)$$ 在 $$(a,b)$$ 上单调递增,如果它仅有有限多个间断点,结论是平凡的,我们只需研究这间断点无限多的场合。\n\n首先,我们熟知所有这些间断都是第一类间断,也即是说 $$f(x)$$ 在每个间断点处的左、右极限均存在,只不过它们并不都等于该点的函数值。\n\n假如 $$c$$ 就是这样的一个间断点,于是有 $$f(c-)< f(c+).$$ 为了叙述方便,我们把形如 $$(f(c-),f(c+))$$的开区间称为 $$f(x)$$ 在 $$x=c$$ 处的跃区,简记为 $$L(c).$$\n\n显然地, $$f(x)$$ 在每个间断点处都存在一个跃区,并且可以证明这些跃区必不相交,也即是说\n\n> $$\\color{blue}{\\text{Prop.}}$$ 对于间断点 $$c'>c,$$ 必有 $$L(c)\\cap> L(c')=\\varnothing.$$\n\n这证明是容易的。考虑在 $$c,c'$$ 之间插入 $$d,d'$$ 使得 $$c 全体间断点与全体跃区形成双射,全体跃区又与某有理数集形成双射,于是全体间断点与这有理数集形成双射。由于任何的有理数集可列,于是全体间断点也可列。\n\n现在,可以回答当前问题如下\n\n> 因为单调函数的间断点至多可列,换言之,单调函数几乎处处连续,于是不存在处处不连续的单调函数。\n\n最后岔开多说几句。 依这里证得的$$\\text{Theorem},$$ 单调函数的间断点构成一个零测集,于是再依 $$\\text{Lebesgue}$$定理,即可直接断定\n\n> 定义在闭区间上的单调函数可积。", "task_type": {"major": ["问答"], "minor": ["知乎问答"]}, "domain": ["通用"], "metadata": "qid:496499789, aid:2235483577", "answer_from": "human", "human_verified": true, "copyright": "暂无版权及作者信息"} {"instruction": "e^(t^2) 的原函数怎么求?", "input": "", "output": "这个事情要先从 $$\\int e^{-x^2}\\,{\\rm d}x$$ 说起。事实上 $$\\int e^{-x^2}\\,{\\rm d}x$$也是积不出来的,于是数学上直接定义了一个与之有关的高等函数,称为高斯误差函数(Gauss error function):\n\n$${\\rm erf}(x):=\\frac{2}{\\sqrt{\\pi}}\\int_0^x e^{-t^2}{\\rm d}t\\\\\\$$\n\n其中之所以带上了 $$\\frac{2}{\\sqrt{\\pi}}$$ 的系数,是因为这个函数在引入当时是作概率论研究用的。很清楚, $${\\rmerf}'(x)=\\frac{2}{\\sqrt{\\pi}}e^{-x^2},$$ 于是很容易就可推得\n\n$$\\int e^{-x^2}{\\rm d}x=\\frac{\\sqrt\\pi}{2}{\\rm erf}(x)+C\\\\\\$$\n\n若将其中的 $$x$$ 换为 $$ix,$$ 这里 $$i$$ 是虚数单位(imaginary unit),则还将有\n\n$$i\\int e^{x^2}{\\rm d}x=\\frac{\\sqrt\\pi}{2}{\\rm erf}(ix)+C\\\\\\$$\n\n于是\n\n$$\\int e^{x^2}{\\rm d}x=-\\frac{i\\sqrt\\pi}{2}{\\rm erf}(ix)+C\\\\\\$$\n\n数学家又索性再定义一个虚误差函数(imaginary error function):\n\n$${\\rm erfi}(x):=-i{\\rm erf}(ix)\\\\\\$$\n\n这样的话,上式又可以表为\n\n$$\\int e^{x^2}{\\rm d}x=\\frac{\\sqrt\\pi}{2}{\\rm erfi}(x)+C\\\\\\$$\n\n这就将当前的不定积分表成了一个高等函数的形式。", "task_type": {"major": ["问答"], "minor": ["知乎问答"]}, "domain": ["通用"], "metadata": "qid:362258957, aid:945705636", "answer_from": "human", "human_verified": true, "copyright": "暂无版权及作者信息"} {"instruction": "二阶导数的符号问题?\n为什么不记做 $\frac{d^2 y}{d^2 x}$ ,而是记作 $\frac{d^2 y}{d x^2}$ ,不会让人误解 $d x^2=2 d x$ 吗? 这么做的好处在哪?", "input": "", "output": "二阶导数之所以记为 $$\\frac{{\\rm d}^2 y}{{\\rm d}x^2}$$ 是有历史渊源的,它来源于二阶差商的概念。\n\n## 一、差分\n\n差商又来源于差分。这里先从差分说起。\n\n对于函数 $$y=f(x)$$ ,赋予自变量 $$x$$ 一个固定的增量 $$\\Delta x$$ ,就定义\n\n$$\\Delta y=f(x+\\Delta x)-f(x)\\\\\\$$\n\n为函数 $$y=f(x)$$ 的一阶差分。那么,将一阶差分再作差分,就得到二阶差分,即\n\n$$\\Delta(\\Delta y)=\\Delta (f(x+\\Delta x)-f(x))\\\\\\$$\n\n类似地,还可以定义三阶、四阶……直至 $$n$$ 阶差分。随着阶数的增加,左端的记号会变成 $$\\Delta(\\Delta(\\Deltay)),\\Delta(\\Delta(\\Delta(\\Delta y))),\\cdots$$ 这过于复杂,于是数学上将它简记为 $$\\Delta^n y,$$这里 $$n$$虽然写在右上角,但并不是乘方幂次,仅仅表示差分阶数。因此, $$\\Delta^2 y$$ 就是二阶差分,$$\\Delta^3y$$就是三阶差分,以此类推。\n\n## 二、差商\n\n有了差分的概念,我们就可以定义差商:\n\n一阶差商 $$\\frac{\\Delta y}{\\Delta x},$$ 二阶差商 $$\\frac{\\Delta^2 y}{(\\Delta x)^2},$$…… ,$$n$$ 阶差商就是 $$\\frac{\\Delta^n y}{(\\Delta x)^n},$$\n\n这里,分母上的 $$n$$ 确实表示 $$\\Delta x$$ 的幂次,括号即使去掉也不会产生误解,只要我们牢记这是 $$\\Delta x$$ 的乘方。\n\n## 三、作为差商极限的导数\n\n现在,我们来研究差商和导数的关系。事实上,可以证明:\n\n> 函数的$$n$$ 阶导数(如果存在)就是 $$n$$ 阶差商在 $$\\Delta x \\to 0$$ 时的极限。\n\n很清楚,对于一阶导数来说,这是成立的,完全就是定义。那么,对于二阶导数又如何呢?这里我们考虑先将 $$\\Delta^2y$$ 作一下变形,容易得到\n\n$$\\begin{align*} \\Delta^2y&=\\Delta (f(x+\\Delta x)-f(x))\\\\\\ &=f(x+\\Deltax+\\Delta x)-f(x+\\Delta x)-(f(x+\\Delta x)-f(x))\\\\\\ &=f(x+2\\Delta x)-2f(x+\\Deltax)+f(x).\\\\\\ \\end{align*}$$\n\n于是二阶差商的极限,就是\n\n$$\\begin{align*} \\lim_{\\Delta x \\to 0}\\frac{\\Delta^2 y}{\\Deltax^2}=\\lim_{\\Delta x \\to 0}\\frac{f(x+2\\Delta x)-2f(x+\\Delta x)+f(x)}{(\\Deltax)^2}. \\end{align*}\\\\\\$$\n\n这个极限怎么求呢?注意,二阶导数是存在的,容易看出这是一个 $$0/0$$ 型极限,于是可以考虑利用洛必达法则,就有\n\n$$\\begin{align*} \\lim_{\\Delta x \\to 0}\\frac{\\Delta^2 y}{\\Deltax^2}&=\\lim_{\\Delta x \\to 0}\\frac{f(x+2\\Delta x)-2f(x+\\Delta x)+f(x)}{(\\Deltax)^2}\\\\\\ &=\\lim_{\\Delta x \\to 0}\\frac{2f'(x+2\\Delta x)-2f'(x+\\Deltax)}{2\\Delta x}\\\\\\ &=\\lim_{\\Delta x \\to 0}\\frac{f'(x+2\\Delta x)-f'(x+\\Deltax)}{\\Delta x}.\\end{align*}\\\\\\$$\n\n根据二阶导数的定义,这末式显然就是 f''(x)\" eeimg=\"1\"> 。于是,我们证得了\n\n$$\\lim_{\\Delta x \\to 0}\\frac{\\Delta^2 y}{\\Delta x^2}=f''(x)=\\frac{{\\rmd}^2y}{{\\rm d}x^2}.\\\\\\$$\n\n仔细观察等式左右两端,右端不过是把左端的极限号去掉,然后把希腊字母 $$\\Delta $$ 改成了拉丁字母 $${\\rm d}.$$ \n\n到这里,我们已经豁然开朗了,为什么二阶导数的记号要写成这个形式?原来分子上的 $$2$$ 代表阶数,分母上的 $$2 $$代表平方。这么说来,二阶导数这个记号确实可以看做是一个比值的极限,即商的极限。如此,我们就大概知道导数又称微商的缘由了。\n\n## 四、一个注记\n\n最后,需要指出,尽管我们在二阶导数存在的前提下证明了,二阶导数就是二阶差商的极限,但却不能直接将二阶导数定义为二阶差商的极限。因为存在着这样一种情况,虽然函数二阶差商极限存在,但函数却不是二阶可导的,读者可以自行寻找这样的反例。", "task_type": {"major": ["问答"], "minor": ["知乎问答"]}, "domain": ["通用"], "metadata": "qid:30140042, aid:872001387", "answer_from": "human", "human_verified": true, "copyright": "暂无版权及作者信息"} {"instruction": "如何证明:两个数之间必有无穷多个无理数?", "input": "", "output": "首先可以证明\n\n> 对任意实数 $$a,b(am,$$ 矛盾。", "task_type": {"major": ["问答"], "minor": ["知乎问答"]}, "domain": ["通用"], "metadata": "qid:608863165, aid:3097483831", "answer_from": "human", "human_verified": true, "copyright": "暂无版权及作者信息"} {"instruction": "如何证明多项式 f(x)=1+x+x²/2!+x³/3!+…+x^n/n! 只有一个实数根?(n=2019)", "input": "", "output": "今将证明:\n\n> 方程 $$1+x+\\frac{x^2}{2!}+\\cdots+\\frac{x^n}{n!}=0(n \\in \\mathbb{N^+})$$ 至多有> $$1$$ 个实根。\n\n为此,置 $$f_n(x):=1+x+\\frac{x^2}{2!}+\\cdots+\\frac{x^n}{n!},$$ 再置$$F_n(x):=e^{-x}f_n(x).$$ 由于 $$(\\forall x)e^{-x}>0,$$ 所以方程 $$f_n(x)=0$$ 与$$F_n(x)=0$$ 同解,这就只需要研究后者。注意到F_n'(x)=e^{-x}[f_n'(x)-f_n(x)]=-e^{-x}\\frac{x^n}{n!}, \\end{align*}\" eeimg=\"1\">以下依 $$n$$ 的奇偶性分别讨论之:\n\n * 当 $$n$$ 为偶数时,除$$F_n'(0)=0$$外, $$(\\forall x \\neq 0)F_n'(x)<0,$$ 这表明 $$F_n(x)$$ 严格递减;同时,不难求得 $$\\lim_{x \\to +\\infty} F_n(x)=0.$$ 由此, $$(\\forall x)F_n(x)>0,$$ $$F_n(x)=0$$ 无实根。 * 当 $$n$$ 为奇数时,除$$F_n'(0)=0$$外, $$(\\forall x<0)F_n'(x)>0,$$ $$(\\forall x>0)F_n'(x)<0.$$ 这表明 $$F_n(x)$$ 在 $$(-\\infty,0)$$ 上严格递增,在 $$(0,+\\infty)$$ 上严格递减。此外,注意到 $$\\lim_{x \\to -\\infty} F_n(x)=-\\infty,$$ $$F_n(0)=1,$$ 以及 $$(\\forall x>0)F_n(x)>0,$$ 所以 $$F_n(x)=0$$ 在 $$(-\\infty,0)$$ 上有且仅有 $$1$$ 个实根。\n\n综上两方面,即证。", "task_type": {"major": ["问答"], "minor": ["知乎问答"]}, "domain": ["通用"], "metadata": "qid:419149559, aid:1456972443", "answer_from": "human", "human_verified": true, "copyright": "暂无版权及作者信息"} {"instruction": "请问如何证明下面这个双中值等式?\n设函数 $f(x)$ 在 $[a, b]$ 连续, 在 $(a, b)$ 可导, 求证在 $(a, b)$ 内存在相异两点 $\\xi$ 和 $\\eta$ 使得 $f^{\\prime}(\\xi) f^{\\prime}(\\eta)=\\left[\\frac{f(b)-f(a)}{b-a}\\right]^2 $", "input": "", "output": "## $$\\bold{\\text{Proof 1}}$$\n\n置 $$F(x):=[f(x)-f(a)]^2-\\left[\\frac{f(b)-f(a)}{b-a}\\right]^2(x-a)^2,\\\\\\$$ 则有$$F(a)=F(b)=0.$$ 于是依 $$\\text{Lagrange}$$ 中值定理,存在 $$\\eta \\in (a,b)$$ 使得$$F'(\\eta)=0,$$ 也即 $$\\frac{f(\\xi)-f(a)}{\\xi-a}\\cdotf'(\\xi)=\\left[\\frac{f(b)-f(a)}{b-a}\\right]^2,\\\\\\$$ 再依 $$\\text{Lagrange}$$中值定理,存在 $$\\eta \\in(a,\\xi)$$ 使得 $$\\frac{f(\\xi)-f(a)}{\\xi-a}=f'(\\eta),$$ 于是 $$f'(\\xi)f'(\\eta)=\\left[\\frac{f(b)-f(a)}{b-a}\\right]^2.\\\\\\$$\n\n* * *\n\n$$\\bold{\\text{Proof 2}}$$\n\n置 $$\\varphi(x):=[f(x)-f(a)]^2,~~\\psi(x):=(x-a)^2,\\\\\\$$ 则依 $$\\text{Cauchy}$$中值定理,存在 $$\\xi \\in(a,b)$$ 使得 $$\\begin{align*}\\frac{\\varphi(b)-\\varphi(a)}{\\psi(b)-\\psi(a)}=\\frac{\\varphi'(\\xi)}{\\psi'(\\xi)},\\end{align*}\\\\\\$$ 也就是$$\\left[\\frac{f(b)-f(a)}{b-a}\\right]^2=\\frac{f(\\xi)-f(a)}{\\xi-a}\\cdotf'(\\xi)\\\\\\$$ 再依 $$\\text{Lagrange}$$ 中值定理,存在 $$\\eta\\in(a,\\xi)$$ 使得$$\\frac{f(\\xi)-f(a)}{\\xi-a}=f'(\\eta),$$ 代入即得结论。\n\n事实上,可以证明:\n\n> $$\\bold{\\text{Corollary}}$$ 若 $$f(x)$$ 在 $$[a,b]$$ 上连续,在 $$(a,b)$$> 内可导,则必定存在互不相同的 $$n$$个点$$\\xi_1,\\xi_2,\\cdots,\\xi_n\\in(a,b)$$ 使得> $${f'(\\xi_1)f'(\\xi_2)\\cdots> f'(\\xi_n)=\\left[\\frac{f(b)-f(a)}{b-a}\\right]^n.}\\\\\\$$\n\n这只需要置 $$\\varphi(x):=[f(x)-f(a)]^n,~~\\psi(x):=(x-a)^n,\\\\\\$$ 进行与前类似的推导再利用归纳法即证。", "task_type": {"major": ["问答"], "minor": ["知乎问答"]}, "domain": ["通用"], "metadata": "qid:400768411, aid:1277727701", "answer_from": "human", "human_verified": true, "copyright": "暂无版权及作者信息"} {"instruction": "如何证明 f(x) 在 [0,1] 上最多只有有限个零点?\n自己做的时候无意间忽略了 $f^{\\prime}$ 不连续的情况,被指出错误后不知道该如何下手了,题目如下:\n设函数 $f(x)$ 在区间 $[0,1]$ 可导, 且 $\\left\\{x \\in[0,1] \\mid f(x)=0, f^{\\prime}(x)=0\\right\\}=\\varnothing$, 证明: $f(x)$ 在 $[0,1]$ 上最多只有有限个零点.", "input": "", "output": "考虑利用反证法。设若 $$f(x)$$ 在 $$[0,1]$$ 上有无穷多个零点,则依 $$\\text{Bolzano-Weierstrass}$$定理,必可求得一列零点 $$\\\\{x_n\\\\}$$ 使得,当 $$n\\to \\infty$$ 时成立 $$x_n\\to x^{*}\\in[0,1].$$\n\n很清楚,由于 $$f(x)$$ 连续,则 $$f(x^*)=\\lim_{n \\to \\infty}f(x_n)=\\lim_{n \\to\\infty}0=0,\\\\\\$$ 同时,由于 $$f(x)$$ 在 $$x=x^{*}$$ 可导,也即 $$f'(x^*)=\\lim_{x \\tox^*}\\frac{f(x)-f(x^{*})}{x-x^*},\\\\\\$$ 存在,于是依 $$\\text{Heine}$$ 定理,又有$$0=\\lim_{n \\to \\infty}\\frac{f(x_n)-f(x^*)}{x_n-x^*}=f'(x^*),\\\\\\$$ 于是$$f'(x^*)=0.$$ 由此可见,存在 $$x^*\\in [0,1]$$ 满足 $$f(x^*)=f'(x^*)=0,$$这违反了那个有关空集的题设。", "task_type": {"major": ["问答"], "minor": ["知乎问答"]}, "domain": ["通用"], "metadata": "qid:553542307, aid:2673774711", "answer_from": "human", "human_verified": true, "copyright": "暂无版权及作者信息"} {"instruction": "这个级数如何求出来呢?直接求解不要文字解释\n$\\sum_{n=0}^{\\infty} \frac{(-1)^n}{(2 n+1)^3}=\frac{\\pi^3}{32}$", "input": "", "output": "$$\\begin{align*}&~~~~\\sum_{n=0}^{\\infty}\\frac{(-1)^n}{(2n+1)^3}\\\\\\&=\\frac{1}{2}\\sum_{n=0}^{\\infty}(-1)^n\\int_0^1x^{2n}\\ln^2 x{\\rm d}x\\\\\\&=\\frac{1}{2}\\int_0^1 \\ln^2x\\sum_{n=0}^{\\infty}(-1)^nx^{2n}{\\rm d}x\\\\\\ &=\\frac{1}{2}\\int_0^1\\frac{\\ln^2x}{1+x^2}{\\rm d}x\\\\\\&=\\frac{1}{4}\\int_0^1 \\frac{\\ln^2 x}{1+x^2}{\\rmd}x+\\frac{1}{4}\\underbrace{\\int_0^1\\frac{\\ln^2 x}{1+x^2}{\\rm d}x}_{x~\\mapsto~1/x}\\\\\\ &=\\frac{1}{4}\\int_0^1 \\frac{\\ln^2 x}{1+x^2}{\\rmd}x+\\frac{1}{4}\\int_1^{+\\infty} \\frac{\\ln^2 x}{1+x^2}{\\rmd}x\\\\\\&=\\frac{1}{4}\\underbrace{\\int_0^{+\\infty} \\frac{\\ln^2 x}{1+x^2}{\\rmd}x}_{x~\\mapsto \\tan x}\\\\\\ &=\\frac{1}{4}\\int_0^{\\frac{\\pi}{2}} \\ln^2 \\tanx{\\rm d}x\\\\\\ &=\\frac{1}{4}\\int_0^{\\frac{\\pi}{2}}(\\ln\\sin x-\\ln\\cos x)^2{\\rmd}x\\\\\\ &=\\frac{1}{4}\\int_0^{\\frac{\\pi}{2}}\\left(\\left( -\\ln2-\\sum_{n=1}^{\\infty}{\\frac{\\cos \\left( 2nx \\right)}{n}} \\right)-\\left(-\\ln2+\\sum_{n=1}^{\\infty}\\frac{(-1)^{n-1}\\cos(2nx)}{n}\\right)\\right)^2{\\rm d}x\\\\\\&=\\frac{1}{4}\\int_0^{\\frac{\\pi}{2}}\\left(\\sum_{n=1}^{\\infty}\\frac{(1+(-1)^{n-1})\\cos(2nx)}{n}\\right)^2{\\rmd}x\\\\\\&=\\int_0^{\\frac{\\pi}{2}}\\left(\\sum_{n=0}^{\\infty}\\frac{\\cos((4n+2)x)}{2n+1}\\right)^2{\\rmd}x\\\\\\&=\\int_0^{\\frac{\\pi}{2}}\\sum_{n=0}^{\\infty}\\frac{\\cos^2((4n+2)x)}{(2n+1)^2}{\\rmd}x\\\\\\&=\\sum_{n=0}^{\\infty}\\frac{1}{(2n+1)^2}\\int_0^{\\frac{\\pi}{2}}\\cos^2((4n+2)x){\\rmd}x\\\\\\ &=\\frac{\\pi}{4}\\sum_{n=0}^{\\infty}\\frac{1}{(2n+1)^2}\\\\\\&=\\frac{\\pi^3}{32}. \\end{align*}$$", "task_type": {"major": ["问答"], "minor": ["知乎问答"]}, "domain": ["通用"], "metadata": "qid:399350470, aid:1264075315", "answer_from": "human", "human_verified": true, "copyright": "暂无版权及作者信息"} {"instruction": "这个反常积分极限要怎么证明呢?\n$\\lim _{n \rightarrow \\infty} \\int_0^{+\\infty} e^{-x^n} d x=1$\n请尝试用最少的知识解决问题。", "input": "", "output": "尝试用最少的知识解决问题。当前问题并不需要什么高端工具,只要通过简单放缩并依夹逼定理即可求得结果。首先,通过拆分积分区间,可得\n\n$$A_n:=\\int_0^{+\\infty} e^{-x^n}{\\rm d}x=\\int_0^1 e^{-x^n}{\\rmd}x+\\int_1^{+\\infty} e^{-x^n}{\\rm d}x=:B_n+C_n,$$\n\n利用基本的不等式 $$(\\forall x)e^x\\ge1+x$$ 可得\n\n$$1-x^n\\le e^{-x^n}=\\frac{1}{e^{x^n}}\\le\\frac{1}{1+x^n}=1-\\frac{x^n}{1+x^n}.$$\n\n于是,对于 $$B_n,$$ 有\n\n$$\\begin{align*} \\frac{n}{n+1}=\\int_0^1 (1-x^n ){\\rm d}x\\le B_n\\le \\int_0^1\\left(1-\\frac{x^n}{2}\\right){\\rm d}x=\\frac{2n+1}{2n+2}, \\end{align*}$$\n\n依夹逼定理, $$\\lim_{n \\to \\infty} B_n=1.$$ 对于 $$C_n,$$ 有\n\n$$0\\le C_n\\le \\int_1^{+\\infty}\\frac{{\\rm d}x}{1+x^n}\\le \\int_1^{+\\infty}\\frac{{\\rm d}x}{x^n}=\\frac{1}{n-1},$$\n\n依夹逼定理, $$\\lim_{n \\to \\infty} C_n=0.$$ 综上, $$\\lim_{n \\to \\infty} A_n=1.$$", "task_type": {"major": ["问答"], "minor": ["知乎问答"]}, "domain": ["通用"], "metadata": "qid:498796656, aid:2223616734", "answer_from": "human", "human_verified": true, "copyright": "暂无版权及作者信息"} {"instruction": "如何证明素数无穷多?", "input": "", "output": "这里给出一个分析的证明,基本思想属于 $$\\text{Leonhard Euler}.$$\n\n仍然利用反证法。设若素数只有有限的 $$m$$ 个,把这 $$m$$ 个素数由小到大排成一列 $$p_1,p_2,p_3,\\cdots,p_m,\\\\\\$$自然,其中 $$p_1=2,p_2=3,p_3=5$$ 等等。\n\n对任意一个正整数 $$n,$$基于算术基本定理,它必能分解为这有限个素数相应幂次——次数取自然数,并且这里我们允许出现零次幂——的乘积。换言之,只要把这包含着零的自然数分配给这些素数作为幂的次数,就能生成任意的正整数。比如$$540=2^2\\cdot3^3\\cdot5^1=p_1^2p_2^3p_3^1 p_4^0p_5^0\\cdotsp_m^0,\\\\\\$$进而对任意的单位分数,也就都可以表为各素数倒数的相应幂次的乘积,也比如$$\\frac{1}{540}=\\frac{1}{p_1^2}\\cdot\\frac{1}{p_2^3}\\cdot\\frac{1}{p_3^1}\\cdot\\frac{1}{p_4^0}\\cdot\\frac{1}{p_5^0}\\cdots \\frac{1}{p_m^0}.\\\\\\$$ 由此,不难想见如下乘积$$P:=\\sum_{k=0}^\\infty \\frac{1}{p_1^k}\\cdot \\sum_{k=0}^\\infty \\frac{1}{p_2^k}\\cdots\\sum_{k=0}^\\infty \\frac{1}{p_m^k},\\\\\\$$ 事实上就表示了所有单位分数之和$$S:=\\sum_{k=1}^\\infty\\frac{1}{k},\\\\\\$$于是 $$P=S.$$ 但 $$P$$是有限的,因为利用几何级数的求和公式,可以得到 $$P=\\frac{p_1}{p_1-1}\\cdot\\frac{p_2}{p_2-1}\\cdots\\frac{p_m}{p_m-1},\\\\\\$$ 而基于分析的基本结论, $$S$$ 是发散到$$+\\infty$$ 的调和级数,矛盾,于是推翻反设。", "task_type": {"major": ["问答"], "minor": ["知乎问答"]}, "domain": ["通用"], "metadata": "qid:590703734, aid:3121132182", "answer_from": "human", "human_verified": true, "copyright": "暂无版权及作者信息"} {"instruction": "如何优雅地证明这条不等式?\n已知正实数 $a, b, c, d$ 满足 $a b c d=1$,\n求证 : $\\frac{a}{a^3+3}+\\frac{b}{b^3+3}+\\frac{c}{c^3+3}+\\frac{d}{d^3+3} \\leq 1$", "input": "", "output": "由于 $$a^3+3=(a^3+1+1)+1\\ge 3a+1,\\\\\\$$ 所以 $$\\sum\\frac{a}{a^3+3}\\le \\sum\\frac{a}{3a+1}=\\sum \\frac{1}{3+\\frac{1}{a}},\\\\\\$$若置$$x=\\frac{1}{a},y=\\frac{1}{b},z=\\frac{1}{c},w=\\frac{1}{d},$$ 则只需再证$$\\sum\\frac{1}{3+x}\\le 1,\\\\\\$$ 其中 $$xyzw=1.$$\n\n两边同时乘上 $$(3+x)(3+y)(3+z)(3+w),$$ 再作整理,这待证等价于$$3\\left(\\frac{1}{xy}+\\frac{1}{zw}+\\frac{1}{xz}+\\frac{1}{yw}+\\frac{1}{xw}+\\frac{1}{yz}\\right)+2\\left(\\frac{1}{x}+\\frac{1}{y}+\\frac{1}{z}+\\frac{1}{w}\\right)\\ge26,\\\\\\$$这是成立的,因为$$\\frac{1}{xy}+\\frac{1}{zw}+\\frac{1}{xz}+\\frac{1}{yw}+\\frac{1}{xw}+\\frac{1}{yz}\\ge6\\sqrt{\\frac{1}{(xyzw)^3}}=6,\\\\\\\\frac{1}{x}+\\frac{1}{y}+\\frac{1}{z}+\\frac{1}{w}\\ge4\\sqrt{\\frac{1}{xyzw}}=4.\\\\\\$$", "task_type": {"major": ["问答"], "minor": ["知乎问答"]}, "domain": ["通用"], "metadata": "qid:564853975, aid:2746106364", "answer_from": "human", "human_verified": true, "copyright": "暂无版权及作者信息"} {"instruction": "如何证明e³>20?", "input": "", "output": "考虑利用展开式进行放缩。事实上, $$\\forall x\\in(0,1),\\forall n\\in\\mathbb{N}$$ 都有\n\n$$\\begin{align*}\\ln\\left(\\frac{1+x}{1-x}\\right)&=2\\left(x+\\frac{x^3}{3}+\\frac{x^5}{5}+\\cdots+\\frac{x^{2k+1}}{2k+1}+\\cdots\\right)\\\\\\\\[8pt]&=2\\left(\\sum_{k=0}^n\\frac{x^{2k+1}}{2k+1}+\\sum_{k=n+1}^{\\infty}\\frac{x^{2k+1}}{2k+1}\\right)\\\\\\\\[8pt]&<2\\left(\\sum_{k=0}^n\\frac{x^{2k+1}}{2k+1}+\\sum_{k=n+1}^{\\infty}\\frac{x^{2k+1}}{2n+1}\\right)\\\\\\\\[8pt]&=2\\left(\\sum_{k=0}^n\\frac{x^{2k+1}}{2k+1}+\\frac{x^{2n+3}}{(2n+1)(1-x^2)}\\right).\\end{align*}\\\\\\$$ 赋值 $$x=\\frac{1}{3},n=2$$ 代入,得\n\n$$\\ln2<\\frac{1123}{1620}.\\\\\\$$ 赋值 $$x=\\frac{1}{9},n=0$$代入,得$$\\ln\\frac{5}{4}<\\frac{9}{40}.\\\\\\$$于是 $$\\ln 20=4\\ln 2+\\ln\\left(\\frac{5}{4}\\right)<4\\cdot\\frac{1123}{1620}+\\frac{9}{40}<3.\\\\\\$$", "task_type": {"major": ["问答"], "minor": ["知乎问答"]}, "domain": ["通用"], "metadata": "qid:613154710, aid:3136533443", "answer_from": "human", "human_verified": true, "copyright": "暂无版权及作者信息"} {"instruction": "已知 $x, y, z$ 均为正实数,证明如下不等式,不需要文字解释:\n$$\n\\frac{x}{\\sqrt{x^2+y^2}}+\\frac{y}{\\sqrt{y^2+z^2}}+\\frac{z}{\\sqrt{z^2+x^2}} \\leq \\frac{3 \\sqrt{2}}{2}\n$$", "input": "", "output": "$$\\begin{align*}&\\left(\\frac{x}{\\sqrt{x^2+y^2}}+\\frac{y}{\\sqrt{y^2+z^2}}+\\frac{z}{\\sqrt{z^2+x^2}}\\right)^2\\\\\\&\\le[x^2(y^2+z^2)+y^2(z^2+x^2)+z^2(x^2+y^2)]\\cdot\\\\\\&~~~~\\left[\\frac{1}{(x^2+y^2)(y^2+z^2)}+\\frac{1}{(y^2+z^2)(z^2+x^2)}+\\frac{1}{(z^2+x^2)(x^2+y^2)}\\right]\\\\\\&=4+\\frac{4x^2y^2z^2}{(x^2+y^2)(y^2+z^2)(z^2+x^2)}\\\\\\ &\\le4+\\frac{4x^2y^2z^2}{2xy\\cdot 2yz\\cdot 2zx}\\\\\\ &=\\frac{9}{2}. \\end{align*}\\\\\\$$\n\n因此得证:\n$$\n\\frac{x}{\\sqrt{x^2+y^2}}+\\frac{y}{\\sqrt{y^2+z^2}}+\\frac{z}{\\sqrt{z^2+x^2}} \\leq \\frac{3 \\sqrt{2}}{2}\n$$", "task_type": {"major": ["问答"], "minor": ["知乎问答"]}, "domain": ["通用"], "metadata": "qid:535415836, aid:2512544595", "answer_from": "human", "human_verified": true, "copyright": "暂无版权及作者信息"} {"instruction": "求极限(tantanx/sinsinx)^(1/x^2)?无需文字说明", "input": "", "output": "$$\\begin{align*} &\\lim_{x \\to 0}\\left(\\frac{\\tan \\tan x}{\\sin\\sinx}\\right)^{\\frac{1}{x^2}}\\\\\\\\[8pt] =&\\lim_{x \\to 0}\\left(1+\\frac{\\tan \\tanx-\\sin \\sin x}{\\sin\\sin x}\\right)^{\\frac{1}{x^2}}\\\\\\\\[8pt] =&\\exp\\lim_{x \\to0}\\frac{\\tan \\tan x-\\sin \\sin x}{x^2\\sin\\sin x}\\\\\\\\[8pt] =&\\exp\\lim_{x \\to0}\\frac{\\tan \\tan x-\\sin \\sin x}{x^3}\\\\\\\\[8pt] =&\\exp\\lim_{x \\to0}\\left(\\frac{\\tan \\tan x-\\tan x}{x^3}+\\frac{\\sin x-\\sin\\sinx}{x^3}+\\frac{\\tan x-\\sin x}{x^3}\\right)\\\\\\\\[8pt] =&\\exp\\lim_{x \\to0}\\left(\\frac{\\tan \\color{blue}{\\tan x}-\\color{blue}{\\tanx}}{(\\color{blue}{\\tan x})^3}+\\frac{\\color{red}{\\sin x}-\\sin\\color{red}{\\sinx}}{(\\color{red}{\\sin x})^3}+\\frac{\\tan x-\\sin x}{x^3}\\right)\\\\\\\\[8pt]=&\\exp\\lim_{x \\to 0}\\left(\\frac{\\tan x- x}{x^3}+\\frac{x-\\sinx}{x^3}+\\frac{\\tan x-\\sin x}{x^3}\\right)\\\\\\\\[8pt] =&\\exp\\lim_{x \\to0}\\frac{2(\\tan x-\\sin x)}{x^3}\\\\\\\\[8pt] =&\\rm {e}. \\end{align*}\\\\\\$$", "task_type": {"major": ["问答"], "minor": ["知乎问答"]}, "domain": ["通用"], "metadata": "qid:624363291, aid:3239133456", "answer_from": "human", "human_verified": true, "copyright": "暂无版权及作者信息"} {"instruction": "$\\sum_{n=1}^{\\infty}\\left[n \\ln \frac{2 n+1}{2 n-1}-1\right]$这个级数怎么求和呢?给出简洁优雅的过程", "input": "", "output": "$$\\begin{align*} &\\sum_{n=1}^{\\infty} \\left(n\\ln\\frac{2n+1}{2n-1}-1\\right)\\\\\\=&\\sum_{n=1}^{\\infty}\\left(n\\ln(2n+1)-n\\ln(2n-1)-1\\right)\\\\\\=&\\sum_{n=1}^{\\infty}\\int_0^1 \\left(\\frac{n}{x+2n}-\\frac{n}{x-2n}-1\\right){\\rmd}x\\\\\\ =&\\int_0^1\\sum_{n=1}^{\\infty}\\left(\\frac{n}{x+2n}-\\frac{n}{x-2n}-1\\right){\\rm d}x\\\\\\=&\\frac{1}{4}\\int_0^1x^2\\sum_{n=1}^{\\infty}\\frac{1}{n^2-\\left(\\frac{x}{2}\\right)^2}{\\rm d}x\\\\\\=&\\frac{1}{4}\\int_0^1 x^2\\cdot\\frac{1-\\pi\\left(\\frac{x}{2}\\right)\\cot\\pi\\left(\\frac{x}{2}\\right)}{2\\left(\\frac{x}{2}\\right)^2}{\\rm d}x\\\\\\=&\\frac{1}{4}\\int_0^1 2-\\pi x\\cot\\left(\\frac{\\pi x}{2}\\right){\\rm d}x\\\\\\=&\\frac{1}{4}(2-2\\ln 2)\\\\\\ =&\\frac{1}{2}(1-\\ln 2). \\end{align*}\\\\\\$$", "task_type": {"major": ["问答"], "minor": ["知乎问答"]}, "domain": ["通用"], "metadata": "qid:429745975, aid:1569485453", "answer_from": "human", "human_verified": true, "copyright": "暂无版权及作者信息"} {"instruction": "请问这个极限怎么做呢?$\\lim _{n \rightarrow \\infty} \\sqrt{n} \\prod_{k=1}^n \frac{e^{1-1 / k}}{(1+1 / k)^k}$", "input": "", "output": "由于\n\n$$\\prod_{k=1}^n\\left(1+\\frac{1}{k}\\right)^k=\\prod_{k=1}^n\\frac{(k+1)^{k+1}}{k^k}\\cdot\\prod_{k=1}^n\\frac{1}{k+1}=\\frac{(n+1)^{n+1}}{(n+1)!},$$\n\n$$\\prod_{k=1}^n e^{1-\\frac{1}{k}}=\\exp\\sum_{k=1}^n\\left(1-\\frac{1}{k}\\right)=e^{n-H_n},$$\n\n所以\n\n$$\\begin{align*} &\\lim_{n \\to\\infty}\\sqrt{n}\\prod_{k=1}^n\\frac{e^{1-\\frac{1}{k}}}{(1+\\frac{1}{k})^k}\\\\\\=&\\lim_{n \\to \\infty}\\frac{\\sqrt{n}(n+1)!e^{n-H_n}}{(n+1)^{n+1}}\\\\\\ =&\\lim_{n\\to\\infty}\\frac{\\sqrt{n}\\sqrt{2\\pi(n+1)}\\left(\\frac{n+1}{e}\\right)^{n+1}e^{n-H_n}}{(n+1)^{n+1}}\\\\\\=&\\sqrt{2\\pi}\\lim_{n \\to \\infty}\\frac{\\sqrt{n^2+n}}{ne^{1+H_n-\\ln n}}\\\\\\=&\\frac{\\sqrt{2\\pi}}{e^{1+\\gamma}}. \\end{align*}$$", "task_type": {"major": ["问答"], "minor": ["知乎问答"]}, "domain": ["通用"], "metadata": "qid:512507247, aid:2318157979", "answer_from": "human", "human_verified": true, "copyright": "暂无版权及作者信息"} {"instruction": "这个极限怎么求,一个考研题?\n$x_n=\\sum_{k=0}^n \frac{1}{k !}$ 则 $\\lim _{n \rightarrow \\infty}\\left(\frac{\\ln x_n}{\\sqrt[n]{e}-1}-n\right)$", "input": "", "output": "$$\\begin{align*} L:&=\\lim_{n \\to \\infty}\\left(\\frac{\\lnx_n}{\\sqrt[n]{e}-1}-n\\right)=\\lim_{n \\to \\infty}\\frac{\\lnx_n-n(\\sqrt[n]{e}-1)}{\\sqrt[n]{e}-1}\\\\\\ &=\\lim_{n \\to \\infty}\\frac{\\lnx_n-n(\\sqrt[n]{e}-1)}{\\frac{1}{n}}\\\\\\ &=\\lim_{n \\to \\infty} n(\\lnx_n-1)-[n^2(\\sqrt[n]{e}-1)-n].\\\\\\ \\end{align*}$$\n\n其中\n\n$$\\begin{align*} J:&=\\lim_{n \\to \\infty}n(\\ln x_n-1)=\\lim_{n \\to \\infty}n\\ln\\left(1-\\sum_{k=n+1}^{\\infty}\\frac{1}{ek!}\\right)\\\\\\ &=\\lim_{n \\to\\infty}n\\left(-\\sum_{k=n+1}^{\\infty}\\frac{1}{ek!}\\right)=-\\frac{1}{e}\\lim_{n\\to \\infty}\\sum_{k=n+1}^{\\infty}\\frac{n}{k!}, \\end{align*}$$\n\n而\n\n$$0\\le\\sum_{k=n+1}^{\\infty}\\frac{n}{k!}\\le \\sum_{k=n+1}^{\\infty}\\frac{k}{k!}=\\sum_{k=n}^{\\infty}\\frac{1}{k!}\\to0(n\\to \\infty),$$\n\n所以 $$J=0.$$ 同时\n\n$$\\begin{align*} K:&=\\lim_{n \\to \\infty} \\left[n^2(\\sqrt[n]{e}-1)-n\\right]=\\lim_{n \\to \\infty}\\left[n^2\\left(\\frac{1}{n}+\\frac{1}{2n^2}+\\cdots\\right)-n\\right]=\\frac{1}{2},\\end{align*}$$\n\n所以\n\n$$L=J-K=-\\frac{1}{2}.$$", "task_type": {"major": ["问答"], "minor": ["知乎问答"]}, "domain": ["通用"], "metadata": "qid:512991064, aid:2322652733", "answer_from": "human", "human_verified": true, "copyright": "暂无版权及作者信息"} {"instruction": "设 $a_{n+1}=a_n+\\frac{1}{a_n} , a_1=1$ ,\n证明 $\\lim _{n \\rightarrow \\infty} \\frac{a_n}{\\sqrt{n}}=\\sqrt{2}$,\n求 $\\lim _{n \\rightarrow \\infty} \\frac{\\sqrt{n}\\left(a_n-\\sqrt{2 n}\\right)}{\\ln n}$", "input": "", "output": "首先, $$\\\\{a_n\\\\}$$ 单调递增,但并不收敛。若其不然,设 $$a_n\\to L<+\\infty,$$ 则将有$$L=L+\\frac{1}{L},$$ 矛盾。于是, $$a_n\\to+\\infty.$$\n\n进而 $$a_{n+1}^2-a_n^2=2+\\frac{1}{a_n^2}\\to2,\\\\\\$$ 再依 $$\\text{Cesaro-Stolz}$$ 定理$$\\frac{a_ {n+1}^2-1}{n}=\\frac{1}{n}\\sum_{k=1}^n(a_{k+1}^2-a_k^2)\\to 2,\\\\\\$$这表明了 $$a_n\\sim \\sqrt{2n}.\\\\\\$$ 由此,即得 $$a_{n+1}^2-a_n^2-2=\\frac{1}{a_n^2}\\sim\\frac{1}{2n}.\\\\\\$$ 所以 $$\\sum_{k=1}^{n-1}(a_{k+1}^2-a_k^2-2)\\sim\\sum_{k=1}^{n-1}\\frac{1}{2k}\\sim \\frac{1}{2}\\ln n,\\\\\\$$ 于是 $$a_n^2-2n\\sim\\frac{1}{2}\\ln n.\\\\\\$$ 所以 $$\\begin{align*} a_n&=\\sqrt{2n+\\frac{\\ln n}{2}+o(\\lnn)}\\\\\\ &=\\sqrt{2n}\\sqrt{1+\\frac{\\ln n}{4n}+o\\left(\\frac{\\ln n}{n}\\right)}\\\\\\&=\\sqrt{2n}\\left(1+\\frac{\\ln n}{8n}+o\\left(\\frac{\\ln n}{n}\\right)\\right)\\\\\\&=\\sqrt{2n}+\\frac{\\ln n}{4\\sqrt{2n}}+o\\left(\\frac{\\ln n}{\\sqrt{n}}\\right).\\end{align*}\\\\\\$$ 所以 $$\\lim_{n \\to\\infty}\\frac{a_n}{\\sqrt{n}}=\\sqrt{2},~~~~~~~~~~~\\lim_{n \\to\\infty}\\frac{\\sqrt{n}(a_n-\\sqrt{2n})}{\\ln n}=\\frac{1}{4\\sqrt{2}}.\\\\\\$$", "task_type": {"major": ["问答"], "minor": ["知乎问答"]}, "domain": ["通用"], "metadata": "qid:570941271, aid:2791349277", "answer_from": "human", "human_verified": true, "copyright": "暂无版权及作者信息"} {"instruction": "请问这道极限题怎么求解?\n$\\lim _{x \rightarrow 1}\\left(\frac{m}{1-x^m}-\frac{n}{1-x^n}\right) \\quad\\left(m, n \\in N_{+}\right)$;", "input": "", "output": "只需利用如下结论\n\n> $$\\color{blue}{\\lim_{x \\to> 1}\\left(\\frac{k}{1-x^k}-\\frac{1}{1-x}\\right)=\\frac{k-1}{2}~~(k\\in\\mathbb{N_+}).}\\\\\\$$\n\n这证明很容易,考虑如下因式分解即可:\n\n$$\\begin{align*}&~~~~~\\frac{k}{1-x^k}-\\frac{1}{1-x}\\\\\\&=\\frac{k}{1-x^k}-\\frac{1+x+x^2+\\cdots+x^{k-1}}{1-x^k}\\\\\\&=\\frac{(1-1)+(1-x)+(1-x^2)+\\cdots+(1-x^{k-1})}{1-x^k}\\\\\\&=\\frac{0+1+(1+x)+\\cdots+(1+x+x^2+\\cdots+x^{k-2})}{1+x+x^2+\\cdots+x^{k-1}}.\\end{align*}\\\\\\$$\n\n对于当前问题,有\n\n$$\\begin{align*} &~~~~\\lim_{x \\to1}\\left(\\frac{m}{1-x^m}-\\frac{n}{1-x^n}\\right)\\\\\\&=\\lim_{x \\to1}\\left(\\frac{m}{1-x^m}-\\frac{1}{1-x}\\right)-\\lim_{x \\to1}\\left(\\frac{n}{1-x^n}-\\frac{1}{1-x}\\right)\\\\\\&=\\frac{m-1}{2}-\\frac{n-1}{2}\\\\\\ &=\\frac{m-n}{2}. \\end{align*}\\\\\\$$", "task_type": {"major": ["问答"], "minor": ["知乎问答"]}, "domain": ["通用"], "metadata": "qid:478204386, aid:2047531015", "answer_from": "human", "human_verified": true, "copyright": "暂无版权及作者信息"} {"instruction": "如何用算数均值不等式证明1+1/n的n+1次方单调递减且有下界?", "input": "", "output": "记\n\n$$a_n:=\\left(1+\\frac{1}{n}\\right)^{n+1},~~~n=1,2,\\cdots\\\\\\$$\n\n则\n\n$$\\begin{align*}\\frac{1}{a_n}&=\\frac{1}{\\left(1+\\dfrac{1}{n}\\right)^{n+1}}=\\left(1-\\frac{1}{n+1}\\right)^{n+1}\\\\\\&=\\left(1-\\frac{1}{n+1}\\right)\\cdot\\left(1-\\frac{1}{n+1}\\right)\\cdots\\left(1-\\frac{1}{n+1}\\right)\\cdot1\\\\\\ &\\leq\\left[\\dfrac{\\left(1-\\dfrac{1}{n+1}\\right)+\\left(1-\\dfrac{1}{n+1}\\right)+\\cdots+\\left(1-\\dfrac{1}{n+1}\\right)+1}{n+2}\\right]^{n+2}\\\\\\&= \\left[\\dfrac{(n+1)\\left(1-\\dfrac{1}{n+1}\\right)+1}{n+2}\\right]^{n+2}\\\\\\&=\\frac{1}{\\left(1+\\dfrac{1}{n+1}\\right)^{n+2}}\\\\\\ &=\\frac{1}{a_{n+1}},\\end{align*}\\\\\\$$\n\n这就表明 $$a_n \\geq a_{n+1},$$ 由此证得了单调性。至于有界性,显然有 $$0\\leq a_n\\leq a_1=4.$$\n\n* * *\n\n利用AM-HM不等式(调和平均不超过几何平均)的另证:\n\n$$\\left(1+\\frac{1}{n}\\right)^{n+1}=1\\cdot\\frac{n+1}{n}\\cdots\\frac{n+1}{n}\\geq\\left(\\frac{n+2}{1+\\frac{n}{n+1}+\\cdots+\\frac{n}{n+1}}\\right)^{n+2}=\\left(1+\\frac{1}{n+1}\\right)^{n+2}.$$", "task_type": {"major": ["问答"], "minor": ["知乎问答"]}, "domain": ["通用"], "metadata": "qid:350217456, aid:853759225", "answer_from": "human", "human_verified": true, "copyright": "暂无版权及作者信息"} {"instruction": "这个极限如何计算?$\\lim _{n \rightarrow \\infty} \frac{\\sum_{k=1}^n k^2 \\cdot k^{\frac{1}{k}}}{n^3}$", "input": "", "output": "这个问题可以利用极限定义、分而治之地解决。\n\n首先, $$k^{\\frac{1}{k}}\\ge1(\\forall k),$$ 于是\n\n$$\\color{blue}{\\frac{1}{n^3}\\sum_{k=1}^nk^2k^{\\frac{1}{k}}\\ge\\frac{1}{n^3}\\sum_{k=1}^nk^2.}$$\n\n同时,我们熟知 $$\\lim_{k \\to \\infty}k^{\\frac{1}{k}}=1,$$ 这表明:\n\n$$\\forall \\varepsilon>0,\\exists N>0,$$ 当 $$k> N$$ 时,有 $$k^{\\frac{1}{k}}\\le1+\\varepsilon.$$\n\n于是\n\n$$\\begin{align*}\\color{blue}{\\frac{1}{n^3}\\sum_{k=1}^nk^2k^{\\frac{1}{k}}}&=\\frac{1}{n^3}\\sum_{k=1}^{N}k^2k^{\\frac{1}{k}}+\\frac{1}{n^3}\\sum_{k=N+1}^nk^2k^{\\frac{1}{k}}\\\\\\ &\\color{blue}{\\le\\frac{1}{n^3}\\sum_{k=1}^{N}k^2k^{\\frac{1}{k}}+\\frac{1+\\varepsilon}{n^3}\\sum_{k=N+1}^nk^2.}\\end{align*}$$\n\n对这两个方向的不等式(着蓝色)取 $$n\\to \\infty$$ 的上、下极限,即得\n\n$$\\frac{1}{3}\\le\\varliminf_{n \\to\\infty}\\frac{1}{n^3}\\sum_{k=1}^nk^2k^{\\frac{1}{k}}\\le\\varlimsup_{n \\to\\infty}\\frac{1}{n^3}\\sum_{k=1}^nk^2k^{\\frac{1}{k}}\\le\\frac{1+\\varepsilon}{3},$$\n\n再由 $$\\varepsilon>0$$ 的任意性,即得\n\n$$\\lim_{n \\to\\infty}\\frac{1}{n^3}\\sum_{k=1}^nk^2k^{\\frac{1}{k}}=\\frac{1}{3}.$$", "task_type": {"major": ["问答"], "minor": ["知乎问答"]}, "domain": ["通用"], "metadata": "qid:524683889, aid:2414501671", "answer_from": "human", "human_verified": true, "copyright": "暂无版权及作者信息"} {"instruction": "是否有办法绕开洛必达法则和泰勒展开来证明当x→0时x-sinx~(x^3)/6?", "input": "", "output": "由于求限函数 $$f(x)$$ 是偶函数,不妨径设 $$x>0$$ 求出右极限就够了。\n\n一方面,有 $$\\prod_{k=1}^n\\cos\\frac{x}{2^k}=\\frac{1}{2^n}\\frac{\\sinx}{\\sin\\frac{x}{2^n}}\\ge \\frac{1}{2^n}\\frac{\\sin x}{\\frac{x}{2^n}}=\\frac{\\sinx}{x},\\tag1$$ 进而 $$f(x)=\\frac{1}{x^2}\\left(1-\\frac{\\sin x}{x}\\right)\\ge\\frac{1}{x^2}\\left(1-\\prod_{k=1}^n\\cos\\frac{x}{2^k}\\right).\\tag2$$ 另一方面,将$$2^n\\cdot \\frac{\\sin\\frac{x}{2^n}}{\\cos\\frac{x}{2^n}}=2^n\\tan\\frac{x}{2^n}\\ge2^n\\cdot\\frac{x}{2^n}=x\\tag3$$ 结合 $$(1)$$ 又得 $$f(x)\\le \\frac{1}{x^2}\\left(1-\\cos\\frac{x}{2^n}\\prod_{k=1}^n\\cos\\frac{x}{2^k}\\right).\\tag4$$ 把 $$(2),(4)$$连接起来,取 $$x\\to 0_+$$ 的上、下极限,得 $$\\frac{1}{6}-\\frac{1}{6\\cdot 4^n}\\le\\varliminf_{x \\to 0_+}f(x)\\le\\varlimsup_{x \\to 0_+}f(x)\\le\\frac{1}{6}+\\frac{1}{3\\cdot 4^n},\\tag5$$ 这式子对一切 $$n$$ 成立,再命 $$n\\to \\infty,$$就有 $$\\frac{1}{6}\\le \\varliminf_{x \\to 0_+}f(x)\\le\\varlimsup_{x \\to 0_+}f(x)\\le\\frac{1}{6},\\tag6$$ 至此断定$$ \\lim_{x \\to 0_+}f(x)=\\frac{1}{6}.\\tag7$$", "task_type": {"major": ["问答"], "minor": ["知乎问答"]}, "domain": ["通用"], "metadata": "qid:626205412, aid:3251201564", "answer_from": "human", "human_verified": true, "copyright": "暂无版权及作者信息"} {"instruction": "若[x²]-2[x]>0,求实数x的取值范围?", "input": "", "output": "若 $$x<0,$$ 则 $$\\lfloor x^2 \\rfloor \\ge0, \\lfloor x \\rfloor \\le-1,$$ 于是$$\\lfloor x^2 \\rfloor-2\\lfloor x\\rfloor \\ge2>0,$$ 这得部分解。\n\n若 $$x\\ge0,$$ 不妨设 $$x=n+r,$$ 其中 $$n\\in\\mathbb{N}, r\\in[0,1),$$ 代入要解的不等式则有\n\n$$\\begin{align*} &&\\lfloor (n+r)^2 \\rfloor-2n>0\\\\\\\\[8pt]&\\Leftrightarrow&\\lfloor n^2+2nr+r^2 \\rfloor-2n>0\\\\\\\\[8pt]&\\Leftrightarrow&n(n-2)+\\lfloor 2nr+r^2 \\rfloor>0\\tag1\\\\\\\\[8pt]\\end{align*}\\\\\\$$ 这时很清楚,若 $$n\\ge3,$$ 必有 $$n(n-2)+\\lfloor 2nr+r^2 \\rfloor\\ge3+0>0,\\\\\\$$ 于是 $$(1)$$ 成立,这又得部分解。\n\n接下来,只需要再考虑 $$n=0,1,2$$ 时的情形。\n\n若 $$n=0,$$ 则 $$n(n-2)+\\lfloor 2nr+r^2 \\rfloor=\\lfloor r^2 \\rfloor=0,\\\\\\$$$$(1)$$ 不成立。\n\n若 $$n=1,$$ 则 $$(1)$$式化为 $$\\lfloor 2r+r^2 \\rfloor>1,\\\\\\$$ 也即$$\\begin{cases}2r+r^2\\ge2 \\\\\\ 0\\le r<1\\end{cases}\\\\\\$$ 解得 $$\\sqrt{3}-1\\ler<1,\\\\\\$$ 于是 $$\\sqrt{3}\\le x<2$$ 是 $$(1)$$ 的解。\n\n若 $$n=2,$$ 则 $$(1)$$式化为 $$\\lfloor 4r+r^2 \\rfloor>0,\\\\\\$$ 也即$$\\begin{cases}4r+r^2\\ge1 \\\\\\ 0\\le r<1\\end{cases}\\\\\\$$ 解得 $$\\sqrt{5}-2\\ler<1,\\\\\\$$ 于是 $$\\sqrt{5}\\le x<3$$ 是 $$(1)$$ 的解。\n\n综上几方面的讨论,可以写出所求解集为 $$x\\in (-\\infty,0)~\\bigcup~ [\\sqrt{3},2)~\\bigcup~[\\sqrt{5},+\\infty).\\\\\\$$", "task_type": {"major": ["问答"], "minor": ["知乎问答"]}, "domain": ["通用"], "metadata": "qid:611344743, aid:3115803986", "answer_from": "human", "human_verified": true, "copyright": "暂无版权及作者信息"} {"instruction": "一个级数问题,如何求解?", "input": "", "output": "事实上, $$f(x):=\\frac{1}{1-x-x^2}$$ 就是著名的 $$\\text{Fibonacci}$$ 序列 $$\\\\{F_n\\\\}$$的生成函数。\n\n为了证得这个事实,考虑 $$f(x)$$ 的展开式\n\n$$f(x)=a_0+a_1x+a_2x^2+\\cdots,$$\n\n将 $$x=0$$ 代入 $$f(x),$$ 得 $$a_0=1;$$ 再将 $$x=0$$ 带入 $$f'(x),$$ 得 $$a_1=1.$$\n\n此时,注意到\n\n$$\\begin{align*} f(x)-1-x=&a_2x^2+a_3x^3+\\cdots,\\\\\\xf(x)-x=&a_1x^2+a_2x^3+\\cdots,\\\\\\ x^2f(x)=&a_0x^2+a_1x^3+\\cdots,\\end{align*}$$\n\n为证 $$a_{n+2}=a_{n+1}+a_n,$$ 只需证\n\n$$f(x)-1-x=xf(x)-x+x^2f(x),$$\n\n这是显然成立的,因为这函数方程的解本来就是 $$f(x)=\\frac{1}{1-x-x^2}.$$\n\n对于当前问题,由于 $$a_n=F_n,$$ 进而\n\n$$\\\\[\\sum_{n=1}^{\\infty} \\frac{F_{n+1}}{F_nF_{n+2}} = \\sum_{n=1}^{\\infty}\\frac{F_{n+2} - F_n}{F_nF_{n+2}} = \\sum_{n=1}^{\\infty} \\left(\\frac{1}{F_n} -\\frac{1}{F_{n+2}} \\right) = \\frac{1}{F_1} + \\frac{1}{F_2} = 2.\\\\]$$", "task_type": {"major": ["问答"], "minor": ["知乎问答"]}, "domain": ["通用"], "metadata": "qid:502158456, aid:2246337566", "answer_from": "human", "human_verified": true, "copyright": "暂无版权及作者信息"} {"instruction": "一道三角函数的积分题,有没有比较方便的做法?$\\int_0^{2 \\pi} \frac{\\mathrm{d} \theta}{(\\sin \theta+\\cos \theta+\\sqrt{3})^2}$.", "input": "", "output": "首先,通过适当变形,可得\n\n$$\\begin{align*} &\\int_0^{2\\pi} \\frac{{\\rm d}\\theta}{(\\sin \\theta+\\cos\\theta+\\sqrt{3})^2}=\\int_0^{2\\pi} \\frac{{\\rmd}\\theta}{\\left[\\sqrt{2}\\cos\\left(\\theta-\\frac{\\pi}{4}\\right)+\\sqrt{3}\\right]^2}\\\\\\=&\\int_{-\\frac{\\pi}{4}}^{\\frac{7\\pi}{4}} \\frac{{\\rmd}\\theta}{\\left(\\sqrt{2}\\cos \\theta +\\sqrt{3}\\right)^2}=\\int_{0}^{2\\pi}\\frac{{\\rm d}\\theta}{\\left(\\sqrt{2}\\cos \\theta +\\sqrt{3}\\right)^2}.\\end{align*}\\\\\\$$\n\n现考虑极坐标方程 $$\\rho(\\theta)=\\frac{1}{\\sqrt{2}\\cos\\theta+\\sqrt{3}},\\\\\\$$事实上,它表示的是直角坐标系下的椭圆 $$\\frac{(x+\\sqrt{2})^2}{3}+y^2=1,\\\\\\$$ 这椭圆长半轴$$a=\\sqrt{3},$$ 短半轴 $$b=1,$$ 于是其面积\n\n$$A=\\pi ab=\\sqrt{3}\\pi.\\\\\\$$ 但这面积 $$A$$ 同样可以用极坐标形式下的定积分表出,即\n\n$$A=\\frac{1}{2}\\int_0^{2\\pi} \\rho^2{\\rm d}\\theta=\\frac{1}{2}\\int_{0}^{2\\pi}\\frac{{\\rm d}\\theta}{\\left(\\sqrt{2}\\cos \\theta +\\sqrt{3}\\right)^2} ,\\\\\\$$ 于是$$\\int_0^{2\\pi} \\frac{{\\rm d}\\theta}{(\\sin \\theta+\\cos\\theta+\\sqrt{3})^2}=\\int_{0}^{2\\pi} \\frac{{\\rm d}\\theta}{\\left(\\sqrt{2}\\cos\\theta +\\sqrt{3}\\right)^2}=2\\sqrt{3}\\pi.\\\\\\$$", "task_type": {"major": ["问答"], "minor": ["知乎问答"]}, "domain": ["通用"], "metadata": "qid:427786981, aid:1554276659", "answer_from": "human", "human_verified": true, "copyright": "暂无版权及作者信息"} {"instruction": "为什么明火炒菜比电磁炉好吃?", "input": "", "output": "其实不仅电磁炉和明火做饭口味有差别,不同功率的明火之间也有差别,甚至厨师对油温火候的把控也会对口味产生影响。通常而言,明火做饭更容易产生爆炒的香味,有的人称这种特殊的香味为锅气。锅气的背后,是复杂且在烹饪中极为重要的一类化学反应:美拉德反应(Maillardreaction)。\n\n美拉德反应代指食物中的还原糖(碳水化合物)与氨基酸/蛋白质在常温或加热时发生的一系列复杂反应,其结果是生成了棕黑色的大分子物质:类黑精。除产生类黑精外,反应过程中还会产生成百上千个有不同气味的中间产物,包括还原酮、醛和杂环化合物,这些物质为食品提供了宜人可口的风味和诱人的色泽。\n\n美拉德反应能快速地在约140℃至170℃环境下进行。虽然电磁炉的温度能够达到200-300度,但功率却只有1000-2000w左右。相比燃气灶,动辄三四千瓦,饭店里的燃气灶甚至能够超过5000w。这也是饭店里的厨师炒菜时能让锅内也燃起火来的原因。由于水的沸点在100摄氏度附近,功率低的烹饪设备没法立即让食物表面的水分快速蒸发掉,也就没法达到美拉德反应的适合温度,因此缺乏炒菜的香味。高功率的烹饪设备,甚至让食物直接接触火焰(烧烤),也就能蒸发掉食物表面的水分,让其表面产生美拉德反应而增添焦香。当然,食物也并不适合长时间在这种温度下,因为高温也会破坏掉维生素、蛋白质和糖分。炒糊的饭可就不好吃了,不仅有苦味,还有一种叫做丙烯酰胺的致癌物。因此,急火往往伴随快炒,小火才有资格慢炖,美拉德反应虽好,可不要贪杯哦~\n\n其实早在1912年时,美拉德就将氨基酸和糖类水溶液混合加热后,溶液产生黄棕色的发现讲给法国科学院的其他科学家听,但当时在座者中几乎没有人能意识到这个反应背后的潜在意义。直到1953年,化学家约翰·霍奇发表论文系统地整理了这类反应,美拉德反应才步入大多数人的视野。\n\n烤面包、褐色的牛排、羊肉串、半透明的红烧肉、烤鱼上的焦褐层、金黄色的宫保鸡丁、以及金色的炸薯条等等都有美拉德反应的身影哦~可惜的是,1953年霍奇总结美拉德反应时,美拉德已经去世了。如果美拉德知道他的发现成为如今许多厨师们的理论基础,为广大极饿势力做了这么大的贡献,想必也会十分欣慰吧。", "task_type": {"major": ["问答"], "minor": ["知乎问答"]}, "domain": ["通用"], "metadata": "qid:340697589, aid:2363967181", "answer_from": "human", "human_verified": true, "copyright": "暂无版权及作者信息"} {"instruction": "晚上盖着被子睡觉,为什么被窝里的温度不会越来越热,而是最终维持适宜的温度?", "input": "", "output": "当环境湿度在 $$40\\%-70\\%$$ 的时候,人体感觉最舒适的温度在 $$18^{\\circ}C-25^{\\circ}C$$,低于正常体温。而盖被子时,被子越厚保温效果越好。另外,人体也在不停地发热,发出的热量一部分穿透被子散发到空气中,另一部分会造成被窝内温度上升。如果留在被窝内的热量足够多,就会导致被窝温度超出人体适宜温度上限,人就会感觉到热。从这些分析可以看出,在严严实实裹好被子的情况下,如果睡得足够久,被子的厚度是决定被窝温度的关键。\n\n我们试着建立一个稍微精确些的模型。我们假设人晚上都是裸睡的,并且把人体抽象成一个半径 $$25cm$$,恒温的无限长圆柱。假设人体是一个绝对黑体,那么根据斯特蕃—玻尔兹曼定律可以得到人体单位面积的发热功率:\n\n$$P=5.67\\times10^{-8}W/(m^2K^4)\\times(310K)^4\\approx50mW/{cm}^2$$\n\n再假设被子是一层热导率与棉花热导率 $$\\kappa=0.05W/(m\\cdot K)$$ 相同、质量定压热容与棉花热容$$c=1.25\\times10^3J/(kg\\cdot K)$$ 相同的厚度为 $$d$$ 的圆筒,另取棉花密度 $$\\rho=0.3g/cm^3$$。环境温度 $$T_0=290K$$ (大约 $$17^{\\circ}C$$ ),与北方冬天城市供暖标准基本一致。\n\n这样问题就被简化成了一个内径 $$R_1=25cm$$ ,外径 $$R_2=(25+d)cm$$ 的圆筒传热问题,满足方程 $$\\frac{\\partialT}{\\partial t}=\\frac{\\kappa}{\\rho c}(\\nabla^2T)$$ ,内壁 $$R_1$$ 处傅里叶导热定律$$P=-\\kappa\\frac{dT}{dr}$$ ,外壁处牛顿冷却定律 $$q=h(T(R_2)-T_0)$$ ,取空气对流换热系数 (量级在$$1-10$$ ), $$q$$ 为外壁单位面积的传热功率。此外体系满足初始条件 $$T(r)=T_0$$ ,稳态边界条件 $$T(R_2)=T_0$$。\n\n经过简单的计算,我们可以定性地得到结果:被子厚度 $$d$$ 越大,达到相对稳态时被子内表面温度 $$T(R_1)$$就越高,达到稳态花费的的时间$$\\Delta t$$也越长。被子无穷厚时,内表面的傅里叶导热定律要求 $$T(R_1)$$ 略小于$$37^{\\circ}C$$ ,被子厚度为 $$0$$ 时, $$\\Delta t\\rightarrow0$$ 。当稳态内表面温度为人体适宜的$$30$$ 摄氏度时,对应的$$\\Delta t$$明显小于每晚睡眠时间 $$8$$ 小时。\n\n所以,如果你的被子盖起来越来越热,请向你亲爱的妈妈求助,换一床薄一点的被子。如果妈妈觉得你冷而拒绝了你的请求,你也可以在半夜热醒了的时候翻一个身,把被子掀开/蹬掉。如果你实在睡得太死,不会翻身掀被子,那么请精确求解上面的模型,并代入你被子的厚度,按照解出来的被子升温曲线确定你的睡眠时间,以保证睡醒的时候刚好被子的温度达到适宜睡眠的温度上限,然后在睡醒以后把求解的结果告诉我。如果这还不能满足你睡眠的需求,请采用微积分的思想,把被子微分成若干层睡衣,每天晚上都多穿一层睡衣睡觉,不断实验。不出一年,一定能找到合适的睡衣层数。\n\n最后,祝安眠。", "task_type": {"major": ["问答"], "minor": ["知乎问答"]}, "domain": ["通用"], "metadata": "qid:512684957, aid:2319437081", "answer_from": "human", "human_verified": true, "copyright": "暂无版权及作者信息"} {"instruction": "苹果切开后时间长了会发生氧化而变色,是不是就是花青素被氧化所致?", "input": "", "output": "这个氧化的过程被称作酶促褐变(EnzymaticBrowning),是水果和蔬菜在加工和储存中,因为机械应力产生的结果。以苹果为例,当你切开它或者将它榨成果汁,细胞遭到破坏,释放出多酚氧化酶(Polyphenoloxidase,PPO),一种邻二酚氧化酶。通过将细胞中的底物多酚化合物氧化为邻醌,随后醌类物质通过与其他蛋白或者氨基酸聚合,醌类物质也可以自身聚合,形成肉眼可见的棕色。这里的底物通常是酚类化合物,最常见的是酪氨酸,也会有其他的氨基酸蛋白质酚类以及糖,只需要是含有单酚或者邻二酚结构。\n\n多酚氧化酶PPO广泛分布于高等植物和真菌之中,所以其他水果也同样存在着酶促褐变的过程。但是在不同水果和蔬菜中,PPO的活性水平和底物浓度存在差异,其中显著的包括苹果、土豆、核桃(这里指的是核桃的果肉)、芒果、牛油果、香蕉。\n\n而这种酶促褐变过程,在日常生活中是可以通过加入还原剂来缓解(柠檬汁富含还原性物质),或者加热灭活PPO酶阻止反应发生(沸水焯一下)。但与其大费周章的阻止褐变,不如赶紧吃掉。\n\n食品工业中对酶促褐变的抑制更加的重视,并且有着非常成熟的实践,这也是我们喝到的果汁,包括苹果汁,都是清亮的原因之一。当然除了抑制酶促褐变过程,我们也会有意去运用这一过程,不信你可以去泡一杯红茶或者普洱,自己观察那红褐色。", "task_type": {"major": ["问答"], "minor": ["知乎问答"]}, "domain": ["通用"], "metadata": "qid:20904124, aid:2310394149", "answer_from": "human", "human_verified": true, "copyright": "暂无版权及作者信息"} {"instruction": "羽绒服洗过之后就不暖和了吗?", "input": "", "output": "羽绒保暖性极好,帮助我们度过严冬。但羽绒清洗过后保暖性能确实会有所下降,不正确的清洗方式甚至可能毁了羽绒服。\n\n羽绒服的保暖性关键在于羽绒,羽绒的中心有一个极小的绒核,绒核上生长有像树枝一样的绒枝。大量羽绒形成的羽绒集合体的绒枝向不同方向伸展,形成一个巨大的网络,可以有效的保持空气。而空气的导热性极差,因此羽绒可以有效阻隔热量传递。另一方面,为了固定羽绒,防止跑绒,同时阻隔空气和水进入羽绒内部,羽绒服的面料一般采用高密度织物,采用涂层压胶等方式优化性能,以防止钻绒跑绒,还能防风防水。\n\n而清洗会影响羽绒和面料的性能。\n\n洗涤会使面料质地变松,表面的涂层还有可能会被压碎、脱落,使面料的透气性增强,增加空气的对流散热。反复的洗涤也会把绒枝折断,降低羽绒的蓬松度,也会影响羽绒服的保暖性能,在常见的保暖衣物中,羽绒清洗后保暖性能下降最明显。\n\n当然,羽绒服也不是不能清洗。但应尽量减少清洗次数,如果不是特别脏,可以只清洗局部。羽绒的清洗建议注意以下几点:\n\n1. 一定先看羽绒服的水洗标。2. 建议手洗。不能干洗,干洗用的药水会影响保暖性能。不建议机洗,机洗搅拧会使羽绒分布不均匀,不能甩干,由于羽绒服面料透水性差,水很难进入,进入又很难排出,甩干容易造成羽绒服内部出现空隙,且羽绒服与机桶内壁摩擦容易产生静电,简短说,甩干容易造成羽绒服爆炸。3. 建议用温水,30℃左右,温水有利于洗涤剂溶解,洗前要充分浸泡。4. 建议用中性洗涤剂,对羽绒伤害最小,不能用加酶的洗涤剂,也不能用碱性洗涤剂。5. 不能拧干,挤出水分后铺平晾干或挂起晾干,不能暴晒,不要熨烫,晾干后轻拍使羽绒重新蓬松。 6. 存放时建议不要使羽绒服受到压迫。", "task_type": {"major": ["问答"], "minor": ["知乎问答"]}, "domain": ["通用"], "metadata": "qid:27694397, aid:2284639123", "answer_from": "human", "human_verified": true, "copyright": "暂无版权及作者信息"} {"instruction": "电子秤称出来的是质量还是重力?\n物体的质量是不变的,而重力是随着地区纬度不同变化的,是不是说明电子秤测量出来的物体的值是重力?那电子天平呢?", "input": "", "output": "市面上的电子秤一般是根据胡克定律或者杠杆原理来设计的。我们来分析一下这两种秤。\n\n如果电子秤是根据胡克定律来设计的,则\n\n$$mg = k\\Delta x$$\n\n$$m = k\\Delta x / g$$\n\n其称量过程主要是将待称量物的重力转化为弹簧的形变,在弹簧的劲度系数和该地的重力加速度已知的情况下,就可以计算出物体的质量了,但是由于重力加速度是和纬度相关的,因此该秤在不同纬度地区使用得出的值有一点差别(如果该秤可以定位地区并使用该地的重力加速度的话,那么另当别论)。\n\n另外,实验室用的电子天平利用通电导线在磁场中受到的安培力来平衡待测物体的重力(也有其他原理的电子天平),如果磁场强度不变,电流大小与重力大小成正比。电子天平不使用传统的弹簧形变,而是使用电流大小来反应物体重力,进而算出质量,反应更灵敏,精度更高。\n\n可以看到,这两种称都是通过重力与其他力平衡计算得到重力最终换算成质量的,其结果都包含重力加速度,因此其准确性与纬度有一定关系。但秤显示的结果当然是质量,只是经过重力换算得到的,所以秤“称出来的”还是质量,而非重力。\n\n如果电子秤是根据杠杆平衡关系来设计的,则\n\n$$m_{测}L_{测}g = m_{标}L_{标}g$$\n\n$$m_{测} = m_{标}L_{标}/L_{测}$$\n\n根据动力矩等于阻力矩,得出待测物体的质量。使用这种原理的电子秤最终得出的数值和重力加速度无关,因此在不同纬度使用都是相同的,但是该方法需要保证秤砣的质量是已知的。\n\n综上,两种电子秤都是使用的物体的重力来测量物体的质量,因此可以说电子秤是称量物体的重力,但是按照胡克定律设计的电子秤与纬度有关,按照杠杆原理设计的电子秤与纬度无关。\n\n以上讨论仅限于这两种原理的电子秤,如有其它设计原理的电子秤,欢迎继续提问。", "task_type": {"major": ["问答"], "minor": ["知乎问答"]}, "domain": ["通用"], "metadata": "qid:501115225, aid:2244210390", "answer_from": "human", "human_verified": true, "copyright": "暂无版权及作者信息"} {"instruction": "如何跟孩子解释「为什么海水不会干涸」?", "input": "", "output": "首先,我们要想第一个问题。我们可以把地球近似看作一个封闭系统,即不与外界发生物质交换的系统,那海水会去哪呢?\n\n说人话啊,地球是有引力的,引力会束缚地球表面的物体不会离开地球,当然,事实上,地球大气每年会有一定逸散,但逸散量很少,而且逸散的分子主要是以氢和氦为主,至少对水分子而言,我们可以看作地球是对水分子的封闭系统,即地球的水分子不会离开地球,那么水分子就只能待在地球上,最多只是在地表的分布不同罢了。\n\n那第二个问题,水分子在地表有什么样的存在形式呢?\n\n水分子是很活泼的,如果他们能量高,可以理解为比较热,他们就会运动非常欢快,跑来跑去的,彼此之间没有束缚,这就是气态水;如果他们能量稍低,他们没那么热,运动就没有那么欢快,这就是液态水;如果他们能量很低,彼此运动就很少,可以理解为水分子很冷,只会在原位振动,彼此有很强的束缚,这就是固态水,也就是冰。不过大家一定要注意,影响水的状态的因素不只有温度,还有压强啊……\n\n这三种状态的水都是水,本质没有什么不同,只是所处的相不同。地球作为宜居星球是这样的。地表的环境下,水很容易在这三种状态之间转变。\n\n前两个问题只是铺垫,交代一下背景知识。\n\n第三个问题,为什么会问出“海水会不会干涸”呢?海水会因为什么干涸呢?是因为海水会蒸发所以减少吗?\n\n海水当然会蒸发,毕竟整天晒太阳,水分子接受来自太阳的能量,会摆脱彼此的束缚,称为气态水。但如上文所言,蒸发只是水换了一种状态,但他还是水,只是由液态水变成了气态水——水蒸气,水蒸气会进入大气,一定条件下可以冷凝成云,比如随大气流动登陆,遇到陆地爬升,温度降低冷凝成云,云就是一些小液滴。而云一定条件下又会凝结成雨,雨落到地面上,会渗入地下,或汇入江河湖海等地表径流,一般会随地势最终再流回海里。这就是地球的水循环,简单来说,就是蒸发成液态水后会再冷凝。当然了,水循环实际比这个复杂得多。\n\n地球的水循环整体上大致是平衡的,这一点你从大气也可以看得出来,大气中水含量一直都很低,比较稳定。当然,陆地水循环会受到人类活动的明显影响。\n\n实际上,因为全球变暖,导致两极冰川融化,这两年海平面实际在上升,也就是海水非但没少,还多了。\n\n因此,你完全不用担心,海水会蒸发干净,水循环会带他们回归大海。\n\n最后还有一个问题,你知道海水有多少吗?\n\n水是地球表面最多的化合物了,地球表面约四分之三都是水,水体一共有13.86亿立方千米,这其中几乎全部都是海水,淡水只有不到3%。平分给每个中国人的话,每人大致可以分到1立方千米,就是一共边长1千米的立方体,这些水重达10亿吨,按5毛一吨的水价算都值5亿呢,这么多水,你还记得吗?地球都流浪去了,还带着呢,甩不掉,也不能甩。\n\n出一个简明版:地球上的水是不会凭空消失的,但水在不同温度下有不同状态,因此水会以不同状态再地球表面转移,这就是水循环。对于海水,在太阳的照射下,海水会蒸发,但蒸发的水分子在大气中运动,被抬升时会重新冷凝,变成雨落在地面上,最终流入河里回到大海,因此海水并不会干涸。\n\n因此,海水总量很大,且大气水循环会让蒸发的海水回到海里,因此,至少就目前而言,海水是不会干涸的。", "task_type": {"major": ["问答"], "minor": ["知乎问答"]}, "domain": ["通用"], "metadata": "qid:487506041, aid:2224694421", "answer_from": "human", "human_verified": true, "copyright": "暂无版权及作者信息"} {"instruction": "风会影响声音传播速度吗?用古籍和物理原理解释", "input": "", "output": "《劝学》有这么一句:“顺风而呼,声非加急也,而闻者彰”(顺风呼喊,声音并不是跑的更快了,但是听的人感到更加清楚了)。\n\n下面让我们分析其中的物理原理。\n\n声音是一种波,是空气的振动,当空气流动起来之后,声波的传播可以分解成两种运动的叠加,空气的流动和声波相对于空气的运动。\n\n我们考虑顺风和逆风两种情况,下面涉及一点点公式推导:\n\n设风速为 $$u$$ ,声波在空气参考系中的传播速度为 $$v$$ ,声波自 $$S$$ 点发出,考虑距 $$S$$ 点 $$L$$的顺风和逆风的两个观察者 $$T_1$$ 和 $$T_2$$ 。\n\n声音自声源发出后,在空气参考系中以球面波向四周传播。\n\n首先我们计算顺风时的能量面密度,在空气参考系中,声波从声源发出后到被接收传播的等效距离是\n\n$$r_1=L-\\frac{L}{v+u}u=\\frac{v}{v+u}L$$\n\n因此顺风时,接收者收到的能量面密度为:\n\n$$\\sigma=\\frac{E}{4\\pi L^2}\\cdot {(\\frac{v+u}{v})}^2$$\n\n同理可以计算出逆风时,接收者收到的能量面密度为\n\n$$\\sigma=\\frac{E}{4\\pi L^2}\\cdot (\\frac{v-u}{v})^2$$\n\n所以说《劝学》中的“声非加急也”,从地面参考系上来看声音确实是“加急”了,而“闻者彰”的原因是因为顺风时,听者接收到的能量更多。\n\n但是可能事实情况并非如此理想,我们取典型的声速 $$340m/s$$ ,空中刮着微风 $$5m/s$$ ,顺风因为刮风带来的声音强度是无风时的$$(\\frac{345}{340})^2\\approx 1.03$$ 倍,实在是太小了。\n\n但是如果风特别大,听者甚至可能因为风声强于人声而听不清说话的声音。", "task_type": {"major": ["问答"], "minor": ["知乎问答"]}, "domain": ["通用"], "metadata": "qid:23127730, aid:2220380933", "answer_from": "human", "human_verified": true, "copyright": "暂无版权及作者信息"} {"instruction": "为什么烧水壶在烧水时的声音会这么响?", "input": "", "output": "这就是所谓的“开水不响,响水不开”啊。\n\n确实,水在即将沸腾时会发出声音,这时由于水过冷沸腾造成的。\n\n水加热到沸腾大概可以分为三个过程,分别时自然对流、过冷沸腾和饱和沸腾。其中自然对流阶段即水刚开始加热时,几乎没有什么噪声,这一阶段,溶解在水中的气体会被气泡包裹,上升,最后到水面析出。\n\n噪声主要来自于过冷沸腾阶段,大概是水被加热到50℃之上,会出现明显噪声。这一阶段,加热盘(或加热丝之类的)表面表面局部活化能达到汽化核心要求,会形成水蒸气气泡,即有小部分水沸腾成为水蒸气,然后被液态水包裹形成气泡。但加热盘周围水温较低,这就是过冷沸腾。\n\n水蒸气气泡只有生长到足够大才能脱离加热盘表面,你不妨观察一下,在水即将沸腾时,是否可以在加热盘(一般是水壶底部)看到小气泡。尽管过冷沸腾气泡脱离直径比饱和沸腾小一些,但过冷沸腾气泡生长缓慢,两个气泡合并后的气泡尺寸也可能达不到离开加热盘的条件,这时小气泡在加热盘表面,底部吸收热量膨胀,顶部接触冷水释放热量收缩,在加热盘表面振荡,这有可能就是噪声的来源。\n\n但水蒸气气泡总会离开加热盘表面的,尤其是随着水温升高,气泡会增多。气泡一旦离开加热盘,进入冷水中,便会被冷水吸收热量,气泡随之破裂,气泡破裂瞬间会振动水体,形成激波,过余能量以声能形式辐射,这就是噪声的来源。\n\n但当水温升高,大概在70℃之后,会逐渐有气泡不会破裂,而是在水体里不断上浮,直到水面,然后再水面破裂,释放大户水蒸气。由于大量气泡再上浮,这时水体会变得浑浊。随着温度升高,越来越多的气泡不会破裂,而是上浮,因此,噪声减小,知道水温达到100℃,水处于饱和沸腾阶段,噪声基本消失。\n\n这种噪声出现在过冷饱和阶段,因此水升温越快越明显。\n\n当然,也有壶嘴经过特殊涉及的壶,在水沸腾后,水蒸气大量通过壶嘴会形成哨音,提醒我们水开了,不过这就跟“烧水声”没啥关系了。", "task_type": {"major": ["问答"], "minor": ["知乎问答"]}, "domain": ["通用"], "metadata": "qid:22420641, aid:2217268809", "answer_from": "human", "human_verified": true, "copyright": "暂无版权及作者信息"} {"instruction": "为什么路灯的灯光多是黄色的,而不是白色的?这种灯有什么优缺点", "input": "", "output": "在LED出现以前,白光灯主要是白炽灯,路等使用的黄光灯是高压钠灯。根据资料,高压钠灯的发光效率是白炽灯的几倍,寿命是白炽灯的二十倍,成本更低,透雾性也更好。此外人眼对黄色光比较敏感,并且黄色光给人温暖的感觉,这些都有助于减少夜间交通事故发生的概率。\n\n我们来聊聊钠灯的缺点,毕竟如果缺点不符合路灯的需求,那么不管它有多少优点,都是会被一票否决的。\n\n高压钠灯的主要缺点在于显色性差。显色性是光源的一个评价指标,通俗来说就是将光源的光投到物体上,显示的颜色与物体本身颜色的区别。显示的颜色越接近物体本色,光源的显色性就越好。白炽灯的显色性较好,可以用在家用照明等多种照明场景。但是钠灯的显色性比较差,无论照在什么颜色的物体上,看过去都是黄澄澄的。刚好合适的是,道路照明对光源显色性要求不高,我们在路上只要能发现远处有一辆车开过来,能辨别它的大小(形状)和速度就可以了,并不需要区分这辆车到底是红色还是白色。\n\n因此,道路照明与高压钠灯几乎是一对“绝配”。路灯需要的优点钠灯几乎都有;钠灯的缺点也都可以被路灯所容忍。因此即使白光LED技术已经成熟,还是有大量的路灯采用高压钠灯。这样就可以把其他光源的产能用到更合适的使用场景去了。", "task_type": {"major": ["问答"], "minor": ["知乎问答"]}, "domain": ["通用"], "metadata": "qid:24699101, aid:2217260395", "answer_from": "human", "human_verified": true, "copyright": "暂无版权及作者信息"} {"instruction": "当孩子问「如果我每天搂着鸡蛋睡觉,能孵出小鸡吗」该如何回答?", "input": "", "output": "当然可以,呃,我是说理论上可以。\n\n鸡蛋上面又没有指纹识别或人脸……鸡脸识别啥的,只要满足孵化的物理条件,不管是鸡呀、鸭呀、鹅呀,还是猫啊、狗啊、猪啊的,都可以孵化小鸡。实际上,现代农业的养殖场都是人工孵化小鸡的。\n\n那我们就来看看孵化小鸡需要什么条件吧。\n\n鸡苗的孵化期为21天,在这期间,要控制鸡蛋处于适宜的温度、湿度,并要注意通风散热。\n\n首先来看温度。人工孵化小鸡有恒温孵化和变温孵化两种。如果采用恒温孵化,则应在孵化期前18天控制温度在37.8℃,出雏期(第19到21天)控制温度在37.3℃。而变温孵化则要在第1-7天控制温度在38.2℃-38.5℃之间,在第8-18天,控制温度在37.8℃-38.1℃,而在第19-21天控制温度为37.3-37.5℃之间。实际生产中一般不采用变温孵化,但变温孵化孵化率高一些。\n\n孵化时应该严格控制温度,温度过高,小鸡发育过快,会死;温度过低,小鸡发育过慢,也会死。举个例子,你把鸡蛋放到100℃的环境里,只需要几分钟,就可以吃了,啊,不是,就死了。在文献【1】中,当温度超过41℃,持续2-3个小时,小鸡胚胎就会全部死亡。温度低也很好理解(比如,我有一桶液氮……咳咳),当温度低至24℃,持续30小时,小鸡胚胎全部死亡。孵化时的温度变化应该控制在0.2-0.3℃之间。\n\n你看,恒温孵化的温度跟人体温还是很接近的嘛,在被窝里应该还是有可能做到的。\n\n第二是湿度。恒温孵化时,前18天相对湿度应该控制在60%-65%,19-21天湿度应为70%-75%;变温孵化前1-7天湿度60%-65%,第8-18天湿度因为50%,19-21天湿度应为70%-75%。\n\n湿度与蛋内水分蒸发和物质代谢有关,也影响胚胎散热,特别是出雏期(19-21天),适当的湿度和空气中的二氧化碳可以促使碳酸钙变成碳酸氢钙,使蛋壳变脆,有利于雏鸡破壳。\n\n看着旁边显示18%的湿度计,我觉得小朋友如果在北京的话,还是在夏天孵化吧。\n\n第三是通风。小鸡孵化过程中,胚胎会有气体代谢,因此需要有足够的新鲜空气,二氧化碳浓度不要超过0.5%,当二氧化碳浓度超过1%时,胚胎死亡率会上升。\n\n第四是翻蛋和凉蛋。生产中孵化过程会有这两步,翻蛋主要是为了让蛋受热均匀(想到煎鸡蛋了是怎么回事),防止胚胎与壳膜粘连,有助于胚胎运动和血液循环。时间间隔为2小时,18日之后可以停止翻蛋。凉蛋则是为了散热,尤其是孵化后期,胚胎代谢会产生大量热,不凉蛋它能烧死自己。凉蛋时间一般为每天上下午各一次。\n\n综上所述,鸡蛋孵化的条件并不算复杂,我们完全可以实现嘛。\n\n不过我觉你首先要解决两个问题。第一是防止睡觉时翻身把鸡蛋压碎了,我觉得你可以先试试,练习一下(妈见打行为)。第二是找到可以孵化的鸡蛋,我们日常吃的鸡蛋一般是没有受过精的。这种蛋,你放到它腐烂了都不可能孵化出小鸡的。\n\n祝你早日成功!", "task_type": {"major": ["问答"], "minor": ["知乎问答"]}, "domain": ["通用"], "metadata": "qid:487479832, aid:2193895934", "answer_from": "human", "human_verified": true, "copyright": "暂无版权及作者信息"} {"instruction": "身处同样的绝对温度,为什么夏季和冬季的感觉差别那么大?", "input": "", "output": "首先大家要明白一个问题,物理学上定义的温度和体感温度是不一样的!体感温度综合了环境因素,风速以及长短波辐射等作用,所以体感温度相对于物理上所定义的温度来衡量人在所处的环境中的舒适度更具有代表意义。\n\n根据1984年,罗伯特·史特德曼发表的《体感温度的通用公式》\n\n$\begin{aligned} & A T=1.07 T+0.2 e-0.65 V-2.7 \\ & e=\frac{R H}{100} \times 6.105 \times \\exp \frac{17.27 T}{237.7+T}\\end{aligned}$\n\n其中AT为体感温度(℃)、T为气温(℃)、e为水汽压(hPa)、V为风速(m/s)、RH为相对湿度(%)。\n\n根据以上公式可以判断出,体感温度(AT)与水汽压成正相关,与风速成负相关。\n\n下面我们以16摄氏度为例,估算一下夏季和冬季的体感温度。\n\n详细计算如下(冬夏取物理学上的温度为16摄氏度,通风的夏天房间的相对湿度取90%,冬天的相对湿度取10%,风速都取0m/s),并根据罗伯特·史特德曼所说\n\n首先,计算冬天与夏天的水汽压:\n\n$\begin{aligned} e_{\text {夏 }} & =\frac{R H}{100} \times 6.105 \times \\exp \\left(\frac{17.27 T}{237.7+T}\right) \\ & =\frac{90}{100} \times 6.105 \times \\exp \\left(\frac{17.27 \times 16}{237.7+16}\right) \\ & \\approx 16.267\\end{aligned}$\n\n$\begin{aligned} e_{\text {冬 }} & =\frac{R H}{100} \times 6.105 \times \\exp \\left(\frac{17.27 T}{237.7+T}\right) \\ & =\frac{10}{100} \times 6.105 \times \\exp \\left(\frac{17.27 \times 16}{237.7+16}\right) \\ & \\approx 1.808\\end{aligned}$\n\n再代入公式计算体感温度:\n\n$\begin{aligned} A T_{\text {夏 }} & =1.07 \times T+0.2 \times e_{\text {夏 }}-0.65 \times V-2.7 \\ & =1.07 \times 16+0.2 \times 16.268-0.65 \times 0-2.7 \\ & \\approx 17.673\\end{aligned}$\n\n$\begin{aligned} A T_{\text {冬 }} & =1.07 \times T+0.2 \times e_{\text {冬 }}-0.65 \times V-2.7 \\ & =1.07 \times 16+0.2 \times 1.808-0.65 \times 0-2.7 \\ & \\approx 14.782\\end{aligned}$\n\n则不难计算,冬夏体感温度差为:\n\n$17.673-14.782=2.891$\n\n也就是说,在上述的条件下,冬天的体感温度会比夏天的体感温度要低2.89摄氏度左右!!\n\n总结:冬天相对夏天的来说其相对湿度会变小,若再加上风速较大,即使和夏天温度一样时,身体也会感觉比较冷。", "task_type": {"major": ["问答"], "minor": ["知乎问答"]}, "domain": ["通用"], "metadata": "qid:22032452, aid:2183905221", "answer_from": "human", "human_verified": true, "copyright": "暂无版权及作者信息"} {"instruction": "为什么光子没有质量,还可以被黑洞吸引?\n就我所知,光子有波粒二象性,它的粒子形式是没有质量的。那么,根据 G=mg,若 m=0,G 自然也就等于零。为什么还可以被黑洞吸引?", "input": "", "output": "想要理解推导光子为何会落入黑洞,可以有几层不同的理解:\n\n一、\n\n其实从经典物理里面也能理解零质量的物体为何经过大质量天体时路径会发生偏折。\n\n只要我们不纠结这里的零质量是真的0,还是接近零的无限小。毕竟根据万有引力公式和牛顿第二定律,粒子本身的质量项在两侧是正好消掉了的(我们这里当然就继续默认引力质量和惯性质量相等了,毕竟验证引力质量和惯性质量相等的单摆实验牛顿就做过)。\n\n$$-\\frac{GMm}{r^3}{\\bf r}=m\\frac{\\mathrm{d}^2{\\bfr}}{\\mathrm{d}t^2}\\quad\\Rightarrow\\quad -\\frac{GM}{r^3}{\\bfr}=\\frac{\\mathrm{d}^2{\\bf r}}{\\mathrm{d}t^2}$$\n\n所以运动的轨迹仍旧是圆锥曲线中的一种,该弯还是要弯,和粒子的质量没有关系。\n\n> Soares, D.S.L. (2009) ‘Newtonian gravitational deflection of light> revisited’, arXiv:physics/0508030 [Preprint]. Available at:> http://arxiv.org/abs/physics/0508030 (Accessed: 22 March 2022).\n\n二、\n\n在牛顿引力的框架里,光子的质量为 0,自然万有引力为0,怎么经过天体以后还会偏折呢?其实根据广义相对论中的等效原理,引力场和加速度等价——把一个人放在密闭电梯里,他其实无法分辨自己是因为处在地球表面受到重力,亦或是在外太空中因为匀加速运动才能踩在地板上。所以在没有引力的时候,光线不应该偏折;在有引力的情况下,引力场等效于一个反向加速的参考系,自然光线发生了偏折。\n\n三、\n\n借助广义相对论的场方程计算大质量天体周围时空如何弯曲。虽然光子没有质量,但我们同样可以利用变分原理构造作用量,推导得到此时时空流形上的测地线就是短程线,这并不是直线。光会沿着路程最短的路线传播,自然光线会被弯曲,或者有可能落入黑洞了。这部分详细内容可翻阅任何广义相对论讲义。\n\n广义相对论的最终计算也证明了上面两种理解都不完全正确,实际体系里光子经过大质量天体偏转角度的大小会比上面的计算更大一些(在小角度情况下是上面计算结果的两倍)。\n\n形象地来说,黑洞的存在弯曲了其周围的时空,使得黑洞附近的所有物体都有向着黑洞运动的趋势。\n\n这就好比在一个橡皮膜上放一个小球,小球使得橡皮膜上产生了一个凹陷,这个凹陷促使橡皮膜上的其他小球掉落到这个凹陷之中。并且越靠近中心的地方越难以逃出。对于黑洞也是如此,黑洞扭曲了时空,这种扭曲的时空“指挥”光子应该如何运动,这就形成了我们看到的黑洞对于光子的引力。", "task_type": {"major": ["问答"], "minor": ["知乎问答"]}, "domain": ["通用"], "metadata": "qid:38680273, aid:2150619795", "answer_from": "human", "human_verified": true, "copyright": "暂无版权及作者信息"} {"instruction": "自地球诞生以来,地球上所有物质(不管有无生命且包括分子的内部活动)的运动路程总量加起来能到达宇宙边缘吗?", "input": "", "output": "直接上式子算!\n\n先考虑多一点的: $$18g$$ 的水分子,也就是 $$1 mol$$ ,在 $$300K$$ 下, $$1s$$ 的时间内走了大约多少路程呢?\n\n使用热力学估计水分子的平均速度: $$\\bar{v} = \\sqrt{\\frac{8RT}{\\pi M}} $$(此公式来源于对理想气体的麦克斯韦玻尔兹曼分布对v做积分求期望值)\n\n带入 $$T=300K$$ 、 $$R=8.31J/(mol K)$$ 、 $$M=0.018kg/mol$$\n\n得到结果大约是 $$600m/s$$ . 这个结果乘以 $$1s $$ 和阿伏伽德罗常数 $$N_A$$ 得到结果。\n\n算式:$$\\sqrt{\\frac{300\\times8\\times8.31}{π\\times0.018}}\\times6.02\\times10^{23}=3.575\\times10^{26}$$\n\n$$300K$$ 这个温度下, $$1mol$$ 水分子 $$1s$$ 走了约 $$3.6\\times10^{26}m$$ 。这是多少光年呢: $$1$$光年大约为 $$9.46\\times10^{15}m$$ 。换算一下单位到光年,上面的距离是 $$3.8\\times10^{10}$$ 光年,也就是$$380$$ 亿光年。而目前可观测宇宙的半径约是 $$465$$ 亿光年。\n\n哪需要自地球诞生以来、所有物质这些限定词啊。一掬水,几秒钟就做到了呀。", "task_type": {"major": ["问答"], "minor": ["知乎问答"]}, "domain": ["通用"], "metadata": "qid:488271715, aid:2133933083", "answer_from": "human", "human_verified": true, "copyright": "暂无版权及作者信息"} {"instruction": "什么是时间晶体?太阳系是时间晶体吗?", "input": "", "output": "时间晶体这个名字听起来容易让人联想到《信条》、《回到未来》这些操纵时间的科幻大片。然而实际上,时间晶体是一种特殊的物相。众所周知,空间晶体(也就是通常意义的晶体)中原子排列成空间中的周期性结构,这样的结构破缺了空间本来具有的连续平移对称性。朗道相变范式下,对称性破缺= 新物相,晶体因此被归类为与液体或气体截然不同的一种物态。\n\n我们知道,宇宙有三个空间维和一个时间维,那么类似的情况有没有可能发生在时间维度上呢?乍看起来,前后两个问题完全类似,物质在时间上形成周期性结构,就构成了所谓的“时间晶体”。但实际上,两者有一个重要区别:空间晶体是热力学平衡态下的物态,如果在绝对零度下,指的就是系统的基态。可是在量子力学中,具有确定能量的基态,一定保持着时间上的平移对称性,这就排除了基态存在时间晶体的可能$$\\\\(^{}\\\\)$$ 。\n\n既然孤立系统的基态不存在时间晶体,科学家们于是将目光转向非孤立系统 ——与外界存在相互作用,哈密顿量中显含时间的系统。这样的系统实际非常常见,例如经典力学中由外力驱动的弹簧振子就是一个非孤立系统。由于非孤立系统与外界存在能量交换,系统一般而言并没有具有确定能量的定态。但是同样类比于经典力学中振子的受迫振动,经过足够长时间,系统将会达到和驱动力具有相同频率的稳态,这样的稳态对应到量子力学中称为Floquet 态 $$^{}$$\n\n那么,具有时间周期性的 Floquet态,是不是就是我们寻找的“时间晶体”呢?其实并不完全是。类比于空间晶体,晶体的周期性是自发形成的,而不是周期性外势场作用下的结果,对应地,时间晶体应该也需要自发地形成和驱动周期不同的时间周期。然而,由于时间晶体必须在一定的外场驱动下形成,时间晶体的周期不能是任意的,而应该是驱动周期的整数倍$$^{}$$ ,为了明确起见,人们把这样的时间晶体称作“离散时间晶体”(Discrete Time Crystals,DTC)。\n\n我们前面已经提到,Floquet 态的周期与驱动周期相同,而 DTC 的周期与驱动周期不同,因此, DTC 并非 Floquet 态,而是多个Floquet 态的线性组合,并且,这样的线性组合在微扰下更为稳定。为了理解这点,我们可以把 Floquet 态想象成薛定谔猫半死不活的叠加态,而 DTC则是死或者活的状态,显然后者在外界的观测下更为稳定 $$^{}$$ 。因此,在实验中观测到的不是 Floquet 态,而是 DTC 相。\n\n不过,时间晶体也绝非罕见,如今,大量系统中都观测到了 DTC 相的存在。例如2018年耶鲁大学的研究团队就在一种常见的晶体 —— 磷酸二氢铵中观测到了DTC 相 $$^{}$$ 。他们施加磁场使晶体中磷原子的原子核发生核磁共振,并通过测量系统的磁矩发现了周期为驱动周期两倍的 DTC 相。", "task_type": {"major": ["问答"], "minor": ["知乎问答"]}, "domain": ["通用"], "metadata": "qid:477113452, aid:2053794166", "answer_from": "human", "human_verified": true, "copyright": "暂无版权及作者信息"} {"instruction": "既然现实中绝对零度是无法达到的,那么绝对零度的值是怎么确定的呢?", "input": "", "output": "绝对零度可以用理想气体理论推导出来。根据盖-吕萨克定律,压强恒定时,降低温度,气体的体积会压缩,并且这种变化是线性的。测量一段温度范围内体积随温度变化的规律,并延长到体积为零的位置,此时的温度值就是最早对绝对零度的定义。更先进的定义要用到关于热机循环的知识,这里就不细说了。物理学中常用单位K(开尔文)计量温度,绝对零度为0K,0℃为273.15K。\n\n目前尚没有方法可以实验上准确测出绝对零度的值。绝对零度是一个理论上的极限值,在接近绝对零度时,很多性质都会发生变化。比如,根据盖-吕萨克定律,保持恒定压强,降温至绝对零度,气体的体积会降为零。实际上这是不现实的,绝对零度附近,盖-吕萨克定律不再准确。再比如,开尔文认为绝对零度下粒子的能量为零(这其实也是如今很多人的认识),但这后来也被推翻。在接近绝对零度时,玻色子会全部占据最低能级形成“玻色-爱因斯坦凝聚”,而费米子会占据“费米能级”以下所有量子态,总之它们的能量都不为零。极低温下还有超导、超流等很多新奇的物态,这些都不能用经典物理的知识理解。\n\n物理学家有很多种降温的方法可以接近绝对零度。凝聚态物理中,使用“稀释制冷机”等手段可以降温到mK(0.001K)的水平。专门研究极低温物理的冷原子物理中,使用激光、磁场等辅助,可以实现nK(0.000000001K)量级的温度。", "task_type": {"major": ["问答"], "minor": ["知乎问答"]}, "domain": ["通用"], "metadata": "qid:26465889, aid:1968196984", "answer_from": "human", "human_verified": true, "copyright": "暂无版权及作者信息"} {"instruction": "物理学「常量」有没有可能是极其缓慢变化的量?", "input": "", "output": "物理学常量真的是亘古不变的常数吗?有没有可能是含时变量呢?\n\n这不仅仅是一个脑洞,更是许多物理学家们所关心的问题,为此他们对小到原子大到类星体进行了测量。\n\n## 引力常数G与大数假说\n\n1687年,牛顿在《自然哲学的数学原理》中发布了著名的万有引力定律:\n\n$$F=G \\frac{m_1m_2}{r^2}$$\n\n时至今日,万有引力定律已经成为高中物理课本上的基础知识,常数G被称为万有引力常数。但是,万有引力常数真的是常数吗?\n\n1938年,英国物理学家狄拉克发现,两个基本作用力——万有引力和电磁力的比例,和宇宙的年龄,其数量级相近。狄拉克认为,这不一定仅仅是一个巧合,也许背后有很深刻的物理。于是狄拉克提出了大数假说,认为万有引力常数是时间变化的,并且构造了万有引力常数反比于时间的宇宙学模型。\n\n## 精细结构常数\n\n精细结构常数(如无特别声明,指电磁相互作用的精细结构常数,下同)是物理学中定义的一个无量纲常量,其表达式为:\n\n$$\\alpha=\\frac{e^2}{2 \\epsilon_0 h c}$$\n\n精细结构常数是一系列重要物理常数的组合,其中 $$e$$ 是基本电荷, $$\\epsilon_0$$ 是真空介电常数, $$h$$ 是普朗克常数,$$c$$ 是真空中的光速。\n\n根据2014年国际科技数据委员会CODATA的推荐值, $$\\alpha=\\frac{1}{137.035999139(31)}$$\n\n一般近似计算中,经常取 $$\\frac{1}{137}$$ 。\n\n精细结构常数是德国物理学家索末菲在分析氢原子光谱时提出的,最开始是为了简化计算,但在后续的研究中,物理学家逐渐发现了精细结构常数背后越来越多的物理意义。\n\n精细结构常数是多个物理学常数的组合,其中基本电荷 $$e$$ 来自四大相互作用之一的电磁相互作用,普朗克常数 $$h$$ 来自量子力学,真空中的光速$$c$$ 来自相对论,精细结构常数可以描述电磁相互作用的耦合常数,它将几大物理学理论联系在了一起。\n\n费曼曾说:“这个数字自五十多年前发现以来一直是个谜。所有优秀的理论物理学家都将这个数贴在墙上,为它大伤脑筋……它是物理学中最大的谜之一,一个该死的谜”。\n\n## “魔数”与大数假说\n\n精细结构常数令物理学家大伤脑筋,费曼甚至把精细结构常数称为“魔数”。不过逐渐地,有人开始疑问,虽然大家一直把它当做一个常数,但是它真的是一个常数吗?\n\n这时候就需要用实验来证明。对精细结构常数的测量可以分为两种思路,其一是利用原子钟等工具测量精细结构常数现阶段是否在变化,即“现代测量”,其二是利用天文望远镜,对类星体等的光谱进行分析,从而判断大时间尺度上(例如一百亿年)精细结构常数是否发生变化,即“宇宙学测量”。\n\n美国宇航局喷气推进实验室的研究人员精确测量了铯原子钟、汞离子钟和氢原子微波激射器的频率在140天内的相对频率漂移。结果发现,在现阶段,精细结构常数的变化率不超过每年30万亿分之一,这个值和狄拉克大数假说的预言相差了一个数量级,因此可以认为否定了大数假说,目前狄拉克大数假说所构造的模型并不为主流的宇宙学理论所认可。\n\n2004年6月,德国的一些研究人员以很高的精度测量了原子钟的数据,并未发现精细结构常数在1999年至2003年间有 $$10^{-15}$$数量级上的变化。\n\n奥克洛天然核反应堆是目前已知的世界上唯一一座天然核反应堆,它形成于大约20亿年前。研究人员测量了奥克洛铀矿中钐149的中子散射截面,发现20亿年来强相互作用的精细结构常数的变化率不超过十亿分之四,年相对变化率不超过$$2\\times 10^{-19}$$。尽管得到的是强相互作用的精细结构常数变化率的数值,但是科学家们倾向于认为,如果精细结构常数的变化是由光速的改变引起的,那么强相互作用的精细结构常数与电磁作用的精细结构常数的变化应该是一致的。\n\n## “魔数”的宇宙学测量与论战\n\n此外,科学家也致力于通过宇宙学和天体物理学的手段对精细结构常数进行测量。宇宙中类星体发出的光穿过弥漫在宇宙中的气体云,形成吸收线。通过测量类星体光谱中的吸收线,可以得到几十亿到上百亿年前精细结构常数的信息。\n\n澳大利亚新南威尔士大学的天体物理学家韦伯领导的研究组从1998年开始类星体光谱进行测量。韦伯组通过对光谱中不同元素吸收线的位置变化,对精细结构常数的变化进行测量。\n\n2010年,他们在分析了153个类星体的相关数据后,认为100亿年前的精细结构常数和现在相差约100,000分之一,论文发表在PhysRevLett.107.191101。\n\n韦伯的论文发布之后,另一位理论物理学家卡罗尔提出了不同的观点。卡罗尔基于韦伯的观点出发,假设精细结构常数不是标量常数,而是一个标量场,并在此基础上做了一些理论计算,发现推导出的结果会与先前的另外一些结论相互冲突。基于此,卡罗尔认为韦伯的结论“总是存在漏洞”(“Thereare always loopholes”),并且认为韦伯的结论“值得怀疑”(“That's why theorists are skeptical ofthis claimed result”)。\n\n在去年发表在Science Advances上的一项研究中,韦伯组得到的最新结论是精细结构常数的相对偏移量为 $$\\Delta \\alpha/ \\alpha=(-2.18\\pm7.27)\\times 10^{-5}$$ 并且“consistent with no temporalchange”,即他们并不认为精细结构常数随时间发生变化。\n\n精细结构常数是否一成不变呢?目前科学家并不能拍着胸脯保证精细结构常数一定是一个标量常数,同时也没有公认的强力证据表明精细结构常数在随时间变化。随着实验技术的不断进步,如果将来某一天有公认的强力证据证明了精细结构常数在随时间变化,那可不仅仅是现有的物理学大厦被打破了几扇窗户的事情,而是发现大厦图纸设计错了的严重问题。", "task_type": {"major": ["问答"], "minor": ["知乎问答"]}, "domain": ["通用"], "metadata": "qid:466076685, aid:1968171791", "answer_from": "human", "human_verified": true, "copyright": "暂无版权及作者信息"} {"instruction": "等效原理中的引力与加速度的关系能否套用电磁效应的麦克斯韦公式?比如将引力场当成电场,动力场当成磁场?", "input": "", "output": "原则上讲,我们无法将引力场与电磁场直接对应起来,因为尽管它们都是规范玻色子,但引力子是自旋为2的无质量粒子,而光子是自旋为1的无质量粒子,有着完全不同的性质。下面将从经典的角度,展示将电磁场的麦克斯韦方程组直接套用在牛顿引力理论中会出现怎样匪夷所思的结果。\n\n首先我们回顾一下麦克斯韦方程组(高斯单位制):\n\n$$\\nabla\\cdot\\vec{E} = 4\\pi\\rho_e$$ (1)\n\n$$\\nabla\\times\\vec{E} = -\\frac{1}{c}\\frac{\\partial}{\\partial t}\\vec{B}$$ (2)\n\n$$\\nabla\\cdot\\vec{B} = 0$$ (3)\n\n$$\\nabla\\times\\vec{B} = \\frac{1}{c}(4\\pi\\vec{J_e}+\\frac{\\partial}{\\partialt}\\vec{E})$$ (4)\n\n考虑到静止点电荷q产生的电场为 $$\\vec{E} = \\frac{q \\vec{r}}{r^3}$$ ,作为对比,牛顿引力场强 $$\\vec{E_g} =-\\frac{G m \\vec{r}}{r^3}$$ ,因而对于引力场可以将上面(1)式改写成:\n\n$$\\nabla\\cdot\\vec{E_g} = -4\\pi G\\rho$$\n\n其中 $$\\rho$$ 是密度,因为引力场总是导致吸引相互作用,所以我们可以要求 $$\\rho\\geq0$$ 。\n\n然后我们将方程(2)(3)直接照抄过来,得到:\n\n$$\\nabla\\times\\vec{E_g} = -\\frac{1}{c}\\frac{\\partial\\vec{B_g}}{\\partial t}$$ ,$$\\nabla\\cdot\\vec{B_g} = 0$$\n\n对于(4)式我们要稍加注意,为了使流守恒方程 $$\\frac{\\partial}{\\partial t}\\rho +\\nabla\\cdot\\vec{J_{\\rho}}=0$$ 成立,,我们需要将(4)式改写成:\n\n$$\\nabla\\times\\vec{B_g} = \\frac{1}{c}(-4\\piG\\vec{J_{\\rho}}+\\frac{\\partial}{\\partial t}\\vec{E_g})$$\n\n最后,我们完全复刻带电粒子在电磁场中受到的洛伦兹力,将质量为m的粒子在引力场中运动时受到的力写做:\n\n$$\\vec{F_g} = m(\\vec{E_g} + \\frac{1}{c}\\vec{v}\\times\\vec{B_g})$$\n\n \n\n综上我们得到了引力场的“麦克斯韦方程组”,以及相应的物质运动方程:\n\n$$\\nabla\\cdot\\vec{E_g} = -4\\pi G\\rho$$ (5)\n\n$$\\nabla\\times\\vec{E_g} = -\\frac{1}{c}\\frac{\\partial\\vec{B_g}}{\\partial t}$$(6)\n\n$$\\nabla\\cdot\\vec{B_g} = 0$$ (7)\n\n$$\\nabla\\times\\vec{B_g} = \\frac{1}{c}(-4\\piG\\vec{J_{\\rho}}+\\frac{\\partial}{\\partial t}\\vec{E_g})$$ (8)\n\n$$\\vec{F_g} = m(\\vec{E_g} + \\frac{1}{c}\\vec{v}\\times\\vec{B_g})$$ (9)\n\n如果我们考察由上述方程组构造的能量守恒方程:\n\n$$\\int_{V}\\frac{\\partial}{\\partial t}w_g\\mathrm{d}V +\\oint_{\\Sigma}\\vec{S_g}\\cdot\\mathrm{d}\\vec{s} +\\int_{V}\\vec{f_g}\\cdot\\vec{v}\\mathrm{d}V = 0$$\n\n其中 $$w_g$$ 是引力场的能量密度, $$\\vec{S_g}$$则是引力场的能流密度矢量(对应于电磁场的波印廷矢量)。结合方程(6)(8)(9)可以得到:\n\n$$w_g = -\\frac{1}{8\\pi G}(E^2 + B^2)$$\n\n$$\\vec{S_g} = -\\frac{c}{4\\pi G}\\vec{E_g}\\times\\vec{B_g}$$\n\n这意味着引力场的能量密度竟然是一个负值,不仅如此,连能流密度矢量都与电磁场的波印廷矢量相反,这意味着一个变速运动的粒子不是往外辐射能量,而是往里吸收能量!\n\n \n\n另一方面注意到我们新构造的守恒流 $$J^{\\mu} = (c\\rho,\\vec{J_{\\rho}})$$,如果我们要求参与引力相互作用的质量就是惯性质量,那么$$J^{\\mu}$$ 将不是Lorentz四矢量,原因在于与电荷 $$Q = \\int\\rho_e\\mathrm{d}V$$ 不同,惯性质量 $$M = \\int \\rho\\mathrm{d}V$$并非是Lorentz不变的。故而,要使得新得到的“麦克斯韦”方程组是Lorentz不变的,我们需要假设参与引力相互作用的荷m是与惯性质量相互独立的,即引力质量$$\\ne$$ 惯性质量,而这将直接违反等效原理……\n\n \n\n综上,将麦克斯韦方程组直接套用在引力场中将会出现两件相当不妙的事情:\n\n(1) 我们得到的场的能量密度是负值,并且波印廷矢量方向与电磁场情形相反。\n\n(2) 如果想使得方程Lorentz不变,那么引力质量 $$\\ne$$ 惯性质量,违反等效原理。\n\n所以直接将电磁场的麦克斯韦方程组套用在引力场上是不合理的。", "task_type": {"major": ["问答"], "minor": ["知乎问答"]}, "domain": ["通用"], "metadata": "qid:453950954, aid:1852203872", "answer_from": "human", "human_verified": true, "copyright": "暂无版权及作者信息"} {"instruction": "为什么在剥煮鸡蛋的鸡蛋壳时,有的鸡蛋壳很容易就剥开了,有的则很难很干净利索地剥开?", "input": "", "output": "从鸡蛋的剖面角度来看,与蛋白紧密相连的结构是内壳膜,我们平时剥煮鸡蛋时所谓的“不好剥”,其实就是鸡蛋被煮熟后蛋白与内壳膜的连接仍旧紧密,因此剥壳时会连壳带蛋白一起被剥下。那么要想让鸡蛋变得“好剥”,就得想办法让蛋白与内壳膜分离。那使用什么方法可以做到呢?\n\n科学家曾研究过影响剥壳难易程度的诸多因素,发现在相同的条件下煮熟刚生产的鸡蛋和放置一段时间后的鸡蛋,刚生产的鸡蛋更加难剥。那放置的这段时间中鸡蛋发生了什么样的变化呢?通过检测蛋清的pH值,发现随着放置时间的增加,蛋清的pH值会稳步上升(由最初约7.5到最终约9.2),而且放置环境的温度对pH值的上升速率有影响,温度越高,蛋清pH值上升得越快(在4℃的放置环境下需要6天,蛋清的pH值从7.4上升至9.0;在24℃的放置环境下只需3天,蛋清的pH值就从7.7上升至9.2)。那是什么原因导致蛋清pH值的上升呢?一种解释是刚生产的鸡蛋中,蛋清里含有较多代谢产生的二氧化碳,故而pH值较低。并且此时蛋液填充相对饱满,使得蛋清与内壳膜接触紧密从而粘附力较大。随着放置时间的增加,鸡蛋内的二氧化碳逐渐扩散至鸡蛋外并且伴随部分水分的散失,蛋清的pH值升高,同时导致蛋液的填充不如之前饱满,蛋清与内壳膜的接触不再那么紧密,使得粘附力降低。粘附力降低了,煮熟后的鸡蛋自然就“好剥”了。\n\n到这里可能有的同学会问了,难道就必须要放个几天才能吃到“好剥”的鸡蛋吗?其实可能也没必要。因为我们已经知道增强蛋清的碱性就能在一定程度上使鸡蛋“好剥”,那么如果在煮鸡蛋的过程增大外界环境的pH值,或许也能起到差不多的效果。所以首先想到的是在煮鸡蛋的水中加一些小苏打,这样就能使水呈弱碱性,增大鸡蛋内的二氧化碳的扩散速率。\n\n在鸡蛋被煮熟之后,为了能尽快享用,我们一般会用冷水浸泡以快速降温,其实这步操作同时也有分离蛋白和内壳膜的功能,其原因可能源于蛋白和内壳膜的热膨胀系数不同。\n\n综上所述,想得到一个“好剥”的鸡蛋,可以把上述方法结合使用,说不定煮出来鸡蛋的壳都能不剥自落。", "task_type": {"major": ["问答"], "minor": ["知乎问答"]}, "domain": ["通用"], "metadata": "qid:20648577, aid:1851004202", "answer_from": "human", "human_verified": true, "copyright": "暂无版权及作者信息"} {"instruction": "如何从微观角度解释光的反射和折射?\n同样是固体,为什么玻璃能被光线穿过,而木板和铁板不能?为什么光线能穿透水和空气,但是会发生折射?由于(由原子、分子构成的)物质几乎是无所不在的,那是否任何到达人眼里的光线都是折射后的?也就是说,我们看到的只是物质经过折射后的位置,而非它们「本来」的位置。这些现象在微观(粒子)的层面上如何去理解?", "input": "", "output": "站在微观的角度,这三个现象从理论上来说都是光与原子中的电子发生相互作用的结果,并且这种相互作用可以完全用量子电动力学来描述。对于散射,从经典角度,在入射光的激励下原子会产生与入射光频率相同的感生电偶极矩,根据经典电动力学的理论,这个振动的电偶极矩将会向外辐射同频率的电磁波,这就是瑞利散射,它是一种弹性散射过程。与之相应的,还有非弹性的拉曼散射,这种散射发生在入射光的频率和分子某一本征振动模式的频率接近时,光子将与分子振动能级发生相互作用,产生能量交换使得散射光子的能量可以降低或者升高$$h\\Delta\\nu$$。当然,当入射光的频率足够高时,还能直接与原子的外层电子发生弹性散射(对,就是大家熟悉的高中物理选修3-5里的康普顿散射)。\n\n反射与折射则是另外两种生活中常见的光学现象。这两个现象利用宏观上的麦克斯韦方程组以及界面处的边界条件可以完美地得到解释。当然,另一方面我们知道介质是由大量原子构成的,并且光子会与介质中的原子发生电磁相互作用,那么一个关键的问题就是这种微观上的电磁相互作用是如何导致光在介质中的行进速度变慢的?(或者说介质有折射率的呢?)在尝试解释这个问题之前,我必须告诉大家,在量子的世界里,粒子的传播没有经典世界里那么“拘束”,事实上,它们可以走任何路径,但每个路径会伴随一个复数振幅,而我们观测到的结果是所有路径的复振幅求和的结果。如此一来,真空中的光子不必沿着直线传播,只不过那些不是直线的路径的复振幅振荡地非常剧烈,求和时相互之间抵消地非常厉害,以至于只有直线与其附近的路径保留了下来,这就是为何我们观测到真空中光子走直线的原因。当光子在透明介质里传播时,它除了自由传播还可能与介质中的原子发生相互作用即被散射。下面我们考察一下透过介质的光子究竟发生了什么事情?首先有一个振幅相应于光子直线穿过介质,没有发生任何散射,这个振幅将是最主要的,故而在复平面上,它的长度最长。但是,光子还有其它的路径到达介质下面的探测器B:光子可能到达X1并散射出一个新光子传播到B处,同样的事情也能在X2处发生等等。因为这些路径都包含介质内一个电子散射一个光子的过程,故而它们的振幅长度是相同的,另一方面这些振幅都指向同一个方向,因为只经历一次散射的这些路径的长度都是相同的。这些小振幅与无散射直接到B的主振幅的夹角成90°(为何是90°?这个问题并不显然,当然我们可以“曲线救国”,说:“是因为我们考虑的是透明材料,介质不吸收光子,于是小振幅与主振幅的夹角必须为90°!”),这些小振幅与主振幅相加,结果是最终振幅的长度与主振幅一样,但在复平面上额外多旋转了一个角度,这意味着光经过透明介质时,强度没有改变,但却积累了额外的相位!光在通过介质时最终振幅额外旋转的程度可以被称为该介质的“折射率”。\n\n好了,我们已经从微观上知道折射率是如何产生的了,之后便可以用我们熟知的惠更斯原理或者费马原理得到正确的折射定律。事实上,即便是从路径积分的角度考虑,由于不同路径之间的复数振幅的幅角差异巨大,使得求和时这些振幅强烈地互相抵消,最终会只剩下最短路径的贡献(这是因为最短路径附近的路劲的振幅幅角变化不大,大体都指向同一个方向,故而只有它们在求和时没有抵消),因而这个结果与费马原理符合的很好!\n\n类似地,反射现象也能从这样的微观原理去理解,更多的细节还是请参阅大名鼎鼎的物理学家费曼所写的《QED:光和物质的奇妙理论》吧,全书没有一个公式,靠一堆箭头解释了量子电动力学。这样神奇的书我想也只有费曼能写出来了Orz。\n\n至于题目里提到的单色光和非单色光,实际上,非单色光就是一些不同频率的单色光的叠加而已,本质并没有什么不同。", "task_type": {"major": ["问答"], "minor": ["知乎问答"]}, "domain": ["通用"], "metadata": "qid:19766271, aid:1656477607", "answer_from": "human", "human_verified": true, "copyright": "暂无版权及作者信息"} {"instruction": "距离多远时超新星爆炸才能把手里土豆片烤熟?", "input": "", "output": "超新星是部分恒星在末期发生剧烈爆炸的阶段,可以释放大量能量。它的持续时间约几周至几年,宋朝观测到的一次超新星持续约八个月,这里我们取典型值$$t=10^7s$$ (约115.7天);释放的能量典型值约 $$E=10^{44}J$$ 。\n\n土豆变熟是土豆内淀粉糊化。糊化温度在50~80℃不等,100℃水煮可在5分钟内使土豆变熟。因此我们取能够将土豆烤熟的极限温度在87℃(360K)。\n\n下面我们做几个假设:1、超新星释放能量在时间上均匀,空间上球对称;2、土豆片与外界热交换仅有热辐射一种形式;3、土豆片是一个绝对黑体,满足斯忒藩-玻耳兹曼定律;4、土豆片正对超新星。\n\n这样我们可以建立一个方程:距爆发的超新星R处的能流密度(单位面积上的功率) $$j_1$$ 等于87℃土豆片发出的黑体辐射能流密度 $$j_2$$的二倍(毕竟土豆片有两个面嘛),也就是 $$\\frac{E}{4{\\pi}R^2\\cdot t}=2\\times\\sigma T^4$$ ,σ是斯忒藩-玻耳兹曼常数。解出 $$R\\approx 2.04\\times 10^{16}m$$ ,这个距离约等于2.2光年。\n\n这些估算是在我们做出的假设基础上进行的,实际结果可能是一个很大的范围。", "task_type": {"major": ["问答"], "minor": ["知乎问答"]}, "domain": ["通用"], "metadata": "qid:613419607, aid:3129859037", "answer_from": "human", "human_verified": true, "copyright": "暂无版权及作者信息"} {"instruction": "为什么脉冲星的自转周期非常稳定?", "input": "", "output": "脉冲星是中子星的一种,它有很强的磁场,并且自转周期很短,目前发现的从1.4毫秒(PSR-J1748-2446)到11秒(PSR-J1841-0456)不等。没错,就在你读完这句话的时候,脉冲星已经转了好几圈了。它的自转速度之所以这么快,是因为在恒星坍缩成中子星的时候半径大幅减小,转动惯量也随之减小,为了保持角动量守恒,角速度就得相应地增大。\n\n脉冲星的自转轴和磁轴一般并不重合,就像地磁场的磁极与地理意义上的南北极并不重合一样。脉冲星自转的时候带着磁轴旋转,辐射出来的电磁波也随之周期性地扫射,在地球上看来就会收到短而稳定的脉冲信号,故名脉冲星。\n\n回到问题,脉冲星的自转周期很稳定其实是相对于人类的时间尺度而言的。毕竟从1967年休伊什和贝尔第一次发现脉冲星到现在,也不过56年。在上千年的人类文明史中,我们都认为地球的自转周期非常稳定。但古生物学表明,地球在5亿年前的寒武纪一天是20.8小时,这说明地球自转变慢了。我们现在认为是月球公转拖慢了地球自转。\n\n对于脉冲星而言,它时时刻刻在损失能量辐射电磁波,自转周期也应该会有变化。我们说它“非常稳定”,其实多少也有点“朝菌不知晦朔,蟪蛄不知春秋”了。而且脉冲星也会受到周围天体的引力影响,例如获得1993年诺贝尔物理学奖的泰勒赫尔斯,就是发现了脉冲星的轨道有一定波动,从而发现了第一个脉冲双星PSR1913+16。[2,3]", "task_type": {"major": ["问答"], "minor": ["知乎问答"]}, "domain": ["通用"], "metadata": "qid:610969697, aid:3108573814", "answer_from": "human", "human_verified": true, "copyright": "暂无版权及作者信息"} {"instruction": "物理学家李方华院士去世,她一生的成就有哪些?", "input": "", "output": "李方华(1932.1.6—2020.1.24),凝聚态物理学家,电子显微学家。中国科学院院士,第三世界科学院院士。中国电子衍射及高分辨电子显微学的先驱者之一。主要从事衍射物理、高分辨电子显微学和晶体学研究。在高分辨电子显微像的衬度理论和图像处理理论与方法研究、微小晶体结构测定、原子分辨率晶体缺陷测定,以及准晶的研究中做出了重要贡献。作为一名共产党员,李方华先生将赤诚爱国的一生奉献给了祖国的科研事业,将严谨治学的一生奉献给了中国的物理学发展。\n\n李方华从1960年起,开始独立从事科研工作,将一台旧的电子显微镜改装成电子衍射仪,测定了二十三烷醇晶体中的碳和氢原子的位置,这是我国最早的单晶体衍射结构分析和测定晶体中氢原子位置的工作,填补了该领域空白的同时,还提出了一种矫正电子衍射动力学效应的方法。\n\n二十世纪七十年代中期,她与王荫君合作,研究钆钴非晶磁性薄膜的结构与磁性的关系,负责测定非晶体的径向分布函数。该工作填补了国内非晶体电子衍射结构分析的空白,并发展了重量悬挂原子对的简便测定方法。此项合作研究成果获得了中国科学院科技成果奖三等奖。\n\n李方华先生是最早向国内同行介绍高分辨电子显微学发展动态的人,并在中国科学院物理研究所建立了我国最早的高分辨电子显微学研究组,开展高温超导体、半导体、合金、氧化物和矿物等材料的点阵像研究,发现了许多新的结构现象。这些工作于1984年获得中国科学院科技成果奖二等奖。\n\n1982年,李方华应邀到日本大阪大学应用物理系任访问学者。借助那里当时的先进实验和计算条件,她完成了对我国发现的新矿物——氟碳铈钡矿的晶体结构的测定。利用在日期间积累的大量实验和计算经验,回国后她率领自己的研究组使我国在高分辨电子显微学实验和计算方面研究工作迈进了一大步。此外,她认识到传统的尝试法在测定较复杂晶体时的局限性,在总结出了显微像衬度规律基础上提出了显微衬度理论,该理论成为后来她建立电子晶体学图像处理技术的理论依据。\n\n二十世纪八九十年代,她的主要贡献是建立测定微小晶体结构的新方法、建立研究原子分辨率晶体缺陷的新技术、发展高分辨电子显微像的衬度理论、准晶体和晶体之间的关系的实验和理论研究等。她所建立的电子晶体学图像处理技术,已成功地用来测定了多个未知晶体结构,其中多为高温超导体的公度或无公度调制结构。她所建立的场发射高分辨电子显微图像处理技术,已应用于半导体晶体中不同类型位错核心的研究。在有关高温超导体的早期研究工作中,她利用目前电子衍射和高分辨电子显微学方法,进行新材料的结构研究,特别是在认定铋系超导体有无公度调制结构方面起了关键作用。\n\n以李方华先生为第一获奖人的奖项有:中国科学院自然科学奖一等奖1次(1992)、二等奖3次(1984、1989、1991)、中国物理学会叶企孙物理奖(1991);以个人名义获奖奖项:中国电子显微学会桥本初次郎奖(1992)和钱临照奖(1993),以及联合国教科文组织颁发的2003年欧莱雅-联合国教科文组织世界杰出女科学家成就奖。\n\n \n\n主要论著:\n\n【1】李方华. 电子衍射强度与结构振幅之间的经验关系及其在结构分析中的应用[J]. 物理学报(11):49-54.\n\n【2】李方华, 王荫君, 高俊杰, et al. 钆钴非晶合金薄膜的电子衍射研究[J]. 物理学报, 1980(02):63-68.\n\n【3】F. H. Li, H. Hashimoto. Use of dynamical scattering in the structuredetermination of a minute fluorocarbonate mineral cebaite Ba 3 Ce 2 (CO 3 ) 5F 2 by high-resolution electron microscopy[J]. Acta Crystallographica, 1984,40(5):454-461. 【4】LI, FH, TANG, D. PSEUDO-WEAK-PHASE-OBJECT APPROXIMATION IN HIGH-RESOLUTIONELECTRON-MICROSCOPY. 1. THEORY[J].\n\n【5】D. Tang, C. M. Teng, J. Zou,等. Pseudo-weak-phase-object approximation inhigh-resolution electron microscopy. II. Feasibility of directly observing Li+[J]. Acta Crystallographica, 2010, 42(4):340-342.\n\n【6】李方华. 用高分辨电子显微镜测定晶体结构[J]. 物理学报(03):9-14.\n\n【7】李方华, 范海福. 用Sayre等式复原高分辨电子显微象[J]. 物理学报, 1979(2).\n\n【8】Hai-fu Fan, Zi-yang Zhong, Chao-de Zheng,等. Image processing in high-resolution electron microscopy using the direct method.I.Phase extension [J].Acta Crystallographica, 2014, 41(2):163-165.\n\n【9】None. Image processing in high-resolution electron microscopy using thedirect method. II. Image deconvolution[J]. Acta Crystallographica,42(5):353-356.\n\n【10】Hu J J , Li F H . Maximum entropy image deconvolution in high resolutionelectron microscopy[J]. Ultramicroscopy, 1991, 35(3-4):339-350.\n\n【11】D. X , Huang, and, et al. Multiple solution in maximum entropydeconvolution of high resolution electron microscope images[J].Ultramicroscopy, 1996.\n\n【12】H, Jiang, and, et al. Electron crystallographic study ofBi2(Sr0.9La0.1)2CoOy[J].\n\n【13】Lu B, Li F H, Wan Z H, et al. Electron crystallographic study of Bi 4 (Sr0. 7 5 La 0. 2 5 ) 8 Cu 5 O y structure[J]. 1997, 70(1-2):13-22.\n\n【14】J Liu, F H Li, Z H Wan,等. Electron crystallographic image-processinginvestigation and superstructure determination for(Pb0.5Sr0.3Cu0.2)Sr-2(Ca0.6Sr0.4)Cu2Oy[J]. Acta Crystallographica, 2001, 57(Pt5):540-547.\n\n【15】G. Z. Pan, F. H. Li. An Identical Quasicrystal-Structure Model for T2-(AI6CuLi3) from Two Different Six-Dimensional Crystals[J]. Physica StatusSolidi, 2006, 182(1):39-49.\n\n【16】Li, F. H . Image processing based on the combination of high-resolutionelectron microscopy and electron diffraction[J]. Microscopy Research andTechnique, 1998, 40(2):86-100.\n\n【17】F. H. Li. Crystallographic image processing approach to crystal structuredetermination [J]. Journal of Microscopy, 2002, 190(1-2):249-261.\n\n【18】Li F H . High-resolution electron microscopy and its applications[J].Journal of Electron Microscopy Technique, 1987, 7(4):237-254.", "task_type": {"major": ["问答"], "minor": ["知乎问答"]}, "domain": ["通用"], "metadata": "qid:368073199, aid:987962469", "answer_from": "human", "human_verified": true, "copyright": "暂无版权及作者信息"} {"instruction": "为什么坐车的时候闭上眼睛会感觉车子在后退,特别是坐大巴的时候,就睡觉,睡不着的时候就不自觉去感受,虽然不明显,但是还是偏向向后走的感觉,偶尔是没有向前向后的感觉,睁开眼就立马正常了,看来每当分不清楚什么的时候,眼睛真是占了很重要的作用。", "input": "", "output": "这个问题牵涉到物理学中一个非常重要的原理,这个原理导致了相对论的产生。那就是大名鼎鼎的相对性原理,这个原理的大意是:在任一惯性系中观察到的物理规律是完全一样的。也就是说你无法通过在惯性中做实验来区分两个相互运动的惯性系。基于这个原理,如果车是完全匀速运动的,你闭上眼睛是无法判断车在前进还是后退的。这里有一个词需要解释:什么叫无法判断?众所周知,人的感觉是不太准确的,即便是在匀速直线运动的车上的乘客,他闭眼睛后也可能感觉车在朝某个方向运动,消除由于人的感觉不准确造成的影响的方法是,让乘客闭眼后随机转动一个角度,然后让他指出他认为列车行进的方向,经过多次试验如果他指出的方向均匀分布在各个方向上我们就可以认为,他无法判断车朝什么方向运动。\n\n在实际生活中根本不存在匀速直线运动的车,每辆车多少都有一点加速度,推背感就是人对加速度的一种常见的感受。人即使闭上眼睛也能感知加速度,但是加速度方向和行进方向并没有直接关系,比如前运动过程中加速人可以体会到推背感,车在向后运动过程中减速人同样可以体会到推背感。不过,人脑会下意识将加速度方向和某个行进方向联系起来,比如,说到推背感我们一般都认为是汽车向前起步加速时的感觉,当你蒙上眼睛后,缺少了外界物体的参考,这时你感到了推背感,你的大脑会下意识认为车在向前加速行进,殊不知,此时也可能是向后减速行进,这时你的判断就会和车的实际运行情况相反。\n\n综上,之所以出现题主说的情况有两个原因:一是人的感觉不准确造成的。另一种车存在加速度造成的。", "task_type": {"major": ["问答"], "minor": ["知乎问答"]}, "domain": ["通用"], "metadata": "qid:35230216, aid:645867363", "answer_from": "human", "human_verified": true, "copyright": "暂无版权及作者信息"} {"instruction": "用钳子夹冰糖为什么会发光?", "input": "", "output": "冰糖的摩擦荧光确有其事。想见证奇迹的童鞋可以做一个小实验:找来一个透明的内部干燥(一定要干燥,越干燥现象越明显)的矿泉水瓶,用其四分之一的空间装上大块冰糖。在一个月黑风高的夜晚,拉上窗帘,关上灯,让室内伸手不见五指,然后迅速地摇晃塑料瓶,你就可以看到瓶中的冰糖一下下地发着蓝紫色的闪光。摇得越快,现象越明显!\n\n你可能不知道,摩擦荧光(Triboluminescece)的研究历史已经有几百年了,早在17世纪就有人发现摩擦糖块会发出亮光。其机理在Halliday的《FundamentalsofPhysics》里面有所叙述。由于冰糖晶体的非对称性,冰糖在断裂过程中断面会带上正负电荷,相当于把震动摩擦的机械能转化为了电势能。而电荷中和的放电过程激发了空气中的氮分子,氮分子退激发将能量以荧光形式放出。相似机理引起摩擦发光的晶体还有LiF、NaCl、SiC等。\n\n虽然多种晶体都有相似的发光现象,但是背后蕴含的机理问题有很多。比如,晶体的压电效应、扭曲和位错都能引起发光;还有些晶体不像冰糖这样靠激发氮分子来发光,而是由晶体本身被激发所致。除此之外,摩擦荧光也不仅限于非对称晶体,在某些对称晶体上也能观察到该现象。这些问题都有待人们去研究。这么看来,一个不起眼的小现象说不定蕴含着很多大学问呢!", "task_type": {"major": ["问答"], "minor": ["知乎问答"]}, "domain": ["通用"], "metadata": "qid:306550630, aid:622296913", "answer_from": "human", "human_verified": true, "copyright": "暂无版权及作者信息"} {"instruction": "大名鼎鼎的芝诺悖论该如何破解呢?\n乌龟说:“好,那我们假设一下。你离我不有100米,你的速度是我的10倍。现在你来追我了,但当你跑到我现在这个位置,也就是跑了 100 米的时候,我也已经又向前跑了10米。当你再追到这个位置的时候,我又向前跑了1米,\n你再追1米,我又跑了 1/10 米……总之, 你只能无限地接近我,但你永远也不能追上我。", "input": "", "output": "哪有什么无限接近,关键问题就是,无穷个大于零的数相加并不是无穷啊。\n\n芝诺悖论主要问题就在于当时人们还没有建立关于无穷的概念,没有意识到一个无穷级数求和是有可能收敛于一个有限值的,因此才会囿于所谓的无限接近又永远追不上。\n\n我们来看距离。开跑时乌龟在100米处,阿基里斯在0米处,经过Δt后,乌龟在110米处,阿基里斯在100米处……按这样前进,乌龟的距离应该是100+10+1+0.1……最后的坐标是111.1111……这其实就是阿基里斯追上乌龟的坐标。我们从时间上也能得到同样的结果,乌龟向前前进十米用时Δt,同时阿基里斯向前100米,第二段路程乌龟向前前进1米,阿基里斯前进10米,用时是0.1Δt,这样持续下去,阿基里斯追上乌龟一共用时是Δt+0.1Δt+0.01Δt……其实最后就是1.1111……Δt嘛,也就是说在这一时刻,阿基里斯就可以追上乌龟。\n\n其实这个问题抽象成数学语言就是求解方程:100+vt=10vt,对于确定的v,这个方程很明显是有解的。用级数的语言来看,这个问题的关键就在于级数$$\\frac{1}{10^n} \\quad(n=0,1,2 \\ldots \\ldots)$$,其求和是不是收敛的,很明显,这个问题没有超出我们的认知,芝诺就是欺负阿基里斯数学没学好。", "task_type": {"major": ["问答"], "minor": ["知乎问答"]}, "domain": ["通用"], "metadata": "qid:573332602, aid:2935762938", "answer_from": "human", "human_verified": true, "copyright": "暂无版权及作者信息"} {"instruction": "从龙头下落的水为什么越来越细?", "input": "", "output": "因为水龙头流出的水流量是一定的,但由于重力,水处于自由落体状态,因此越往下,水下落的速度越快,单位体积的水在单位时间内走过的路程越长,因此横截面积越小。\n\n我们用高中物理知识推导一下,不过会用到一点点微积分。\n\n首先,水龙头流出的水流量是一个常量,这是由自来水公司决定的。流量Q就是单位时间内由水龙头流出的水,这里我们用体积定义:\n\n$$ Q=\\frac{d V}{d t} $$\n\n在一个极小的时间 $$dt$$ 内,我们可以近似认为水柱是一个圆柱体,其上下表面积不变,这样,水的体积就是:\n\n$$ d V=A(t) d l $$\n\n$$A$$ 是 $$t$$ 时刻水柱的上下表面积, $$l$$是水柱的高,由于水柱是自由落体运动,在不考虑空气阻力的情况下,我们可以根据运动学公式写出水柱的高,这样上式就写成:\n\n$$ d V=A(t) d l=A(t) d\\left[\\int v(t) d t\\right] $$\n\n其中 $$ v(t)=v_0+g t $$ , $$v_0$$ 是水由水龙头流出时的初速度,积分以后可得:\n\n$$dV=A(t) d\\left[v_0 t+\\frac{1}{2} g t^2\\right]=A(t)\\left[v_0 d t+g t dt\\right] $$\n\n联立第一个式子 $$ Q=\\frac{d V}{d t} $$ ,我们就可以得到水流的横截面积与时间的关系式:\n\n$$A(t)=\\frac{Q d t}{v_0 d t+g t d t}=\\frac{Q}{v_0+g t} $$\n\n由这个式子可以看出,横截面积与时间呈反比,因此水流越往下越细。", "task_type": {"major": ["问答"], "minor": ["知乎问答"]}, "domain": ["通用"], "metadata": "qid:585921752, aid:2933644679", "answer_from": "human", "human_verified": true, "copyright": "暂无版权及作者信息"} {"instruction": "物理学四大神兽是什么?如何看待各神兽背后的哲理?\n物理学上有四大神兽:薛定谔的猫、芝诺的乌龟、麦克斯韦的妖精、拉普拉斯的恶魔。这四大神兽都是难以理解,且有着深刻内涵的假设。", "input": "", "output": "物理学四大神兽是芝诺龟,拉普拉斯兽,麦克斯韦妖,薛定谔的猫。它们不是真实存在的动物,而是科学家们假想出来用于佐证科学理论的思想实验。篇幅有限简要介绍。\n\n芝诺龟(Zeno'sparadox)是一个悖论:假定人的速度是龟的10倍,现在让人去追100米远的龟,人到达龟的起点时龟已经向前走了10米,人再追10米龟也前进1米,还是在人的前面。如此无限循环下去,龟永远不会被人追上,但在现实中这是不可能的,这就是著名的芝诺悖论。但现在我们知道,无穷个数相加不一定等于无穷。芝诺悖论里的这个无穷数列是收敛的,芝诺的乌龟并非永远追不上。\n\n拉普拉斯兽(Démon deLaplace)也叫拉普拉斯妖,它知道宇宙中每个原子确切的位置和动量。拉普拉斯相信决定论,他认为掌握了这些信息就能计算出整个宇宙的过去和未来,但热力学第二定律和不确定性原理告诉我们,这样的恶魔是不可能存在的。\n\n麦克斯韦妖(Maxwell'sdemon)是麦克斯韦假想的,能探测并控制单个分子的运动的妖。如果一个绝热容器被分成相等的两格,中间的麦克斯韦妖只允许冷原子通过,就可以0消耗在两侧制造出温差并对外做功,从而突破热力学第二定律制造出永动机。但实际上,就算麦克斯韦妖真的存在,它处理信息的这个过程也是热力学不可逆的,必然伴随着更高的熵增,仍然无法突破热力学第二定律,也不可能造出永动机。\n\n关于薛定谔的猫(Schrödinger’sCat)的讨论至今还在继续,在四大里的知名度也是最高。它本是薛定谔为质疑量子力学所制造的武器:量子力学认为,微观粒子可以处于叠加态,但如果把猫和放射性原子放在黑箱里,且原子衰变会导致猫死亡,原子处于衰变和未衰变的叠加态就会导致猫也处于既活又死的叠加态。这一实验将微观的量子态映射到宏观的猫,非常巧妙。\n\n最后,四大神兽这个名号怎么成名的这个问题,目前并没有统一的答案。虽然除了薛定谔的猫外都已经被证伪,但这四个思想实验推动了科学的发展,在科学史上有重要意义。", "task_type": {"major": ["问答"], "minor": ["知乎问答"]}, "domain": ["通用"], "metadata": "qid:54842159, aid:2803863692", "answer_from": "human", "human_verified": true, "copyright": "暂无版权及作者信息"} {"instruction": "在温度极低的地方尿尿会不会形成一根冰冻的尿柱?", "input": "", "output": "考虑到尿液比热容较大,在空中滞留时间较短,空气传热能力较低等因素,这几乎是不可能的。在这里无法很严格地计算,但是可以给出一个很粗略的模型来估计所需温度的量级。首先,我们可以不考虑尿液液柱内部的传热细节,假定“冻住”的条件就是发生在液柱表面的传热可以在落地前的时间内将对应的一段液柱降低至其凝固点。尿液含有尿素、尿酸、钠盐等,根据稀溶液浓度升高凝固点降低的原理可知其凝固点应当略低于0°C,有的说法是$$-1.3~-2.2^{\\circ}C $$ (不妨取 $$-2^{\\circ}C $$ )。而尿液初始温度是略高于体温 $$37 ^{\\circ}C$$ (不妨取 $$38^{\\circ}C $$ )的,因此粗略认为需要降低 $$\\Delta T=40^{\\circ}C $$ (已取绝对值,下同)。\n\n考虑某一段近似为圆柱体的液柱,参考输尿管的直径近似取半径为 $$R =2mm $$ ,其体积为 $$V = \\pi R^2h$$\n\n假定尿液的密度为水的密度 $$\\rho =1g/cm^3$$ ,其质量为 $$m=\\rho V = \\pi R^2h\\rho$$\n\n再假定其比热容为水的比热容 $$C = 4200J/(kg \\cdot ^{\\circ}C)$$ ,忽略比热容随温度的变化。因此下降一定温度所需要的传热为$$Q=Cm\\Delta T = C\\pi R^2 h \\rho\\Delta T$$\n\n假定尿液没有竖直初速度,液柱在竖直方向上的分运动为自由落体运动。每一小段液柱在落地前所经历的时间为 $$t=\\sqrt{\\frac{2H}{g}}$$\n\n简化起见,取 $$H = 0.98m$$ (略高于人的一半),重力加速度取 $$g=9.8m/s^2$$。流体(这里为空气)流过液体表面时发生的传热一般认为是对流传热,其单位时间流过单位表面积的热量大小,即热流密度为 $$q=\\alpha(T_u-T_e)$$\n\n其中 $$\\alpha$$ 为表面传热系数,对于空气和水的界面,由于空气的表面换热系数很低,取最高大概为 $$\\alpha =100W/(m^2{\\cdot}K)$$ (此处参考资料,真实情况应该更低)。上式中 $$T_u$$ 和 $$T_e$$分别代表尿液和环境的温度,由于尿液的温度是在不断降低的,因此正确求解其实需要用到微积分。这里为了简化,假定 $$q$$ 恒定为 $$T_u =18^{\\circ}C$$ (即初末平均近似温度)时的数值。\n\n因此,对于上述液柱侧面,在落地前完成的传热量为 $$Q'=\\alpha (T_u-T_e){\\cdot}2\\pi R h{\\cdot}t$$于是临界环境温度 $$T_{es}$$ 应满足 $$Q=Q'$$ 。即有 $$T_{es}=T_u-\\frac{C\\pi R^2h\\rho \\DeltaT}{\\alpha {\\cdot}2\\pi Rh\\cdot t }$$\n\n代入上述的各个物理量近似值,可以得到 $$T_{es}\\approx -3740 ^\\circ C $$\n\n这显然是一个无法达到的温度,因为宇宙中最低温也不过是大约 $$-273 ^\\circ C $$。究其原因,一方面是尿液的比热容很大,另一方面是空气的传热效率较低。假定我们可以增加空气的传热效率,令其可以媲美导热性能优良的固体界面,此时可以取水为流体时传热的极限值$$\\alpha = 5000W/(m^2 \\cdot K)$$ ,此时计算的结果为 $$T_{es}\\approx -57 ^\\circC $$\n\n可见,即使是空气拥有远超其传热极限的传热性能,也要南极、北极等地的极低温才可能实现。\n\n不过上述计算只考虑了传热面为圆柱侧面这样粗略的模型。实际尿尿的时候液体会呈现复杂的形状,最终张开的比表面积远大于圆柱,这时理论上的临界温度可能会有所升高。\n\n参考资料:\n\n杨世铭, 陶文铨. 传热学(第4版) [Heat Transfer][M]. 高等教育出版社, 2006.\n\neveryday chemistry - What would explain urine that resists freezing?\n\n输尿管-维基百科\n\nConvective Heat Transfer Coefficients Table Chart", "task_type": {"major": ["问答"], "minor": ["知乎问答"]}, "domain": ["通用"], "metadata": "qid:37179993, aid:2794903827", "answer_from": "human", "human_verified": true, "copyright": "暂无版权及作者信息"} {"instruction": "科学家是如何测得地球的质量约为 5.9742 × 10^24kg 的?", "input": "", "output": "题主提问的地球质量是1976年给出的推荐值 $$(5.9742\\pm0.0036)\\times10^{24}kg$$。截至2021年对地球质量的最佳估计为 $$M=5.9722\\times10^{24}$$ 千克,绝对不确定度为$$6\\times10^{20}$$ 千克,相对不确定度 $$10^{-4}$$ ,精确度有所提高。对应密度约为 $$5.515g/cm^3$$ 。\n\n科学家对地球质量的估算是逐步精确的,估算方法也随着理论和实验技术的进步而逐渐演化。早期科学家主要通过估算地球的体积和密度来估算地球质量,而对地球体积的估算由于纬度原因使得低估了地球半径,导致体积比正确值小约三分之一。艾萨克·牛顿(IsaacNewton)估计地球的密度是水的密度的五到六倍,与实际地球的平均密度非常接近,但由于体积的低估了约30%,得到地球质量约为$$(4.2\\pm0.5)\\times10^{24}$$ 千克。\n\n18世纪,牛顿万有引力定律的提出使得通过万有引力常数来间接估算地球质量成为可能。根据万有引力和牛顿第二定律, $$F=GmM/r^2=mg$$ ,$$G$$ 为万有引力常数, $$M$$ 为地球质量, $$r$$ 为地球半径, $$m$$ 为地球上物体质量, $$g $$为引力加速度。如果知道了重力加速度 $$g$$ ,地球的半径 $$r$$ ,和万有引力常数 $$G$$ ,就可以估算得到地球的质量 $$M=gr^2/G$$。其中引力加速度 $$g$$ 可以通过物体的自由落体运动进行测试, $$\\frac{1}{2}gt^2=h$$,可以在固定高度以初速度为零使物体自由下落,根据所用的时间即可得到重力加速度。万有引力常数可以通过卡文迪许扭称实验得到。据此,卡文迪许给出了较为精确的地球质量$$5.965\\times10^{24}$$ kg,对应密度 $$5.45g/cm^3$$ ,低于标准值仅1%。\n\n在此期间,科学家也尝试了许多方法来确定地球的平均密度,但后来发现高精度的重复进行卡文迪许实验的效率更高,地球质量标准值的不确定性主要取决于引力常数G测定的不确定性。到19世纪90年代,相对不确定性降低到约0.2%,直到目前,地球质量标准值的相对不确定性可以达到$$10^{-4}$$ 。", "task_type": {"major": ["问答"], "minor": ["知乎问答"]}, "domain": ["通用"], "metadata": "qid:567475715, aid:2764808597", "answer_from": "human", "human_verified": true, "copyright": "暂无版权及作者信息"} {"instruction": "分子永不停息做无规则运动的能量从哪里来?", "input": "", "output": "宏观物质的内能实际上是其内部分子的能量,而在微观世界,只能是分子的动能和势能,对于理想气体或较为稀薄的气体则只有动能,这部分动能就是分子无规则运动的能量。\n\n分子的运动可以分为三个部分,分别是平动、转动和振动。单原子分子只有平动,双原子分子只有平动和振动,多原子分子存在以上三种运动。分子可以通过碰撞将动能分配到平动、转动和振动上。根据能量均分定理,在温度为$$T$$ 的平衡态下,分子热运动的每个自由度都对应 $$\\frac{1}{2}kT$$的平均动能。所谓自由度是指确定一个系统的状态或位形的独立变量的个数。例如一个单原子分子,确定其位置需要x,y,z三个独立变量,则它的自由度就是3,如果是双原子分子,则需要加入两原子连线的方向(2个自由度)和原子间距离(1个自由度),共六个自由度。\n\n在室温下,并不是分子的所有自由度都起作用。微观上,分子的振动和转动都要用量子力学描述,振动和转动的能量只能取分立的值。通常转动能量间隔约为$$10^{-5}\\sim10^{-3}eV$$ ,而振动能量间隔约为 $$10^{-2}\\sim10^{-1}eV$$,室温下分子平动的能量可以很好地激发转动能级,但几乎不能激发振动能级,通常将此称为振动自由度的“冻结”。\n\n分子间碰撞一般只存在以上三种能量的转化,但其他形式的能量也可能参与进来。原子的热运动速度足够高时,原子间碰撞可能导致原子的激发或原子间电荷的交换,这个过程中原子的内部能级或者说是电子的势能参与进来。弗兰克-赫兹实验就利用了电子和汞原子发生非弹性碰撞的动能损失的特征验证了原子能级的存在。", "task_type": {"major": ["问答"], "minor": ["知乎问答"]}, "domain": ["通用"], "metadata": "qid:291115538, aid:2744481799", "answer_from": "human", "human_verified": true, "copyright": "暂无版权及作者信息"} {"instruction": "同样由分子构成,为什么玻璃能透光,木板之类的东西不透光?", "input": "", "output": "材料是否透光的问题同时涉及宏观和微观尺度的影响。微观层面主要是解释光为什么会被吸收、吸收的选择性和材料吸收光后如何反馈。宏观层面则表现出吸收、反射、折射、散射等具体现象,且宏观结构也在影响光被消耗的比例。\n\n简单来讲,不同组成或者结构的相的交界面是产生光与材料作用的宏观现象的主要部位。在每一个界面处,光都涉及从某一种介质运动到另一种介质中,此时光作为电磁波,其电场会与材料中原子的核外电子产生作用,即发生电子极化。电子极化会影响光子的能量,体现为光子运动速度的改变,这便是折射产生的原因。\n\n此外,电子可以吸收特定频率的光子跃迁到高能级,当其从高能级跃迁回基态时会重新释放光子。重新释放的光子如果返回到原介质,就体现为反射光。而介质中的微小的颗粒、气孔、杂物等则会诱发散射现象。一方面,吸收和散射都会消耗光的能量,随着入射深度的增加,光强按照Bouguer定律衰减\n\n$$I=I_0e^{-(\\alpha+S)x}$$\n\n其中 $$\\alpha$$ 和 $$S$$ 分别是吸收系数和散射系数。另一方面,折射率相差较大的界面处会由于反射带来较大的能量损失,损失比例$$R=(\\frac{n_{21}-1}{n_{21}+1})^2 ,n_{21}=\\frac{n_2}{n_1}$$ 称为反射系数。\n\n首先从宏观对比。玻璃是以非晶态存在的,主要成分是SiO₂以及Na₂O,CaO等氧化物,其主要特征就是原子的排布无规则导致的统计上的均匀性。这一性质使得反射等现象主要只发生在玻璃两侧与空气的界面处,而内部则可以看作完整的均匀介质。木材则含有很多细小的孔隙,而空气与木材本身折射率相差较大,因此由于反射等原因光在界面处会被严重消耗。\n\n接下来从组分的吸光性质对比。前面提到了电子可以吸收一定的光子跃迁,然而可以吸收什么频率的光子取决于材料是否有特定的能极。玻璃的能带具有较宽的带隙,体现为可见光波段的电子都没有足够的能量将玻璃价带中的电子激发到禁带以上的空轨道,阻止了可见光范围的吸收。而木材中含有的木质素,是一类复杂的有机聚合物,其中含有丰富的官能团结构。含有共轭羰基、芳香环、碳碳双键等的一些官能团在可见光范围有较强的吸收。综上,光能否通过原子或者分子,主要看能否被吸收。", "task_type": {"major": ["问答"], "minor": ["知乎问答"]}, "domain": ["通用"], "metadata": "qid:544791011, aid:2587876552", "answer_from": "human", "human_verified": true, "copyright": "暂无版权及作者信息"} {"instruction": "光的折射、透镜的原理可以从微观角度来解释吗?", "input": "", "output": "初中的物理老师告诉我们,光从一种介质斜射入另一种介质时,传播方向会发生改变,原因是在不同介质中,光的传播速度一般不同,导致波前方位发生改变,也就意味着光波的传播方向发生改变。那为什么光在不同介质中的传播速度不一样呢?\n\n这里大家需要知道,光的本质是一种电磁波,遵循麦克斯韦电磁方程。当一束光从空气射进介质中时,介质中那些绕原子核运动的电子在受到电场力后会受到极化,其运动方程如下所示。\n\n$\frac{d^2 \tilde{x}}{d t^2}+\\gamma \frac{d \tilde{x}}{d t}+\\omega_0^2 \tilde{x}=\frac{q}{m} E_0 e^{-i \\omega t}$\n\n当核外电子相对于平衡位置产生偏移时,等效于产生许多电偶极子,这些电偶极子也将会反过来影响光,改变电磁波本身的相位与幅值,产生等效的介电常数。\n\n而求解麦克斯韦方程组时,我们可以得到以下方程:\n\n$\tilde{\\epsilon}_r=\frac{\tilde{\\epsilon}}{\\epsilon_0}=1+\frac{N q^2}{m \\epsilon_0} \\sum_j \frac{f_j}{\\omega_j^2-\\omega^2-i \\gamma_j \\omega}$.\n\n这是物理学中经典的波动方程,这样的波动方程对应波的波速是:\n\n$\\sqrt{\tilde{\\epsilon} \\mu_0}$\n\n代入之前求得的介电常数,采取适当近似可以求得折射率的表达式为:\n\n$n=1+\frac{N q^2}{2 m \\epsilon_0} \\sum_j \frac{f_j}{\\omega_{j=}^2-\\omega^2}$.\n\n这就是介质中光速不同的一种微观解释,由于透镜也算是一种介质,其晶格间的各原子有很多这样的绕核运动的电子,它们受到光场作用后又反过来影响光场,影响光波的传播速度,从而改变波阵面取向,也就改变了光线的传播方向。", "task_type": {"major": ["问答"], "minor": ["知乎问答"]}, "domain": ["通用"], "metadata": "qid:515257964, aid:2484311158", "answer_from": "human", "human_verified": true, "copyright": "暂无版权及作者信息"} {"instruction": "阿基里斯为什么永远追不上先行的乌龟?", "input": "", "output": "一言以蔽之,曰:时间不够。\n\n芝诺悖论是这个样子的:跑步健将阿喀琉斯(或者翻译成阿基里斯,这不重要,下面叫他老阿)和乌龟进行百米赛跑,乌龟提前十米出发。老阿的速度是乌龟的十倍。老阿跑完十米,到达乌龟的起点时,乌龟已经往前走了一米;老阿再跑一米,乌龟又往前走了0.1米……如此循环,每次老阿跑到上一个计时点乌龟的位置时,乌龟总是又往前爬了一段距离。于是芝诺说,老阿永远追不上乌龟。\n\n这个悖论的关键在于,老阿和乌龟跑的距离有一个上限,是10+1+0.1+0.01+0.001+……=11.111……米,还不到12米。难道是有什么结界挡住它们不能再往前跑了吗?这个结界就是时间。假设老阿跑前十米用的时间是$$\\Delta t$$ ,那么跑第二段1米用时 $$0.1\\Delta t$$ ,第三段用时 $$0.01\\Delta t$$ ……,总用时\n\n$$t=\\Delta t+0.1 \\Delta t+\\cdots=\\Delta t \\times(1+0.1+0.01+\\cdots)=1.111\\cdots \\times \\Delta t$$\n\n本来,只需要 $$1.12\\Delta t$$,老阿就可以超过乌龟,可惜芝诺没有给他这个时间。在芝诺的年代,无穷的概念还没有很好地建立,人们还没有意识到无穷项等比数列求和的值可以是一个有限值,这就是芝诺悖论的症结了。", "task_type": {"major": ["问答"], "minor": ["知乎问答"]}, "domain": ["通用"], "metadata": "qid:455616550, aid:2462879977", "answer_from": "human", "human_verified": true, "copyright": "暂无版权及作者信息"} {"instruction": "近视眼在老了以后真的会恢复并且中和老花眼吗?", "input": "", "output": "结论是不会。因为老花(Presbyopia)和近视(Myopia)虽然同属于屈光不正(Refractiveerror),但是两者的成因并不相同,这也直接决定了即使同时得了近视和老花,最终视力并不会恢复正常。\n\n近视的成因目前研究很多,目前认为遗传+环境共同决定了你是否近视。曾经主流理论认为近视主要是遗传性的,也筛选鉴定出了一系列基因。但随着东亚地区获得性近视的大幅增加(东亚地区高中毕业生近视率在80-90%,可以看一下你的同学们有几个不近视的)现在认为阳光等环境因素在近视形成也发挥着极为重要的作用。\n\n根据病变部位的不同,可以把近视的机制分为:眼球轴向伸长(这是近视的主要成因)、屈光性变化(通常是晶状体屈光过度)、角膜曲率变化、眼介质折射率变化。但是近视总体上是远处像成像在视网膜前。\n\n而老花是一种正常的生理现象,一般在35岁后开始出现,逐渐失去对近距离物体的成像能力,成因是晶状体的老化失去弹性,使近物成像于视网膜后。并且随着年龄的增加,会逐渐加重,到60岁左右才会稳定下来。\n\n近视和老花在成因上是截然不同的,但是令人欣喜的是,近视患者在老花发展的初期确实会出现视力改善,所以一般近视患者发觉自己老花会更迟。但这只是暂时的,随着时间的增加,你依然会收获无法看清近物的老花,并且有可能会因为失去调节晶状体的能力,出现近远物都无法看清的情况。衰老和死亡才是生物的宿命。", "task_type": {"major": ["问答"], "minor": ["知乎问答"]}, "domain": ["通用"], "metadata": "qid:327277281, aid:2440858641", "answer_from": "human", "human_verified": true, "copyright": "暂无版权及作者信息"} {"instruction": "为什么有时候觉得玩游戏是在浪费时间,却还是停不下来?", "input": "", "output": "因为游戏好玩呀~\n\n匈牙利裔美国心理学家奇克森特米哈伊·米哈伊(MihalyCsikszentmihalyi)提出过心流理论,用于描述一种高度集中的,有利于生产力的精神状态。根据米哈伊的理论,进入“心流”状态的人具有以下特点:\n\n1. 做事情的时候无需多加思考,身体自动做出反应2. 对时间的感受是扭曲的,无法准确得感受到时间的流逝3. 专注投入到活动中,不易察觉到活动之外的刺激4. 在活动完成后可以感受到满足和愉悦\n\n当我们在玩游戏的时候,就有可能会进入到这种心流的状态中。此时,我们对时间的感受是扭曲的,并且几乎免疫外界的刺激,就会出现一打游戏就停不下来的情况。\n\n除了游戏以外,还有很多活动可以让我们进入到心流的状态中,例如阅读、绘画、竞技体育、做科研。这些容易让人进入到心流状态的活动都有一些共同的特征:\n\n1. 具有明确的目标2. 能够得到及时的反馈3. 难度和个人的能力相当\n\n不难发现,很多游戏都符合这三个要素:明确的任务、及时的奖赏、合适的难度,这些都是经过游戏开发者精心设计过的。在整个游戏的流程中,玩家的能力也是随之增长的,所以随着游戏流程的推进,游戏的难度也随之上升,让玩家能够一直处于“心流”的状态中,这也是让玩家能够一直玩下去的动力。\n\n不过,成龙说过:游戏里的你,再强大,也是假的,不是真的,所以不要贪玩迷恋", "task_type": {"major": ["问答"], "minor": ["知乎问答"]}, "domain": ["通用"], "metadata": "qid:305033646, aid:2429851434", "answer_from": "human", "human_verified": true, "copyright": "暂无版权及作者信息"} {"instruction": "为什么偶尔会感觉眼前的一幕曾经发生过(应该是叫既视感)?", "input": "", "output": "这种似曾相识的感觉在法语中有一个确切的名词叫做:Déjà vu。关于这种现象大家很多人肯定都体验过,研究表明多达97%的人口在一生中至少经历过一次Déjàvu,67%的人经常经历这种情况。对于焦虑症和去人格化等患者,这种经历则是更为常见。\n\n那究竟是什么导致了似曾相识的感觉呢?关于这个的解释有非常多种理论,但是其确切机理尚不清楚。大多解释表明该现象源自大脑的颞区,进一步确定原因则需要神经学家、脑科学家、精神病学家和实验心理学家参与的多学科方法来深刻探究。\n\n另一种理论认为,似曾相识是由知觉差距或感知分裂造成的。当大脑在特定时刻背靠背处理相同的感觉信号两次时,就会发生感知分裂。具体来说就是,第一个过程中,信号是短暂的,在有意识的头脑中经常被忽视。而在几乎紧随其后的第二个过程中,由于第一个信号就会建立一种熟悉感(似曾相识)而不是清晰的回忆感。\n\n还有其他诸如时空隧道的碰撞、时光倒流、时空错乱等一些七七八八的原因猜想,咱就看看暂时当一乐呵就行。偶尔(一年那么几次)的似曾相识体验即使科学家们还不确定为什么会发生我们也完全不用担心,但是,如果经常且伴有头痛、虚弱、意识丧失或模糊、心跳加速等症状,请及时就医咨询。", "task_type": {"major": ["问答"], "minor": ["知乎问答"]}, "domain": ["通用"], "metadata": "qid:382478033, aid:3292617140", "answer_from": "human", "human_verified": true, "copyright": "暂无版权及作者信息"} {"instruction": "是不是手机屏幕越黄,护眼效果越好?\n我的手机下载了一个小护眼app,我把屏幕调得挺黄的,是不是这样过滤蓝光效果会更好?", "input": "", "output": "黄光对智能手机和数码设备的保护作用主要集中在过滤蓝光上。蓝光是可见光光谱的一部分,波长较短,因此比红光或黄光等其他颜色的光具有更强的能量。能量的增加意味着蓝光可以更深地穿透眼睛,时间长了可能会对眼睛造成伤害,导致眼部疲劳,甚至会由于影响褪黑激素的分泌而影响睡眠周期。\n\n而黄光的波长较长,能量较低。如果在数码屏幕上使用黄色滤光片,就可以通过减少蓝光照射来增强对比度,从而有助于减轻眼睛疲劳。这些滤光片之所以能发挥作用,是因为眼睛的色觉锥状细胞对黄光的敏感度低于对蓝光的敏感度。此外,眼睛中一种叫做黑视蛋白的蛋白质参与调节睡眠-觉醒周期,它对蓝光的反应更灵敏;因此,过滤蓝光也有助于提高睡眠质量。\n\n以下是智能手机的黄光护眼模式设置有益的原因:\n\n- 减少蓝光照射:屏幕上的黄色色调可以最大限度地减少到达眼睛的蓝光量,这有助于减少潜在的视网膜损伤和眼睛疲劳。\n\n- 增强对比度:黄色屏幕可以增加对比度,使眼睛更容易集中注意力,减轻长时间看屏幕带来的疲劳。\n\n- 维持昼夜节律:通过减少蓝光,尤其是傍晚时分的蓝光,黄光可以帮助维持自然的睡眠-觉醒周期,而蓝光是众所周知的干扰因素。\n\n但需要注意的是,更多的黄光并不一定等同于更好的护眼效果。关键是要控制有害光波长的照射,而不仅仅是增加黄光的存在。各种设备都提供 \"夜间模式 \"或\"防蓝光滤镜 \"等设置,可将显示屏调整为暖色调,减少蓝光的发射。\n\n此外,专门的眼镜镜片镀膜层可以选择性地阻挡蓝光,在眼睛和设备屏幕之间提供一道物理屏障。还有一些软件解决方案可以根据一天中的时间调整屏幕色温,以减少蓝光照射,特别是蓝光在夜间尤其有害。\n\n总之,智能手机上的黄光设置是防止屏幕发出高能蓝光的一种预防措施。它们提供了更舒适的观看体验,可能有助于改善眼睛健康和睡眠模式。然而,关键在于光线照射的平衡和有害蓝光的减少,而不仅仅是光线本身的色调。", "task_type": {"major": ["问答"], "minor": ["知乎问答"]}, "domain": ["通用"], "metadata": "qid:402964655, aid:3285065247", "answer_from": "human", "human_verified": true, "copyright": "暂无版权及作者信息"} {"instruction": "为什么铜大多以铜离子存在而不是亚铜离子,亚铜离子不是全满更稳定吗?", "input": "", "output": "铜是一种我们称为过渡金属的特殊元素,在过渡金属中常有的规则常常变得没有那么绝对。这个问题的实质在于比较继续填充(失去)电子时,3d轨道和4s轨道所需的能量比较。而在过渡金属中,3d轨道和4s轨道的能量几乎相同。\n\n对这个问题,我们有两点需要注意:\n\n第一点是为什么能够失去d轨道电子:s轨道一般是球形的,但是密度很低,更大的主量子数意味着远离原子核,使得原本的内壳层d轨道实际上不会被完全屏蔽,也使得原本的d电子更容易与外界相互作用,因此即使是全满的d轨道,也不一定是稳定的。离原子核很远的d电子仍然有可能成为活性的价电子。\n\n第二点是失去d轨道电子的“动因”:当 $$\\ce{Cu}$$在水中成为离子配对时,它和溶剂(比如水)也会发生结合,结合降低的能量称作水合能。铜离子实际上比亚铜离子水合降低的能量更多。 $$\\ce{Cu^{2+}}$$中缺乏完全填充的3d轨道而导致的稳定性损失被 $$\\ce{Cu}$$ 离子的水合能所补偿。因此,可以说在溶液中 $$\\ce{Cu^{2+}}$$更加稳定。但是失去了两个电子后的铜离子里原子核对剩余电子的影响更大,使得+3价和+4价的铜离子相对更少见。", "task_type": {"major": ["问答"], "minor": ["知乎问答"]}, "domain": ["通用"], "metadata": "qid:491500997, aid:3275885893", "answer_from": "human", "human_verified": true, "copyright": "暂无版权及作者信息"} {"instruction": "如何证明由1到9这9个数字(数字不重复)组成的九位数,一定能够被九整除?", "input": "", "output": "首先,我们可以观察10^n-1,经过简单的减法运算,我们就知道这样的数每一位上都是9,我们通过除法就可以很容易地得知这个数除以9就等于一个每一位上都是1的数(比如10^3-1=999,999/9=111),也就是说对于任何一个10^n-1,它都可以被9整除。\n\n接下来,我们继续证明一个数如果所有位上的数字之和是9的倍数,那么这个数也是9的倍数:我们假设这个数有n位,从右到左每一位数分别为a0,a1……a(n-1),那么通过十进制的知识,我们就知道这个数字的具体数值就是a0+(10^1)a1+……+(10^(n-1))a(n-1)=(a0+a1+……+a(n-1))+(10^1-1)a1+……+(10^a(n-1)-1)a(n-1),我们可以很轻松地看到,后面的所有项都是10^n-1的形式,他们都能被9整除,而对于第一项,也就是所有数位上数字的和,由我们的条件可知,它也是9的倍数,自然这些项加起来的和也是9的倍数。\n\n再回到这个问题,我们用1~9不重复地组合成9位数,那么这个数每个数位上数字的和自然就是1+2+……+9=45,也是9的倍数,按我们上面的证明,它自然也可以被9整除。", "task_type": {"major": ["问答"], "minor": ["知乎问答"]}, "domain": ["通用"], "metadata": "qid:628159517, aid:3267638210", "answer_from": "human", "human_verified": true, "copyright": "暂无版权及作者信息"} {"instruction": "为什么大多数人都是右撇子?\n这种进化趋势是怎么形成的?在其他动物中也有这样的现象存在吗?这样的进化可能为个体带来什么生存优势吗?", "input": "", "output": "非常有趣的问题!可惜,这个问题影响因素太多,目前还没有一个让大家都很信服的答案。所以我们这里给出一些主要结果供参考,欢迎大家在评论区分享自己的看法~其实,自然界的“右撇子”也不少见:一些猿类也喜欢用右手抓取食物;座头鲸更喜欢用右鳄来撕咬;鸡、鸽子、蟾蜍等更喜欢用右侧视野来来判断捕食。这样的单侧利好行为,本身在生物学上具有一定的意义。\n\n超声研究发现,12周大的宝宝已经开始偏向于动右手臂,15周时开始吮吸右手手指。这一偏好比例大概就在1:9.我们可以肯定,必定存在相应的遗传基础。但是如果我们就这样回答说是因为遗传,显然不能让人满意。因为这是演化的结果而不是原因。\n\n一个直接的思路是研究历史上人类的利手倾向是如何出现并逐步演化的。但是相关考证较为困难,例如Toth通过研究剥脱石皮的区别,认为早在190~140万年前的石器时代右利手倾向已经出现,但是该结论在统计上受到质疑。历史上的演化过程仍然是有待研究的问题。具体是什么原因导致了人类的演化倾向,人们在很多方面都有猜测。\n\n人类身体与大脑的结构可能对演化具有影响。一方面,心脏位于左侧可能有一定影响。在远古时期,战斗对于古人类是不可避免的。当使用锋利的武器战斗时,用于握住武器的手可能会带动该侧胸腔旋转,导致该侧心脏更多地暴漏。当右手拿着武器时,左臂通过部分覆盖左胸,也可以为心脏提供一些保护。这在现代击剑技术中也依然有体现。另一方面,大脑左右侧的分工可能对利手倾向有重要影响。将更多的反应活动交给一侧后,可以降低同时参与某项进程而可能产生的“争抢”。\n\n此外,有人认为或许是无意造成的结果。如果人类群体在某一时段偶然形成群体偏向,就有可能形成群体习惯,代代相传。比如人类偶然获得的一些技能手段、武器的使用方法产生偏向性后,就代代相传;随着人类文明的发展、文字文化的产生,这一倾向性也被不断加强。甚至在具体的文化教育环境下,左撇子可能要被迫用右手。随机观点听起来不无道理,但是具体证明其在演化中起到的作用也不是件简单的事。\n\n紧跟着这个问题是一个同样有趣的问题,为什么仍然有人是左撇子呢?遗传因素之外,一方面是因为右撇子多到一定程度后,左撇子成为被人忽略的少数群体,他们在一定方面获得了策略优势。另一方面,脑影像科学也表明,左撇子人群的右手使用通常也是较为顺利的,并不会导致同侧初级运动皮层的失活不均,在某些类型的运动中具有先天优势。所以人群中仍然会有一些人保留了左撇子的习惯。\n\n当然啦,左撇子右撇子本身并不重要,也没有证据表明哪个更聪明或如何,发挥好自己的特长手就好~", "task_type": {"major": ["问答"], "minor": ["知乎问答"]}, "domain": ["通用"], "metadata": "qid:22377365, aid:3257558849", "answer_from": "human", "human_verified": true, "copyright": "暂无版权及作者信息"} {"instruction": "温度对原子核束缚电子能力有什么影响?", "input": "", "output": "物理上在不考虑亚原子结构的时候,计算原子的平动动能 $$E_K = \\frac{3}{2}k_BT$$ ,其中玻耳兹曼常数 $$k_B = 1.380649\\times 10^{-23}J/K$$ ,这就把温度和能量联系了起来。这样,有时候物理学家也会用温度来表达能量。电子伏特与开尔文的换算关系是1eV ≈11600K。\n\n在孤立原子中,原子核与电子结合的能量一般在10eV左右,按上面的换算大约是11万开尔文。比如氢原子的第一电离能为13.6eV。常见元素原子中第一电离能最低的是K,是4.3eV,换算成温度大约是50000K。这个温度大概就可以看作孤立原子中的电子仅靠热运动脱离原子核束缚时的温度。很显然,这个温度远大于室温,因此在通常环境下不需要考虑温度变化对原子核-电子束缚强度的影响。\n\n在晶体中,大量原子规则排列,互相影响,形成所谓的强关联体系。物理学家发展了能带论在描述这种体系中的电子,被原子核束缚的电子形成价带,可以在不同原子间自由移动的电子形成导带。导带与价带之间的最小能量差被称作能隙。能隙一般较小,有时可达几电子伏,但很多材料的能隙都在毫电子伏量级,甚至零能隙的材料也很常见。毫电子伏的能隙对应几十到几百开尔文,正是实验室关心的温度区间。这时,温度就会显著影响电子的激发,实验上表现为电阻等电学性质随温度的变化在2-300K变化明显。", "task_type": {"major": ["问答"], "minor": ["知乎问答"]}, "domain": ["通用"], "metadata": "qid:622267022, aid:3212987924", "answer_from": "human", "human_verified": true, "copyright": "暂无版权及作者信息"} {"instruction": "为什么有的中性笔墨水速干,有的干的很慢?", "input": "", "output": "所谓的“速干”主要受触变恢复性影响。\n\n良好的墨水表现为:中性笔开始书写时,墨水可以由不能流动的凝胶状态迅速转变为自由流动的溶胶状态,这样不会出现起笔位置无墨的现象;当停止书写,墨水受到的剪切应力消失后,墨水由粘度较小的溶胶状态可以迅速转变为不能流动的凝胶状态,也就是墨水干了。触变恢复性越好,上述两个转变就越快,墨水也就干得越快。\n\n不过很遗憾,虽然触变恢复现象很明确,但我们对其机理的讨论仍未达成共识。我们一般认为,触变恢复现象的产生是流体在剪切应力作用下,应力对流体稳定体系的破坏和布朗无规则运动对稳定体系的重建之间较量的结果。这其中的重要角色就是增稠剂。\n\n在增稠剂的作用下,流体的稳定状态其实是在笔管中的不能流动的凝胶状态。当墨水静置时,增稠剂与水分子形成氢键或增稠剂形成胶束将水分子包围,形成一个网状的空间结构,此时,墨水粘度高,处于不能自由流动的凝胶状态。书写时,笔尖滚珠转动,此时,墨水会受到来自笔尖的剪切应力,增稠剂的分子链由原来的无序交联状态转变为平行流动状态,彼此间的相互作用减弱,导致粘度急剧下降,从而使墨水由凝胶状态转变为可以流动的溶胶状态,顺着笔尖流出。而当附着在纸面上时,来自笔尖的剪切应力消失,墨水又重新恢复到凝胶状的稳定状态,不再具有流动性,也就是墨水干了。\n\n* * *", "task_type": {"major": ["问答"], "minor": ["知乎问答"]}, "domain": ["通用"], "metadata": "qid:49896701, aid:2363970482", "answer_from": "human", "human_verified": true, "copyright": "暂无版权及作者信息"} {"instruction": "为何金属与绝缘体在导热性与导电性方面差别这么大?", "input": "", "output": "这很大程度上可以用固体物理中的能带理论进行解释。我们先了解一下固体中能带是怎样形成的。首先,量子力学告诉我们,对于单个孤立原子来说,原子中的电子在原子核以及其他电子的共同作用下分布在不同的能级上,能级从小到大用s,p,d等符号来表示。如果多个原子靠在一起会发生什么呢?以两个原子为例,当两个原子互相靠近时,原子中的电子同时受到本身原子势场以及另一个原子的势场的影响,通过量子力学的方法处理可以得到结果就是使每个原子之前的分立能级分裂成两个能级,两个原子靠近的越近,能级分裂地越厉害。越多的原子靠近在一起,能级就会分裂成更多份。我们知道,金属和绝缘体是由大量原子组成的晶体,当如此多的原子靠近在一起时,能级就会无限地分裂,原本的一个能级随之就变成了一条带子,我们称之为能带。当然实际上考虑到轨道杂化的影响,原先的每一个能级不一定与每一条能带一一对应。在能带中电子可以占据的部分称为允带,不能占据的部分称为禁带。\n\n其次,固体能够导电,是因为电子在外加电场的作用下做定向移动,也即是说外加电场改变了电子的能量和动量,从能带的观点来看,就是电子从一个能级跃迁到另一个能级。电子按照能级大小从小到大排布,如果一条能带每一个可以容纳电子态的能级都被占据了,我们称之为满带,电子要改变能级无处可去(因为电子是费米子服从泡利不相容原理),所以表现出不能自由移动的特征。反之,对于不满带来说,依然有空缺的状态可以让电子跃迁过去,所以不满带的电子是可以导电的。我们常说的金属导电的自由电子就是处于不满带中的电子,而绝缘体因为电子完全占据了价带,价带之上的导带中没有电子,也就是不存在自由电子了。事实上,只要存在温度,绝缘体也是可以导电的,这是因为温度的影响使一小部分电子从价带跃迁到导带变成自由电子,同时又在价带中留下空穴,而绝缘体(或半导体,半导体价带与导带间的间隙相对与绝缘体更小)导电和金属导电的本质区别就是前者存在空穴导电。只是因为这些电子和空穴非常少,所以其导电能力也非常差,生活中我们很难感受到绝缘体中的微弱电流,所以姑且认为它们是不导电的。\n\n至于导热性,固体中传导热流可以由电子(空穴)承载也可以由声子承载,但对于绝缘体前一种方式被极大程度地压制了,导致其导热性也远不如金属。", "task_type": {"major": ["问答"], "minor": ["知乎问答"]}, "domain": ["通用"], "metadata": "qid:410286281, aid:2310387061", "answer_from": "human", "human_verified": true, "copyright": "暂无版权及作者信息"} {"instruction": "为什么说两条腿的就应该跑不过四条腿的,有科学根据吗?", "input": "", "output": "跑得快一般指爆发力强,而跑得远是耐力好,一般来说这两项能力就像鱼和熊掌不可兼得。现存的动物中,人类已知的跑得最快的陆生动物是猎豹,毫无疑问是四条腿的,但是猎豹只能全力奔跑几分钟。猎豹的高速奔跑能力和它的四肢肌肉、脊椎形态乃至尾巴和爪子都有关系,可以说猎豹的整个身体就是为了极速奔跑而生的。那么哪种动物跑得远呢?\n\n一个在古人类研究中比较著名的说法是,古猿人在掌握制造石器之前,捕猎的方式除了围猎,另外一种重要方法就是长时间的追逐。由于猎物体能恢复的速度不如人类,因此在可以长达一天甚至几天的的追逐后,猎物最终力竭而亡。这种说法虽然还有些争议,但主体是被很多人接受的。\n\n所以说,如果把奔跑的时间拉长到几天,那么人类——至少人类的祖先,是“跑得远”的有力竞争者。事实上,由于缺乏良好的评判标准,到底哪种动物跑得远并没有公论。那么为什么很多人都会有四条腿的动物跑得更快更远的印象呢?我们不妨考虑一下基数。现存的动物中,能长期两条腿奔跑的种类除了一些鸟类(比如鸵鸟)外,只有部分灵长类(主要是人类)和有袋类(袋鼠),数量其实很少。但是大多数哺乳动物都是四条腿奔跑的,它们可以选拔出很多“运动员”,跑得快的有猎豹、羚羊,耐力好的有马和犬(尤其雪橇犬)。在人类的驯化下,许多动物的奔跑天赋被放大、利用并为人所知,这大概是这种未必正确的印象的来源吧。", "task_type": {"major": ["问答"], "minor": ["知乎问答"]}, "domain": ["通用"], "metadata": "qid:21184612, aid:2275554626", "answer_from": "human", "human_verified": true, "copyright": "暂无版权及作者信息"} {"instruction": "为什么很多水果刚买不甜,越放越甜?", "input": "", "output": "这就是水果的成熟过程。\n\n对于部分较容易腐坏的水果,往往是以不完全成熟的状态进行运输和销售,比如香蕉和猕猴桃。随着时间的推移,这些未成熟的果实,多数情况下会自发走向成熟。也就是不甜的未成熟果实变成了香甜的成熟果实。在这一过程中,乙烯扮演了极为重要的角色(在部分水果成熟过程中发挥主要作用的激素不是乙烯,而是脱落酸等其他激素,这里不做论述)。\n\n乙烯是极为重要的一种植物激素,来源于植物细胞中甲硫氨酸的分解,随后以气体形式释放出细胞,被目的细胞的乙烯受体感知响应。乙烯在植物中的作用很广,包括但不限于:促进果实成熟、叶片脱落以及枝干伸长。\n\n植物果实在成熟的过程中,会自发释放乙烯,诱导果实的成熟。这一过程中,生物碱和丹宁等苦涩物质含量会下降,果酸和可溶性糖含量上升(糖分通常来源于细胞壁多糖降解,所以成熟的水果会更软,这也解释了果实为何在离开植株后依然可以变甜)。\n\n当然现在也有人工方式来催化水果的成熟过程,常见的是通过在密闭环境中释放乙烯和乙炔气体,来诱导水果的成熟。需要说明的是,这是一种世界各国都允许的安全技术,毕竟这和把未熟水果和其他成熟水果放一起催熟的土方法,利用的都是乙烯。这种催熟的过程中最大的危险,反倒是催熟的库房乙烯浓度过高有爆炸风险。", "task_type": {"major": ["问答"], "minor": ["知乎问答"]}, "domain": ["通用"], "metadata": "qid:23765421, aid:2101334196", "answer_from": "human", "human_verified": true, "copyright": "暂无版权及作者信息"} {"instruction": "到底谁慢了?我看高速航行的人觉得他慢了,他看我也觉得我慢了。为什么只有黑洞边上的人慢了,不是相对的吗?", "input": "", "output": "运动是相对的,引力是绝对的。运动不是时空的固有属性,而引力是时空的固有属性。\n\n在双生子佯谬中,狭义相对论会给出所谓的相互看对方变慢了这样的结果,是因为运动是相对的。抛开两兄弟一个人在地面上一个人在飞船上的固有观念,在超高速飞船上的人和在太阳系中静止失重的人的运动状态有本质区别吗——没有。大家都在惯性系中。\n\n我们可以做一个思想实验:把两人装入电梯,一个电梯在宇宙飞船上,一个电梯在太阳系中静止失重。两人可以在电梯里做任何物理实验,但是无法走出电梯,也无法看到电梯外面的景象。他们没有办法判断自己到底是在地球附近还是飞船上。这也就意味着,两个人所处的时空是等价的,这时一切才是相对的。因此没有“高速运动就会让时间流逝变慢“,只有“相对我高速运动就会让我觉得他的时间变慢了”。\n\n但是引力是绝对的,没有相对引力这种说法。还是刚才那个思想实验:两个电梯,一个在飞船上,一个在有重力的地面上,电梯里的人有办法判断自己在太空还是在地球吗——有办法:扔一个球,看这个球是不是加速运动即可,地面上的球才会加速下落。所以受重力的电梯和不受重力的电梯的时空是不一样的。广义相对论告诉我们,在受重力的时空中,时间流逝会更慢。\n\n总结一下,狭义相对论/相对运动引起的变慢全都是相对地变慢,你看我慢我看你也慢。引力/加速度引起的变慢都是绝对的变慢,会有-你看我慢我看你快这样的效果。", "task_type": {"major": ["问答"], "minor": ["知乎问答"]}, "domain": ["通用"], "metadata": "qid:464454991, aid:1934867840", "answer_from": "human", "human_verified": true, "copyright": "暂无版权及作者信息"} {"instruction": "把一张纸卷成圆筒形,罩在耳朵上,可以听到里面的声音很大,这种声音是如何形成的,或者说怎么听到的?", "input": "", "output": "简单来说,纸卷放在耳旁后听到了声音,是纸卷内空气柱从环境噪音中“选出”了与空气柱共振的声波放大的结果。\n\n首先,声波其实就是在声介质中传播的机械波。我们知道,任何一段振动都可以分解为一系列正弦波的叠加,这在数学上被称为傅里叶变换。而我们对于正弦波是可以讨论其频率的。因此,我们常说“人耳可以听到20~20000Hz之间的声音”正是指:我们能够听到任意声波经过傅里叶变换后,位于这一区间的组分。我们生活的环境中充满了噪音,我们所谓“寂静无声”的情况,通常也是指这种底噪很小而非不存在底噪。\n\n共振是生活中常见的一种现象。例如,有一位朋友想让你在他荡秋千时推他,使他荡的尽可能高,最佳的策略就是在每个周期他最接近你的时候沿他运动方向推他。这个例子中,朋友与秋千体系运动的频率仅依赖于体系的固有属性(转动惯量)与重力加速度,因此可以定义为系统的固有频率。而你推他的频率定义为外界对体系施加力的频率。驱动力的频率等于固有频率时,体系发生共振,即振幅达到最大值。对于一个传导机械波的体系,仍然会有一个仅由其内禀属性决定的固有频率,如果我们用机械波代替秋千的周期运动,共振的概念就可以自然拓展。共振有时会造成灾难,历史上就曾有因大风引起的涡旋与吊桥共振,最终使吊桥坍塌的事件。不过好在在我们讨论的问题中,共振温和而无害。\n\n回到这个问题,筒内的空气柱会与频率等于空气柱固有频率的声波发生共振,将这一频率的声波放大。而空气中噪声的频率组成是杂乱无章的,其中总会存在这一频率的声波。如果我们在完全安静的地方做这个实验,由于没有了噪声这一驱动,我们将会什么也听不到。空气柱的共振频率也会随着空气柱的形状改变而改变,比如短纸筒“发出”的声音频率会高一些,而长纸筒“发出”的声音频率会低一些。这种空气发生共振的现象在物理学中被称为亥姆霍兹共振,打开高速行驶汽车的窗户会听到“呼呼”的风声,也是亥姆霍兹共振的一种体现。", "task_type": {"major": ["问答"], "minor": ["知乎问答"]}, "domain": ["通用"], "metadata": "qid:36751435, aid:1912433552", "answer_from": "human", "human_verified": true, "copyright": "暂无版权及作者信息"} {"instruction": "为什么灰尘、水雾等物质比空气的密度大得多,却可以悬浮在空中?", "input": "", "output": "灰尘、小液滴能够漂浮在空中主要是由其在重力场中的终端速度小,空气对流这两个因素所导致的。\n\n我们知道,在空气中运动的物体所受的空气阻力通常与物体相对空气的运动速度正相关。灰尘等小颗粒在自由落体时,空气阻力会逐渐增大并最终等于自身的重力,此时颗粒保持匀速运动。对应速度即为粒子的终端速度。对流是指由于空气各处温度不均匀产生的空气流动。\n\n我们通常讨论的灰尘与水雾中的小液滴的直径在1~100 $$\\mu\\mathrm{m} $$之间,这个大小的粒子能够与可见光发生明显的散射作用,进而被我们所观察到,这也是高中化学中学过的丁达尔效应的原理。这个尺度下的颗粒可以利用流体力学中的Stockes’Law计算其终端速度。根据参考资料,计算公式为:$$ V_{t}=\\frac{\\rho_{p}D_{p}^{2}g}{18\\mu} $$ , $$V_{t}$$ 是颗粒的终端速度, $$ \\rho_{p}$$ 是颗粒的密度, $$D_{p}$$ 是颗粒的直径(设颗粒为球形), $$g$$ 为重力加速度, $$\\mu$$ 是空气的黏度。我们不妨取颗粒直径为10$$ \\mu\\mathrm{m} $$ ,颗粒密度为4 $$^{\\circ}\\mathrm{C}$$ 时水的密度1000$$\\mathrm{kg/m}^{3}$$ 。将各个参数带入公式,可以计算出水雾中小液滴的终端速度约为:0.00301m/s。可以看出,这个速度相当的小。灰尘等颗粒的终端速度就很容易在和空气作用中被对流所抵消,最终导致灰尘在空中以不确定的速度漂浮。\n\n最后,对于灰尘悬浮于空中通常的解释有一个误区,认为灰尘悬浮是由于灰尘小颗粒在空气中进行布朗运动。但其实在我们所讨论的颗粒尺度:1~100$$\\mu\\mathrm{m} $$ 下,颗粒由于气体分子碰撞产生的无规则运动极其微弱,运动几乎不受布朗运动影响。", "task_type": {"major": ["问答"], "minor": ["知乎问答"]}, "domain": ["通用"], "metadata": "qid:457397327, aid:1885676041", "answer_from": "human", "human_verified": true, "copyright": "暂无版权及作者信息"} {"instruction": "留刘海能避免发际线变高吗?", "input": "", "output": "也许你是觉得头发如果正常长的话,会翘出去,力臂长。如果变成刘海,耷拉在脑门上,力臂会变短,发根处所需要的力矩也会减小,这样头发就可以多一点,但我只能残酷地告诉你,答案是不能……这样只能掩盖你脱发的现实,科研狗看了眼镜子哭了出来。\n\n事实上,绝大多数的防脱方法都是没有什么作用的,食补更是不现实。\n\n所以如何科学防秃?\n\n秃头是基因决定的(这里主要讨论最为常见的雄激素脱发,拔毛癖、真菌感染这些不做讨论),所以要认命,逃不掉的。而且男性雄性激素水平更高的原因,秃头会很更加严重和显著(女性也有雄激素脱发,只不过女性的表型主要是头发稀疏且主要在头顶表现)。\n\n目前有效的秃头治疗药物有:1)米诺地尔:直接涂抹于头皮,防止进一步脱发,多数防脱水的有效成分,仅对30%-40%的人是有效的,需要一年才会有效,副作用是刺激皮肤外加促进脸部毛发生长;2)非那雄胺:口服,阻断雄激素对毛囊的影响,效果显著,副作用是降低性欲、增大乳房,勃起功能障碍,潜在的前列腺风险;3)激素调节剂:仅对部分女性安全有效,副作用和口服避孕药一样,毕竟这东西就是口服避孕药……\n\n最好的治疗方法还是植发,基本上一劳永逸,缺点就是贵,论根计价。当然像提问者一样通过留刘海等美发手段来遮盖秃头是有效且便宜的,还比假发舒适,不过当风太大吹乱发型还是会暴露秃头的。", "task_type": {"major": ["问答"], "minor": ["知乎问答"]}, "domain": ["通用"], "metadata": "qid:300380784, aid:1677459776", "answer_from": "human", "human_verified": true, "copyright": "暂无版权及作者信息"} {"instruction": "为什么冬天下雪不会电闪雷鸣,而夏天下雨时会呢?", "input": "", "output": "打不打雷和下雨还是下雪没关系,夏天之所以更容易打雷,简单来说,是因为夏季天热且水汽充足,更容易出现雷雨天气。\n\n雷雨天气是一种强对流天气。强对流天气是气象学上所指的发生突然、移动迅速、天气剧烈、破坏力极大的灾害性天气,主要有雷雨大风、冰雹、龙卷风、局部强降雨等,是仅次于热带气旋、地震、洪涝之后第四位具有杀伤性的灾害性天气。强对流天气一般尺度较小,一般水平范围大约在十几公里至二三百公里。生命史短暂,具有突然性。\n\n强对流天气的原因就在于空气强烈的垂直运动。最典型的就是夏季午后的强对流天气:白天地面不断吸收太阳的辐射,温度上升,同时放出辐射加热大气。近地面大气温度上升,膨胀,在浮力作用下上浮,如果水汽充足的话,就会形成一个上升的湿热空气流。当上升到一定高度时,由于高空气温下降,水蒸气饱和凝结成水滴,这就是积雨云,高层的积雨云可能还会有冰晶。雷电一般就产生于对流发展旺盛的积雨云中,由于对流,积雨云中冰晶的凇附,水滴的破碎以及空气对流等过程,都会使云中产生电荷。云中电荷分布比较复杂,不同的云可能带有不同电性的电荷,带电的雨云会使大地感应出相反电性的电荷,当电势超过空气击穿强度之后,就会对地放电。带有不同电性的雷云之间也有可能放电,这就是云间闪电。\n\n所以,夏季更容易满足出现雷雨天气的气象条件,不过,这也不意味着冬天一定无法出现雷电天气。\n\n* * *", "task_type": {"major": ["问答"], "minor": ["知乎问答"]}, "domain": ["通用"], "metadata": "qid:373897422, aid:2686794690", "answer_from": "human", "human_verified": true, "copyright": "暂无版权及作者信息"} {"instruction": "除了平面镜,还有可以完美成像的光学系统吗?", "input": "", "output": "目前没有。\n\n成像的光学系统可以分成平面镜反射和透镜折射两大类,再独特一点的还可以开一个直射的小孔成像和干涉的全息成像的小分类。在平面镜反射的类型中,凹面镜和凸面镜显然都会产生像哈哈镜那样的畸变,只有平面镜可以严格地完美成像。在透镜折射的类型中,光线会不可避免地色散从而产生色差,还会有球差、彗差、像散、场曲、畸变,总有一款让折射式的光学系统变得不完美。小孔成像中,只要孔的大小不是无限小,就必然会发生弥散。全息成像中,不同颜色(波长)的光的干涉效果是不一样的,所以目前比较成熟的全息成像还是得用单色激光实现,另外,再现光在全息干板的自干涉也是无法避免的,这些都会产生像差。\n\n我们再来看看刚刚提到的透镜成像系统中的像差具体是怎么产生的。\n\n球差:光轴上一点发出的不同倾角大小的光经过透镜后没有再次汇聚为一点,而是汇聚成同心圆;\n\n彗差:光轴外一点发出的不同倾角大小(相对于透镜中心)的光经过透镜后没有再次汇聚为一点,而是汇聚成偏心圆,共同组成类似彗星的形状;\n\n像散:对于光轴外的一个点光源发出的光,即使倾角大小一样,只要倾斜方向不一样,它们的汇聚点也依然不一样,横向倾斜的光的焦点和纵向倾斜的光的焦点之间的距离称为像散。\n\n场曲:之前考虑的都是单个点的成像的偏差。场曲考虑的是一个平面状的物体,它上面的点经过成像后不再保持在一个平面上,而是会在不同像距处成像,形成曲面。用平面传感器来接收它,会发现各点不能同时清晰成像。\n\n畸变:离光轴不同距离处的点,即使能够清晰成像,像的放大率也不一致。\n\n色差:光轴上的点发出的不同颜色的光会成像于不同位置,形成位置色差。光轴外的点发出的不同颜色的光在成像后与光轴的距离也不同,形成倍率色差。\n\n透镜有这么多的debuff,看来还是平平淡淡的平面镜才是真呐。", "task_type": {"major": ["问答"], "minor": ["知乎问答"]}, "domain": ["通用"], "metadata": "qid:550079173, aid:2646171539", "answer_from": "human", "human_verified": true, "copyright": "暂无版权及作者信息"} {"instruction": "奥斯特先发现了电流的磁效应,然后法拉第才发明的发电机,那么,奥斯特哪来的电呢?是闪电转换而来的吗?", "input": "", "output": "人类利用电的历史远比法拉第发明发电机要早。\n\n早在十八世纪,人们就发明了原始的电容器——莱顿瓶,并用它来储存静电、用做电学实验的供电来源。典型的莱顿瓶是一个玻璃容器,内外包覆着导电金属箔作为极板。瓶口上端接一个球形电极,下端利用导体(通常是金属锁链)与内侧金属箔或是水连接。充电之后瓶子内外的金属箔就会带上符号相反的电荷——至于怎么充电,先卖个关子。\n\n十八世纪末,伽伐尼发现死青蛙腿在接触到电火花的时候会痉挛并产生电力——当然他对这一现象的解释受到了时代的局限。受伽伐尼启(争)发(论),伏打(就是电压单位的那个伏特)发明了人类历史上第一个原电池——伏打电堆。伏打注意到了不同种金属通过盐水接触时会产生电流,1800年,伏打堆叠了几对交替的铜或银以及锌盘(电极),其中用浸有盐水(电解液)的布或纸板隔开,就制成了伏打电堆。\n\n翻回前面卖的关子。法拉第的能够输出电流的发电机确实要到19世纪中叶才被制造出来,但是在此之前,人们已经能够制造静电发电机,并用来给电容器充电了。德国发明家奥托·冯·格里克在1663年就制造出了初始形态的摩擦发电机,通过摩擦一个硫球来起电;之后,艾萨克·哪里都有我·牛顿把硫球替换成了玻璃球;到了1787年——比法拉第发电机早大约50年的时候——经过不停改进的静电起电机已经可以用于医疗目的。经过二百多年的发展,静电发电机在20世纪初叶进化出了高达形态——物理课本上介绍过的范德格拉夫起电机。", "task_type": {"major": ["问答"], "minor": ["知乎问答"]}, "domain": ["通用"], "metadata": "qid:538148151, aid:2532607675", "answer_from": "human", "human_verified": true, "copyright": "暂无版权及作者信息"} {"instruction": "自行车转弯时为什么要倾斜?", "input": "", "output": "我们假设我们的自行车即将右转弯,某观察者在车后方观察。\n\n自行车准备要右转了,我们向右转动车把手,前车轮会被车把手带动的指向右前方,如果摩擦力足够,那么车轮和地面之间不会发生滑动,因此前车轮应该获得向右的速度。这样我们就完成了右转的动作。\n\n那这和向右倾斜有什么关系呢?我们从车后方的观察者的角度看,车受到一个向右的摩擦力来提供向心力。根据质心系的理论,对于一个体系我们希望研究它的转动,我们都可以选取质心C作为支点来研究。力矩等于力乘力臂,重力一定过质心,因此不用考虑。如果自行车不倾斜,那么支持力也过质心,摩擦力就会让自行车向左倾,而左倾之后支持力在重力右侧,会让车继续左倾,自行车完全无法平衡。\n\n为了让我们能够顺利地完成右转弯而不摔倒,我们需要让自行车转动平衡。如果我们将身体右倾,那么支持力在重力的左侧,会提供让自行车右倾的力矩而与摩擦力的左倾力矩相抵消。二者一旦平衡,只要地面足够粗糙,提供足够的摩擦力,自行车就可以顺利地右转。\n\n根据向心加速度公式,速度越大,转弯半径越小(也就是我们把车把手转的角度越大),转弯需要的向心力就需要越大,因此,为了平衡摩擦力,所需要的力矩也就越大。支持力大概就是重力,不能变,那我们只能够倾斜的更多一些,让重心偏离车轮与地面接触点更多,增大力臂让自行车平衡了。这就是为什么我们骑车如果想快速过弯或者转一个小弯的时候,如果不倾斜多一点会有“转不过去”的感觉。\n\n总的来说就是“想转弯需要倾斜”。当然了,这只是一个简化的模型,事实上我们骑车时有可能会左右摇晃,车的转动未必是平衡的。", "task_type": {"major": ["问答"], "minor": ["知乎问答"]}, "domain": ["通用"], "metadata": "qid:21567036, aid:2429853528", "answer_from": "human", "human_verified": true, "copyright": "暂无版权及作者信息"} {"instruction": "为什么人们第一次听见一个陌生的字音就能模仿发音呢?\n在不看对方嘴型的情况下,仅听声音就能知道嘴部和舌头该以怎样的动作去发出这个陌生的音调,大脑是怎样仅通过读音就能准确地推断出发出这个读音需要的动作呢?", "input": "", "output": "听到一个发音就能发出同样的音通常是很困难的,除非母语或者熟练掌握的语言中有类似的发音。在更多情况下,听到一个发音就能准确地重复它,这需要一定的经验和发音知识。\n\n对于母语中不作区分,但是发音口型或发音方式有区别的发音,人们是很难仅靠听将其区分开来的,例如送气和非送气的/k/发音在英语中表示相同的意义(如果你不知道送气音和非送气音,你可以将非送气音粗糙地理解为所谓的“浊化”,而送气音则为正常),因此以英语为母语的人难以区分这两个发音,那么他们在发音时也就不会刻意区分它们。这样一来,听到一个发音后重复,可能就是另外一个了。\n\n此外,人们在学习外语的时候,倾向于将母语的发音迁移到目标语上,当一个人听到母语中没有的发音时,他可能会选择一个母语中与之最接近的发音来重复这个新的发音。这个现象较为常见,例如中国人说英语往往会带有中国口音,印度人则往往带印度口音。\n\n所以,要想准确地重复一个发音,仅听是不够的。但我们似乎总能模仿个大概,这是因为不同语言的元音、辅音有共同特征(例如爆破音p, k, t,元音a, u, o,i, e普遍存在于多种语言),我们学过中、英两种语言能够应对相当范围的发音。\n\n成年人在模仿别人发音的时候,会将对方的话语拆成发音单元,每个发音单元对应到自己会的(例如母语中的)某个发音,而婴儿则不是这样。\n\n婴儿学习发音的具体机制尚有争议,其中模仿理论认为婴儿通过模仿成人的声音来发出自己的声音,不需要口型参与,模仿理论的另一方观点则认为婴儿通过模仿成人的口型学习发音。\n\n除模仿外,有理论认为婴儿通过父母的反馈学习发音。婴儿在咿呀学语阶段会尝试发出各种奇怪的发音,而他的家人会从这些发音中识别出自己母语中有的发音,并将自己识别出的发音通过一种夸张的方式说出来,作为给婴儿的反馈,婴儿收到反馈比较并修正自己的发音,如此迭代婴儿就学会说话了。", "task_type": {"major": ["问答"], "minor": ["知乎问答"]}, "domain": ["通用"], "metadata": "qid:623219032, aid:3314159321", "answer_from": "human", "human_verified": true, "copyright": "暂无版权及作者信息"} {"instruction": "为什么金刚石是自然界最硬的物质?", "input": "", "output": "金刚石也被称为钻石,是由碳元素组成的一种无色晶体,是自然界最硬的物质。自然界里的金刚石一般形成于地球内部高温高压的环境,随后被火山喷发等地壳运动带到地表。\n\n那么,金刚石为什么是自然界最硬的物质?在谈论这个问题之前,我想先说说其他物质相较于金刚石究竟差在了哪里。\n\n首先是同样为碳的同素异形体的石墨,石墨拥有层状的结构,虽然层内由强力的共价键连接,但层与层之间却由微弱的范德华力维持。范德华力比共价键弱得多,通常其能量小于5kJ/mol,比共价键要弱2个数量级。因此,我们掰开石墨只需要破开它弱小的层间作用力即可。石墨的硬度也因此不高。\n\n然后是与金刚石结构类似的二氧化硅,虽然既学到了金刚石的结构,还同样拥有强力的共价键,但二氧化硅的硅氧键是极性共价键,金刚石的碳碳键是非极性共价键。极性共价键由于既有共价性又有离子性,更易被破坏。举个通俗的例子,这就像两只狗狗在拖拽一块肉,如果一只狗比另一只强壮,就很容易将肉抢走,结束拖拽。但如果两只狗势均力敌,则会相持不下,维持拖拽的状态。\n\n再看金刚石,其不仅由稳定的非极性共价键组成,所有的价电子都参与了共价键的形成,而且碳碳键键长短而键能高,其三维的构型也能够将外力分担到晶体各处。尽管有的物质拥有强力的共价键,有的物质拥有稳固的构型,但极少能像金刚石这样同时做到如此优秀的程度。\n\n那么有物质比金刚石更硬吗?答案是有的,目前人类已经能够合成很多比金刚石更硬的材料了。例如用压缩富勒体合成的聚合钻石纳米棒,以及聚合物聚甲炔在1,000°C氩气气氛下热分解而成的蓝丝黛尔石(Lonsdaleite)。值得一提的是,蓝丝黛尔石不仅可以人工合成,而且还是天外来客。其第一次被鉴别出,是在1967年美国亚利桑那州的巴林杰陨石坑。但天然的蓝丝黛尔石拥有杂质,硬度低于金刚石,如果人工合成则比钻石硬58%。\n\n虽然人工合成的许多物质硬度已经能够超过金刚石,但其硬度受限于共价键的强度。如果有一种物质能够不依赖电子,只需要核子构成,那么它的硬度就能突破电子的限制。那就是水滴中子星。\n\n中子星是恒星演化到末期,经过引力坍缩发生超新星爆炸后形成的星体。电子被强大的引力压进原子核中,质子和电子将会因引力的作用结合在一起成为中子,泡利不相容原理则会维持中子星的稳定,阻止其进一步坍缩。中子星是宇宙中已知最刚硬的物体,它的杨氏模量比钻石还刚硬20个数量级。", "task_type": {"major": ["问答"], "minor": ["知乎问答"]}, "domain": ["通用"], "metadata": "qid:299990575, aid:2353407260", "answer_from": "human", "human_verified": true, "copyright": "暂无版权及作者信息"} {"instruction": "电话号码是怎么排的?有多大概率会发生我和别人后四位相同的情况而导致外卖或快递错拿的情况?", "input": "", "output": "十一位手机号的前三位是移动接入码,一般由工信部按不同的号段拨付给不同的运营商,因此在不考虑携号转网的情况下,前三位号码基本可以确定手机号的运营商(为什么说基本,因为后面要提到特殊情况);中间四位是地区号,最初用户比较少的时候使用当地固定电话区号,后来随着用户数量增加,一般使用相近的号段;最后四位是用户号,代表用户的身份。但是,190、196、197号段在2017年被整体调整到了1241、1243和1242号段,因此对于这种特殊情况,需要前四位号码才能确定运营商,这几个号段后边几位规则过于特殊,我们在这里不讨论。空出来的190、196和197号段在2019年重新被下发。在十一位手机号之前,有时会有两位国家码,在打跨国(或地区)电话时需要添加,我国大陆对应的编码是86,这两位号码在大陆内手机通话时不用添加。\n\n由于手机号实名制的普及,一个手机号只对应一个身份证号,所以完全不用担心自己和别人同时使用同一个手机号造成生活不便的。至于手机号某一部分重合造成的困扰,这种问题应该主要出现在取外卖、快递等的时候,因为快递一般会有一个专属取件码发到手机上。那我们就来看看取外卖的过程:顾客用真实手机号下单,外卖员看到的是隐去了中间四位的号码,一个虚拟号和你的称呼(一般是姓氏);送餐的时候外卖员会通过系统生成的虚拟号叫顾客下楼,然后顾客报自己的真实手机号后四位,外卖员确认称呼以后把食物交给顾客。所以仅仅因为手机号后四位相同而导致取错餐的情况大概是…同一个外卖员在一趟路程送的(大约十几份)餐中,有两份(十份取两份,用组合数算概率是1/45)的顾客手机号后四位相同(万分之一的概率)且姓氏相同(估计一下应该不会比1/10更大),把括号里提到的概率乘起来,得到总的概率大约是,五百万分之一。", "task_type": {"major": ["问答"], "minor": ["知乎问答"]}, "domain": ["通用"], "metadata": "qid:505382771, aid:2266855371", "answer_from": "human", "human_verified": true, "copyright": "暂无版权及作者信息"} {"instruction": "「流星雨」这种天文现象是如何形成的?是如何起源的?", "input": "", "output": "流星雨这种现象从古代就有记录,其中我国的记载又早于其他国家,早在出土于战国\n\n时期魏国古墓的《竹书纪年》,明确记载了3000 多年前夏朝的一次流星雨事件:“夏帝癸十五年,夜中星陨如雨。”\n\n我们观测到的流星雨中,最著名的就属狮子座流星雨了,狮子座流星雨发生在每年11月中旬,平时看到的流星数目比较少,但每33年,狮子座流星雨就会迎来一场大爆发,我们对流星雨的研究也正是从记录狮子座流星雨开始的。我们记录狮子座流星雨的周期,并测量流星雨辐射点,发现其在狮子座方向,因此确定流星雨来自天外。1866年初, 暗弱的坦普尔-塔特尔彗星被发现,经过计算其绕太阳运转的轨道周期为33 年,我们发现流星的轨道与\n\n彗星轨道也有着惊人的相似。几乎同时,意大利天文学家斯基亚帕雷利通过计算,提出英仙座流星雨应该和1862年发现的斯威夫特·塔特尔大彗星有关。我们终于知道,流星雨与彗星有关。\n\n流星的现象非常简单,在外太空的尘埃等物质在地球引力下进入大气层,与大气层剧烈摩擦发光,这些物质我们称为流星体,流星体以每秒11-72公里的速度进入大气层,与空气剧烈摩擦,产生大量热,使空气发生电子跃迁,这时流星距离地表大概又80-120公里高,绝大多数流星体非常小,比沙尘还小,只会在空气中闪光一两秒钟,便燃烧殆尽了。极少数流星,例如火流星,体质量体积比较大,速度更快,产生的能量高,闪光也更强烈,可以持续数秒甚至超过10秒,有的甚至会再次碎裂发生爆闪,即便在远低于80公里以下,依然可以用肉眼看见闪光,最后燃烧不完的部分,落到地上,就是陨石。\n\n大量流星在同一段时间了出现,就是流星雨。流星雨一般不会有陨石。由于流星的轨迹是几乎平行的,沿着轨迹向远处看,近似交在一点,这个点就叫做流星雨的辐射点,辐射点在那个星座附近,我们就用那个星座的名字命名流星雨。\n\n那流星体是怎么来的呢?\n\n来自彗星。彗星的彗核是一个由冰河尘埃组成的松散的脏雪球,因此彗星在运行时,尤其是奔向太阳时,会在身后留下尘埃,这些尘埃会散落在彗星轨道上,如果地球轨道与彗星轨道有交点,在地球公转到交点附近时,便会吸引这些尘埃进入大气层,这就是流星雨的来源。", "task_type": {"major": ["问答"], "minor": ["知乎问答"]}, "domain": ["通用"], "metadata": "qid:340507948, aid:2224658510", "answer_from": "human", "human_verified": true, "copyright": "暂无版权及作者信息"} {"instruction": "为什么彗星有一条长长的尾巴?", "input": "", "output": "实际上,彗星并不是任何时候都有一条长长的尾巴。只有彗星奔向太阳的时候才会有彗尾,就是那条长长的尾巴。\n\n彗星,俗称扫帚星,中国古代称“星孛(bèi)”或“孛星”,是太阳系中一种比较奇特的成员,他们很多在扁长轨道上围绕太阳公转,称为“周期彗星”,也有一些不绕太阳运动,只是太阳系的过客。彗星的本体是由冰、一氧化碳、干冰和尘埃组成的彗核,其中冰占多数,我们称为“脏雪球”,彗核一般为1-40千米大小,质量大多在$$10^{11}-10^{16}$$ 千克范围内,也有更大或更小的。\n\n彗星的尾巴并不是时时都有,一般只有在彗星向太阳方向运动时,才会有这条尾巴。当彗星奔向太阳时,不断接近太阳,接受到的太阳辐射就更多,彗核表面的冰会升华,同时带着尘埃挥发,这就形成了彗星的大气——彗发。彗星的表面并不滑,某些区域的升华会更剧烈,升华的气体大量生成,就会产生“喷流”等近核现象。\n\n而所谓的尾巴,我们叫做彗尾,就来自彗发。在奔向太阳的过程中,由于受到太阳光的辐射压以及太阳风作用,彗发会被排斥,彗发中的尘埃和离子就会在远离太阳方向形成一道长长的尾巴,这就是彗尾。彗发和彗尾中的尘埃和离子会在太阳辐射的激发下发光,因此彗星和他的尾巴都会很亮。越靠近太阳,尾巴也越长越亮。\n\n不过当彗星离开太阳的时候,基本就是裸露的彗核了。由于彗发,彗星每次回归,尤其是在日彗距离小的时候,彗星会损失质量,损失大约在0.1%-1%之间,因此彗星的寿命比较短。如果彗核分裂或撞击到其他天体,彗星会“死”得更快。\n\n对彗星的探测也是航天航空的一共重要课题,像彗核的大小和形状、其表面和内部结构、化学成分和矿物组合、有机物种类和含量,尤其是跟生命有关的复杂有机物等问题都是地面探测很难得出结果的,因此,利用航空器对彗星进行探测和取样还是十分有必要的。\n\n我们最熟悉的彗星应该就是哈雷彗星了,他是以英国天文学家哈雷的名字命名的。1682年,哈雷彗星回归,当时,牛顿万有引力已经建立,哈雷便与牛顿对这颗彗星的轨道进行计算。1695年,他开始专心研究彗星,它不仅计算了1682年那颗彗星的轨道,还从1337到1698年的彗星中挑选了24颗彗星进行计算,并发现1531年、1607年和1682年出现的彗星轨道近乎相同,于是他想到,这是同一颗彗星的三次回归,他又继续向前搜索,发现1456年、1378年、1301年、1245年,一直到1066年,历史上都有类似大彗星的记录。于是他提出,这是同一颗彗星,并预言这颗彗星将在1758年底或1759年初再度出现。可惜,哈雷死于1742年,并没有等到哈雷彗星再次回归,但1759年3月,哈雷彗星确实应约而来。后来人们就将这颗彗星命名为哈雷彗星,其周期约为76年,但周期也会有所变化。哈雷彗星的上一次回归是1986年,再次回归要在2062年左右。", "task_type": {"major": ["问答"], "minor": ["知乎问答"]}, "domain": ["通用"], "metadata": "qid:322224904, aid:2224586056", "answer_from": "human", "human_verified": true, "copyright": "暂无版权及作者信息"} {"instruction": "如何解释「\"银河\"不是真的河」?", "input": "", "output": "银河只是一个比喻,是我们对夜晚中我们在天空中看到的那条由银河系中恒星组成的银白色光带的命名。银河是银河系的一部分,由于恒星非常密集,距离我们又非常遥远,因此无法用肉眼看清,看起来是一条乳白色的连续的光带,因此,得名银河。希腊神话故事中认为银河是天后赫拉的乳汁,因此将银河称为MilkyWay,也是同样的道理。\n\n那我们来看看为什么“银河“不是真的河?怎么定义一条河呢?或者说,怎么样才能称为一条河呢?一般来讲,我们要对事物进行分类,要抓住它的主要特征,特征相同的可以分为一类。\n\n那河有什么特征呢?\n\n由百科可知:河是一种自然形成的水道,一种水体形式。简单来说,河就是河道中含有流动的水。那我们来提炼一下河的主要特征:河道上的流水。考虑银河是天文现象,不妨更广义一些,河道上的流体,甚至再简化一些,连河道也不要,流动的连续介质。\n\n实际上,银河不满足这上面的任何一个特征。\n\n还是先来看河道。银河是没有河道的,但它外观上确实是带状。尽管太阳系处于银河系中,按理太阳系与其他星系的相对位置应该是不变的,不过由于地球公转自转的影响,银河的位置其实是会发生变化的,但你不论何时望向银河,你并不能看到其中包含的天体有什么变化。\n\n至于流动的连续介质,就更不符合了。首先,银河里的星星虽然在运动,但显然不会定向流动,他们所受的力不支持他们做出这样的选择。至于连续介质,你还别说,早先我们还真的认为银河是连续的,后来1910年,意大利天文学家伽利略·伽利莱使用望远镜进行天文观测将光带分解为单个恒星。光来源于恒星和位于银河平面方向的其他物质的积累。带周围较亮的区域表现为柔和的视觉斑块,称为星云。其中最引人注目的是大人马座星云,它是银河系中央凸起的一部分。带内的黑暗区域,例如大裂谷,是星际尘埃阻挡来自遥远恒星的光的区域。银河系遮蔽的天空区域称为回避区。\n\n因此,银河实在是跟河没啥关系,只是单纯看上去像而已,银河是我们早期对天体不够了解而得名。不过我想,以中国人骨子里的浪漫,即便了解银河的本质后,还是会起这样一个名字吧。\n\n最后简单介绍一下银河的观测。在北半球,冬季银河很暗淡,不容易观测到银河,夏季的夜晚则比较容易观测到银河,但也需要天气晴朗,一般在夜空视星等达到5.5(也就是我们能看到视星等至少为5.5的星星的夜空)以上,才能观测到银河,在视星等达到6.1时能看到大量细节。在灯火通明的城市,由于光污染,是很难看到银河的,因此你最好去乡下或旷野等人造光源不多的地方观测。", "task_type": {"major": ["问答"], "minor": ["知乎问答"]}, "domain": ["通用"], "metadata": "qid:322273129, aid:2224503055", "answer_from": "human", "human_verified": true, "copyright": "暂无版权及作者信息"} {"instruction": "为什么高铁的车头是子弹型的?", "input": "", "output": "高铁,高速铁路,就是设计标准等级高、能让列车高速运行的铁路系统。中国铁路在速度方面上分了高速铁路(250-380km/h)、快速铁路(160-250km/h)、普速铁路(80-160km/h)三级标准,其中中国的高速铁路一般采用无砟轨道,也有少部分采用有砟轨道。中国高铁线路统一运营构造速度达250km/h及以上的电力动车组列车,车次分“G、D、C字母开头”三种,车辆分CRH和CR系列车型。\n\n那高铁的车头为什么要设计成子弹型呐? 其实其目的很简单,打败其最大的对手,空气!\n\n并且其中最主要的就是压差空气阻力。当列车快速前进一段距离,车头前方的空气瞬间被挤压,来不及向周围散开,于是形成了一片高压区域。同样,车尾在快速驶离原来位置时,周围空气还没来得及填入车尾原来占据的空间,因此这里的气压比周围更低,形成一片低压区域。于是,在车头和车尾之间形成了压强差,高压区域将车向后推,低压区域将车往回拉,列车整体就受到一个从高压区指向低压区的力,即压差阻力。这个力正好与列车前进方向相反,阻碍列车前进。\n\n所以高铁选择子弹头的流线型设计,可以在列车高速前进时让车头单位面积的空气排开量减少,同时减少车尾后方单位面积的空气填充量,让车头附近的气压上升变化率和车尾附近的气压下降变化率不明显,来减小压差阻力,保证我们行驶的安全!", "task_type": {"major": ["问答"], "minor": ["知乎问答"]}, "domain": ["通用"], "metadata": "qid:487505679, aid:2223506255", "answer_from": "human", "human_verified": true, "copyright": "暂无版权及作者信息"} {"instruction": "为什么月亮又圆又大的晚上,星星特别少?有什么科学原理", "input": "", "output": "这就是“月明星稀”啊。\n\n一个直观的原因就是,满月实在是太亮了,尽管月亮本身并不发光,但满月离地球太近,仅靠反射太阳光就成为我们在夜晚的空中能看到的最亮的天体。\n\n我们用视星等来描述我们在夜晚看到天空中的天体的亮度。星等并不仅仅取决于天体自己的发光能力,还与其离地球的距离有关。\n\n星等最早由希腊天文学家喜帕恰斯(Hipparchus)提出,他将夜空中的天体分为六等,一等最亮,六等最暗。现代天文学便沿用了这一分级方式。我们发现一等亮度的星星比六等亮度的星星亮了大约100倍,因此,我们定义相邻两个星等的亮度差距为$$100^{1/5}$$,例如二等星比三等星亮了大约$$100^{1/5}$$,即约2.512倍。当然,更亮的星星可以出现零星等或负星等。至于星等零点,美国哈弗大学天文台以小熊座λ星的星等为6.55来规定星等零点。我们用m表示星等,两颗天体的星等差距为:\n\n$$m_2-m_1=-2.51 \\lg \frac{E_2}{E_1}$$其中E为两颗天体的照度,即发光体在某一距离处单位面积上的光照强度。\n\n再次说明,星等数值越小,天体越亮。\n\n当然,视星等只是对地球观察者而言的相对观念,并不能真正反映天体亮度,因此由绝对星等的概念。天文学规定把天体置于距离观察者10 pc(parallaxsecond,是天体物理长度单位,秒差距,其长度为32.616光年)时,所计算出来的视星等定义为该天体的绝对星等。下文我们叙述的星等为视星等。\n\n夜空中我们能看到的星体最暗为6m(也有说6.5m),当然这也与个体差异、天气状况等有关。而大家认为比较亮的星体——北极星的星等约为1.98.而我们能看到的的最亮的星为天狼星,视星等为-1.47.而我们能看到的星星总计也只有6000余颗,其中由约4800颗都是六等星。一等星以及比一等星更亮的星一共只有不到30颗。但是满月的视星等为-12.9,太阳的视星等-26.74.这意味着,满月比夜空中最亮的星——天狼星——亮了37000多倍。而太阳比满月还要亮340000多倍,这就是我们白天看不到任何星星的原因,太阳公公一枝独秀啊。但新月的视星等只有-2.5.只比天狼星亮了大概2.58倍。值得一提的是金星最亮时视星等可以达到-4.9.也是夜空中非常亮的星。\n\n因此,满月相对其他星星,实在是太亮了。而我们能不能看清楚物体主要看对比度(又叫衬比度),他反映了相近两块区域内的光强最大值和最小值的差距:\n\n$$V=\\frac{I_{max}-I_{min}}{I_{max}+I_{min}}$$\n\n$$I_{max}$$ 与 $$I_{min}$$分别为相邻区域光强的最大值与最小值,可见,如果相邻区域光强最大值与最小值的差距越大,我们的视觉效果就越好,反之,视觉效果就越差。而满月时,由于月亮亮度实在太高,导致整个天空都比较亮,背景亮度就很高,即满月时上式中$$I_{min}$$ 很高,那对于相同的星星(即 $$I_{max}$$ 不变),对比度就明显下降,因此我们视觉效果就很差,所以有些视星等低的星星(即$$I_{max}$$ 本来就不高)就看不见了。而白天时,由于太阳更亮,星星所在区域的背景亮度可能 $$I_{min}$$比星星都亮,因此,白天几乎完全看不到星星。\n\n另一方面,当有强光刺激时,人的瞳孔会缩小,进入眼睛的光线会减少,以保护眼睛。而当满月出现在天空中时,人眼就会主要接受来自月亮的光线,瞳孔收缩,接受的一些六等星的光变少,可能就看不见了,上文也提到了,我们能看到的星星中,绝大多数都是勉强能看见的六等星,因此,满月的天空中我们能看到的星星确实变少了。就像白天时我们看不到任何星星一样。\n\n还有一种可能的原因是马赫带效应。这是人眼的一种视错觉效应。它是指当亮度从一个亮度平台过渡到另一个亮度平台时,在较高亮度平台和过渡带的边缘会出现一条比亮平台更亮的亮条,而在较低亮度平台和过渡带的边缘会出现一条比暗平台更暗的暗条,物理上检测不出这两个条带的存在,它们的存在实际上是人眼的视觉特性造成的。简单来说就是,亮的更亮,暗的更暗。\n\n因此当满月出现在夜空中的时候,马赫带效应会导致月亮看起来更亮,而星星看起来更暗,因此视觉上,满月出现的时候,星星更少。\n\n综上,月亮又大又圆的夜空中,我们能看到的星星确实更少,这个现象其实很早之前我们就发现了。曹丞相就有诗:月明星稀,乌鹊南飞。绕树三匝,何枝可依。", "task_type": {"major": ["问答"], "minor": ["知乎问答"]}, "domain": ["通用"], "metadata": "qid:487505928, aid:2222999960", "answer_from": "human", "human_verified": true, "copyright": "暂无版权及作者信息"} {"instruction": "为什么月亮每天都在变化", "input": "", "output": "因为月亮靠反射太阳光发亮,每天月亮被太阳照到的地方都会发生变化,即反射太阳光的部分变化,月亮也就呈现出不同的形状,这些不同的形状便是月相。\n\n为了更好地理解月相的变化,我们需要补充一些天文小知识。在天文学中,为了方便描述太阳系中天体的位置,有时会用到黄道坐标系。把地球保持球心与自转轴不变,无限放大后得到一个球体(天文学上称之为天球),以黄道为基圈,以经过春分点的黄经圈为始圈,以春分点为原点组成的天球坐标系便是黄道坐标系;可以简单理解成以黄道面为纬度0°重建的坐标系。黄道坐标系的经度叫做黄经,两个天体的黄经的度数差便是黄经差。\n\n当月球与太阳的黄经差为0°时,月球恰好处在地球与太阳连线上,在地球上几乎无法看到月亮,此时的月相称为朔月,此时对应农历的初一;月球绕地球公转,当转到月球与太阳黄经差为90°和270°时,地月连线与地日连线垂直,恰好能看到一半的月亮,此时的月相为上弦月和下弦月,分别对应农历的初八左右与二十三左右;当月球与太阳黄经差为180°时,月球处在日地连线的延长线上,此时能看到满月,称为望月,通常在农历十五左右出现。\n\n从一个朔月到下一个朔月的变换周期为29.5天,称为朔望月。农历上说的一个月指的就是一个朔望月。", "task_type": {"major": ["问答"], "minor": ["知乎问答"]}, "domain": ["通用"], "metadata": "qid:487505941, aid:2219719864", "answer_from": "human", "human_verified": true, "copyright": "暂无版权及作者信息"} {"instruction": "为什么是红灯停,绿灯行?为什么不是其他颜色?", "input": "", "output": "“红灯停,绿灯行。”从小听到大的口诀一直到现在也在被忠实地执行,这样的规则既是传统,也是科学。\n\n从科学角度来讲,首先来理解为什么信号灯要选择红黄绿三种颜色:\n\n1\\. 人类的视网膜含有三种锥状和杆状的感光细胞, 三种锥状细胞对红、绿光最敏感, 而杆状细胞则对黄色的光特别敏感,也就是说人们对这三种颜色最为敏感;\n\n2\\. 红黄绿三色的波长比其他颜色的长,更具穿透力,在雾天等恶劣天气下也能传播出较远距离,保持正常工作。\n\n下面再来看为什么是红停绿行呢?\n\n从心理角度来说, 红色象征危险, 绿色能给人安全感,红灯停绿灯行符合人类的行为习惯。各种安全通道指示灯也是绿色,绿色代表了通行。\n\n从传统来讲,早在1868年就有了第一个红绿交通信号灯,红灯停绿灯行已经是一种惯例。\n\n中国历史上也曾有人想要把这个规则反过来,即实现“红灯行,绿灯停”。《周恩来世纪行》一书记载道:“他们通令要把交通指挥信号由绿灯行、红灯停改成红灯行、绿灯停,还要让车辆一律由原来的靠右行改成靠左行。”“此事被周总理知道后,他在百忙中来到西城区的一所中学,周总理对他们说:‘交通指挥信号绿灯行、红灯停是国际惯例,是有科学依据的,红光穿透力强,无论雨天雾天,驾驶人员都能远远地看见红灯。因此,不能随意改变现行信号和交通规则。’”\n\n黄色加入信号灯的起源众说纷纭,这里介绍其中一种:黄灯由中国人胡汝鼎在美国留学时发明,一次他走在街上时,绿灯亮后正要继续前行,一辆转弯的汽车突然擦身而过吓了他一身冷汗,于是他产生了用黄色信号灯的想法,用来在红灯向绿灯的转换中缓冲,提醒大家注意警戒。", "task_type": {"major": ["问答"], "minor": ["知乎问答"]}, "domain": ["通用"], "metadata": "qid:23672758, aid:2218207925", "answer_from": "human", "human_verified": true, "copyright": "暂无版权及作者信息"} {"instruction": "如何回答孩子问我「为什么每年过年的日子不一样」?", "input": "", "output": "其实每年过年的日子是一样的呀!\n\n都是“一月初一”呢!\n\n不信?妈妈带你翻看一下日历:每年春节的那一天的日历,看看大数字下面的小中文,标注的就是当天的“农历日期”:写着“春节”这一天的,后面一天就都会写着“初二”;说明春节就是“初一”咯!\n\n春节是一个“农历”的节日,按照“农历”来说,其实每年的日期都是一样的。\n\n当然,宝宝,我知道,在你所看来,每年过年都是不同的日子,有时候是一月,有时候在二月。这就是因为我们平时记录时间的方法,用的不是和春节相关的那个“农历”,而是日历上的大字——“阳历”,这也是我们现在世界上最广泛使用的方法,学校上学安排啊、公司工作中的交流啊,说的都是阳历的日子。阳历和农历的计算方法有不同,所以每一年的春节,对于农历是个固定的日子,但是对于阳历就各不相同了。\n\n说到这里,你肯定会问,农历阳历又分别是什么,为什么会有两个记录时间的方法呢?\n\n其实要严格说起来,从古到今,人们主要三个不同的历法:阴历、阳历和农历。大家都知道,地球绕着太阳公转,月亮绕着地球公转,而地球自己还会自转这个道理对吧,历法就是和这些转动有关。\n\n简单来说:\n\n- 阳历:又叫公历,也是太阳历。这是把地球围绕太阳公转一圈的时间作为“一年”为基础,然后把这一年划分成365天和12个月,但这里的每个 月,和月亮围绕地球的转动完全没有关系。\n- 阴历:是以月亮的公转为核心的。把月亮绕着地球转一圈的时间(大概是29.5天)作为一个基础“月”,12个月为一年。阴历只关心月亮,完全不考虑地球围绕太阳的公转。\n- 农历:可以看做一种阴阳结合的历法。很多人认为阴历就是农历,其实这是不准确的。其实阴历和阳历每年有十多天的差,这样如果不加以调整,很快两种历法计时,就能差一年乃至数年了。但是这种情况没有发生,因为中国农历有闰月,通过19年中7个闰月,调整了阴阳历的时间差,保持了二者的同步。另外呢,农历还 在阴历的基础上,考虑了太阳的位置,把一年分为24个节气来指导农业生产。这24个节气,到今天依然被人们广泛使用呢!\n\n我们每天都在用的日历,背后其实还有这么多学问呢呀!亲爱的孩子,这下你知道了吧,当我们在同一个日子的时候,这个日子会有两个不同的表达方法:阳历是一个,农历是一个。而这两种,在日历上都可以发现。\n\n接下来我们不妨翻翻日历一起来做些小探索吧\n- 找找你的生日,那一天是农历的什么日子吧?\n- 9月1日是开学的日子,这在农历里,又是哪一天呢?\n- 24个节气你最熟悉哪几个?每当到这个节气的时候,我们会做什么特别的事情吗?", "task_type": {"major": ["问答"], "minor": ["知乎问答"]}, "domain": ["通用"], "metadata": "qid:487505729, aid:2164008384", "answer_from": "human", "human_verified": true, "copyright": "暂无版权及作者信息"} {"instruction": "为什么有时候会早上在东边看到月亮?", "input": "", "output": "日出月落的规律并非绝对,日月同辉也并不罕见。\n\n由于地球自转的同时,月球也在绕地球公转,每天月亮升起的时间都会比前一天延后一段时间。根据timeanddate网站提供的数据,每天的延后时间也有所不同,会根据月球在公转轨道的位置不同在20-70分钟的区间内波动。\n\n所以理论上,月亮必然会有几天和太阳差不多同时升起,也就是早上月亮出现在东方。\n\n但是要观察到东方的月亮是有难度的。要在白天看到月亮有两个条件:一是白天时月亮仍在地平线以上,且高度足够高不被建筑阻挡;二是月亮的亮度足够,不会被太阳的光芒掩盖。刚刚我们只是满足了第一个条件,白天时月亮在地平线以上;但是月亮在早上出现在东方一般是在农历初二左右,此时月相在朔月与峨眉月之间,地球上只能看到很小一半的月亮,而且月亮同时与太阳在东方的天空,看到月亮的难度之大可想而知。\n\n一般来说,初八前后下午的东南方位,二十三前后上午的西南方位就比较容易看到月亮。也就是上下弦月的时候。\n\n一方面,出现弦月时,月球与太阳的黄经差恰好是90°(270°),地月连线与地日连线垂直,月亮与太阳的升落恰好差了半天,所以总有半天左右的时间太阳和月亮是同时出现在天空的;另一方面,上下弦月的月相较大,反射太阳光较多,亮度较高,当太阳光不是很强的时候便能够看到。这便是前面所说初八以及二十三前后容易在白天看到月亮的原因。", "task_type": {"major": ["问答"], "minor": ["知乎问答"]}, "domain": ["通用"], "metadata": "qid:492845036, aid:2178425322", "answer_from": "human", "human_verified": true, "copyright": "暂无版权及作者信息"} {"instruction": "为什么有些材质的面料会缩水?", "input": "", "output": "几乎所有的纤维织物在初次洗涤之后都会发生一定程度的收缩。这是因为,织物纤维都是高分子有机物的长链编织而成,而这些有机物长链,在其自由能最低的状态下都是卷曲起来的。为了织出挺直的衣物,在纺织天然纤维或是合成人造纤维的过程中,都会加热并拉伸纤维。卷曲的纤维在加热过程中舒展开来,在外力的作用下进一步拉直,冷却过程中纤维之间形成新的键联,伸直的形态就被固定下来。\n\n在初次洗涤的过程中,水分子进入到织物纤维的间隙之间,削弱了纤维分子的相互作用,绷紧的纤维分子于是纷纷松弛下来,晒干后衣物的尺寸便减小了。洗涤过程中衣物受到的外力捶打,也有可能使纤维分子间的键联断裂,导致纤维松弛收缩。除此之外,羊毛织物在水洗过程中还可能出现“缩绒性”,这是因为毛发表面覆盖着一层层角质蛋白构成的鳞片,在纺织过程中鳞片间隔被拉大,水洗过程中收缩,这也会导致织物缩水。\n\n总而言之,洗涤过程中水分子进入纤维间隙,织物纤维分子间的键联被削弱,纤维发生卷曲和收缩,这是造成织物缩水的主要原因。而纤维分子间的键联削弱则主要是水分子进入分子间隙引起的。因此,虽然不同材质的衣物都可能发生缩水,纤维排列疏松的织物,如羊毛、纯棉衣物的缩水更为明显。", "task_type": {"major": ["问答"], "minor": ["知乎问答"]}, "domain": ["通用"], "metadata": "qid:38941332, aid:1862424369", "answer_from": "human", "human_verified": true, "copyright": "暂无版权及作者信息"} {"instruction": "月球上会有极光现象吗?", "input": "", "output": "可以产生极光。\n\n很容易想到的是地球上极光的产生,太阳风带来的高能带电粒子,与地球大气分子碰撞,激发原子中的电子从低能级跃迁到高能级甚至形成等离子体,当电子从高能级跃迁回低能级时会发出不同能量的光子(例如激发的原子氧发出630nm红色的光,激发的氮分子会发出428nm蓝色的光,还有在紫外和红外的不可见的光等),地磁场参与其中将极光的产生限制在地球的南北极,从而形成我们在地球上能够看到的极光。极光的产生本质是能级跃迁,地球上的极光产生是由太阳风、地磁场和大气层共同作用形成。\n\n太阳系中大多数行星,一些天然的卫星、褐矮星、甚至彗星是有极光的。不同行星上的极光产生原因有所不同,太阳风的对磁场的扰动可以让带电粒子下降到大气中,使得大气中的分子发生电离和激发发出光,木星上的磁场(4.3高斯)比地球磁场(0.4高斯)更强,极光的产生类似于地球,由太阳风驱动,但木星上具有更复杂的等离子体传输机制;金星上没有磁场,极光的产生是太阳风和大气中的分子碰撞产生;彗星中含有的水电解,产生氢和氧,与太阳风碰撞也可以产生极光。由此看来,极光的产生主要因素是太阳风带来的高能粒子(在太阳系中宇宙背景中的高能粒子比太阳风中带来的要少)和星体周围的气体(大气),磁场并不是产生极光的必要条件。\n\n月球可以接收到太阳风的高能粒子外,周围也是含有稀薄的大气,当然相比地球大气中的分子要少得多。太阳风与月球表面的物质相互作用、表面物质的蒸发、彗星等对月球的碰撞,都可能会给月球带来大气,因此月球上存在产生极光的条件,只不过月球极光的光强比较低,观测有困难。", "task_type": {"major": ["问答"], "minor": ["知乎问答"]}, "domain": ["通用"], "metadata": "qid:296995724, aid:1765798918", "answer_from": "human", "human_verified": true, "copyright": "暂无版权及作者信息"} {"instruction": "白洞和黑洞是什么关系?", "input": "", "output": "黑洞是爱因斯坦广义相对论中预言存在的一种天体,但是与宇宙中的其他天体相比有所不同的是,黑洞是由恒星等大质量物体的引力坍缩形成的,由于其质量大,体积小,黑洞表面的逃逸速度大于光速,也就是说,任何运动速度小于光速的物体都无法逃离黑洞的引力而只能围绕黑洞旋转而不能离开。但是,其实黑洞并不“黑”,只是光不能从中逃逸使得我们不能直接观测到黑洞的存在,可我们看不到并不代表黑洞就不存在,就像黑夜中的一堵墙,我们看不到墙的存在,但当我们走过去的时候仍然能感受到墙在阻止我们继续前进。同样的,科学家们通过探测黑洞与其他物质间的相互作用产生的辐射等信号,可以间接推断出黑洞的相关信息,这也是我们得以在去年看到黑洞的照片的原因。\n\n与已经间接观测到的黑洞相比,白洞则更加神秘。“白洞,白色的明天在等着我们!”也许这是一部分人第一次接触到白洞这个词,童年武藏和小次郎的反派台词还可以清晰回忆起来。但是,白洞里真的会有白色的明天在等着我们吗?到目前为止,白洞还只是一个理论模型,并未被证实存在。正如白洞的名字,它的某些性质与黑洞正好相反。白洞是一个强引力源,其外部引力性质均与黑洞相同,而内部性质却截然相反,黑洞能吞噬邻近宇宙区域的所有光线和任何物质,但是按照白洞理论,白洞可以把它周围的物质吸积到边界上形成物质层,可以向外部区域提供物质和能量,但不能吸收外部区域的任何物质和辐射,所以白洞是一个只发射、不吸收的特殊宇宙天体。对应于黑洞的从有到无,白洞的从无到有显然更神秘也更令天文学家们费解,目前白洞对我们而言依旧是个未解之谜。", "task_type": {"major": ["问答"], "minor": ["知乎问答"]}, "domain": ["通用"], "metadata": "qid:399834577, aid:1278016796", "answer_from": "human", "human_verified": true, "copyright": "暂无版权及作者信息"} {"instruction": "一个一块钱硬币的材料成本是多少?", "input": "", "output": "我国目前共发行了五套流通人民币,从第二套流通人民币开始有一元硬币,除了第二套流通硬币的一元硬币的材质是铜镍合金,其他套一元硬币的材质均为钢芯镀镍。\n\n简单来说,可以从三种角度评价一枚硬币的价值:\n\n 1. 原值(Intrinsic Value),即硬币所含金属(特别是贵金属)本身的价值; 2. 面值(Face Value),即硬币币面上所标明的价值; 3. 现值(Market Value),指的是硬币在市场交易的价值。\n\n国家在发行货币的时候,会事先规定好货币的面值。由于货币的面值往往远高于原值,所以在市场上能否以面值流通,需要国家以国家信用为担保。货币面值与原值的差即铸币税,铸币税是政府财政的重要来源。\n\n一元硬币实际的制作成本国家是保密的,我们并不知情。但是如果硬币的实际成本超过面值的话,那么把硬币花掉,明显不如积攒下来更加有利。此时该硬币会很快退出流通,而变成收藏品。所以,多数硬币都不可能拥有超过面值的成本或者价值。当然,有时也有流通中的硬币原值超过面值的情况。这时,大家就会把高价值的硬币储存起来,在市场上使用等面值的低价值产品来进行交换,这与经济学中“劣币驱逐良币”的理论相一致。这样一来,该硬币本质上变为了收藏品。", "task_type": {"major": ["问答"], "minor": ["知乎问答"]}, "domain": ["通用"], "metadata": "qid:21341963, aid:1277895465", "answer_from": "human", "human_verified": true, "copyright": "暂无版权及作者信息"} {"instruction": "请问汽车驾驶员用的偏光眼镜能防强烈日光、能透过对向车挡风玻璃看到车内物体是什么原理?", "input": "", "output": "首先要认识到,光无非是以波动形式传播的电磁场。根据麦克斯韦电磁理论,光的电磁场振动方向和能量的传播方向垂直,属于横波。作为类比,想象你拿着一根长绳并摇动它的一端,绳上便会有起起伏伏的“波动”,这就是一种横波。由于光电场的振动方向只需要在垂直于光线的平面内,所以它可以沿着不同的方向振动,这就是光的偏振。自然光的电场是大量随机取向振动的电场的集合,但经过镜面反射后的光可能只有特定振动方向的电磁场。\n\n而偏光眼镜镜片的材料由沿一定方向排列的分子组成,电子的振动方向受到限制。当电磁场与其相互作用时,沿这一方向振动的电磁场与材料中的电子相互作用,电子便开始振动,同时带动临近的电子沿统一取向振动,使得电磁场具有的能量被无数取向一致的电子传递吸收。而垂直于材料分子取向振动的电磁场则不会被吸收。所以,只有某一偏振方向的光可以通过。可以想象透过栏杆传递一本书,当书本横着传递时会被栏杆挡住,竖着传递时便可以轻易穿过栏杆。\n\n对于自然光,譬如日光,由于它本身具有沿所有方向的偏振光,所以总会透过偏光镜片,但同样有一部分透不过,于是透过的光比起入射的光强度会降低约一半。但是反射光常常有很大一部分具有单一的偏振取向,透过偏光眼镜的比例很低,于是可以达到消除反射光的目的。对于车内的物体,由于一般物体表面不是镜面,发生的是漫反射,于是在不同条件的反射下各种偏振方向的光都有,也就可以通过偏光眼镜看到了。", "task_type": {"major": ["问答"], "minor": ["知乎问答"]}, "domain": ["通用"], "metadata": "qid:529469035, aid:2452705374", "answer_from": "human", "human_verified": true, "copyright": "暂无版权及作者信息"}